Download Section II

Document related concepts

Gene therapy of the human retina wikipedia , lookup

Bio-MEMS wikipedia , lookup

Organ-on-a-chip wikipedia , lookup

Transcript
Professor Anwar Sheikha, MD, FRCP, FRCPath., FACP
One Thousand Hematology MCQ
ONE THOUSAND
HEMATOLOGY
MULTIPLE CHOICE QUESTIONS
EXAMINATION
BANK
Professor Anwar Sheikha
MD, FRCP, FRCPI, FRCPA, FCAP, FRCPath., FACP
Department of Pathology
University of Mississippi Medical Center
Jackson, MS 39216, USA
Professor of Clinical & Laboratory Hematology
College of Medicine
Salahaddin University
Erbil, Kurdistan, IRAQ
Senior Consultant Clinical Hematologist
Blood Hospital, Hawler, Kurdistan, IRAQ
C.E.O., Medya Medical & Cancer Center
Hawler, Kurdistan, IRAQ
1
Professor Anwar Sheikha, MD, FRCP, FRCPath., FACP
One Thousand Hematology MCQ
ONE THOUSAND
HEMATOLOGY
MULTIPLE CHOICE QUESTIONS EXAMINATION BANK
Professor Anwar Sheikha
MD, FRCP, FRCPI, FRCPA, FCAP, FRCPath., FACP
Select the most appropriate single answer:
Q 1. The early myeloid cell shown with agranular pale blue cytoplasm and a large
multi-nucleolated nucleus is called:
(A)
Myeloblast
(B). Promyelocyte
(C). Myelocyte
(D). Metamyelocyte
(E). Band Form
Q 2. The large myeloid cell shown with both nucleoli and cytoplasmic granules is
called:
(A). Myeloblast
(B)
Promyelocyte
(C). Myelocyte
(D). Metamyelocyte
(E). Band Form
Q 3. The myeloid cell shown with no nucleoli but cytoplasmic granules is called:
(A). Myeloblast
(B). Promyelocyte
(C)
Myelocyte
(D). Metamyelocyte
(E). Band Form
Q 4. Leukemia of the mature segmented neutrophils and maturing myeloid cells
shown is called:
(A). M1 (Acute Myeloblastic Leukemia without maturation)
(B). M3 (Acute Hypergranular Promyelocytic Leukemia)
(C)
CML (Chronic Myeloid Leukemia)
(D). M5 (Acute Monoblastic Leukemia)
(E). M7 (Acute Megakaryoblastic Leukemia)
Q 5. In the FAB classification of acute leukemias, leukemia of the early myeloid cell
shown with an agranular pale blue cytoplasm, a large multi-nucleolated nucleus
and an Auer rod is called:
(A)
M1 (Acute Myeloblastic Leukemia without maturation)
(B). M3 (Acute Hypergranular Promyelocytic Leukemia)
(C). M4 (Acute Myelomonocytic Leukemia)
(D). M5 (Acute Monoblastic Leukemia)
(E). M7 (Acute Megakaryoblastic Leukemia)
2
Professor Anwar Sheikha, MD, FRCP, FRCPath., FACP
One Thousand Hematology MCQ
Q 6. In the FAB classification of acute leukemias, leukemia of the large myeloid
cell shown with both nucleoli and cytoplasmic granules is called:
(A). M1 (Acute Myeloblastic Leukemia without maturation)
(B)
M3 (Acute Hypergranular Promyelocytic Leukemia)
(C). M4 (Acute Myelomonocytic Leukemia)
(D). M5 (Acute Monoblastic Leukemia)
(E). M7 (Acute Megakaryoblastic Leukemia)
Q 7. Leukemia of morphologically mature small lymphocytes is called:
(A). AML (Acute Myeloblastic Leukemia)
(B). ALL (Acute Lymphoblastic Leukemia)
(C). CML (Chronic Myeloid Leukemia)
(D)
CLL (Chronic Lymphocytic Leukemia)
(E). Multiple Myeloma
Q 8. Hemolytic anemia secondary to Glucose-6-Phosphate Dehydrogenase
deficiency could present itself in any of the following forms EXCEPT:
(A). Intravascular hemolysis secondary to antimalarial medications
(B). Favism
(C)
Pancytopenia
(D). Blister red cells in the blood smear from biting-out of Heinz bodies
(E). Neonatal Jaundice
Q 9. One of the following features relate to warm autoimmune hemolytic anemia:
(A)
Positive Coomb’s test
(B). Pancytopenia
(C). Elliptocytosis in the blood smear
(D). Blister red cells
(E). Teardrop red cells
Q 10. What type of red cell abnormalities are expected in iron deficiency anemia?
(A). Macrocytic oval red cells with hypersegmented neutrophils
(B). Normocytic, normochromic red cells
(C). Spherocytosis
(D)
Hypochromic, microcytic red cells
(E). Fragmentation of the red cells
Q 11. What type of red cell morphological abnormalities are expected in pernicious
anemia (megaloblastic anemia)?
(A). Oval macrocytes with hypersegmented neutrophils.
(B). Normocytic, normochromic red cells
(C). Spherocytosis
(D). Hypochromic, microcytic red cells
(E). Fragmentation of the red cells
Q 12. What type of red cell morphological abnormality is expected in both hereditary
spherocytosis and warm autoimmune hemolytic anemia?
(A). Macrocytic oval red cells with hypersegmented neutrophils
(B). Normocytic, normochromic red cells
(C)
Spherocytosis
(D). Hypochromic, microcytic red cells
(E). Fragmentation of the red cells
3
Professor Anwar Sheikha, MD, FRCP, FRCPath., FACP
One Thousand Hematology MCQ
Q 13. What is the most significant feature of ITP (Immune Thrombocytopenic Purpura)?
(A). Bleeding with the bone marrow full of blasts
(B). Philadelphia chromosome
(C). Hairy lymphocytes in the blood smear
(D). Pancytopenia with empty bone marrow
(E)
Severe thrombocytopenia with purpuric mucocutaneous bleeding
Q 14. What is the most significant feature of aplastic anemia?
(A). Bleeding with the bone marrow full of blasts
(B). BCR-ABL transcription gene
(C). Hairy lymphocytes in the blood smear
(D)
Pancytopenia with empty bone marrow
(E). Severe thrombocytopenia due to immune destruction.
Q15. What is the most significant diagnostic feature of Chronic Myeloid Leukemia?
(A). Marked lymphadenomegaly
(B)
BCR-ABL transcription gene
(C). Hairy lymphocytes in the blood smear
(D). Pancytopenia with empty bone marrow
(E). Severe thrombocytopenia with purpuric bleeding.
Q 16. Priapism (painful persistent penile erection) could be seen in the following
hematological disorders:
(A). Sickle cell disease
(B). Chronic myeloid leukemia
(C). Acute leukemia with very high blast cell count
(D)
All of the above
(E). None of the above
Q 17. One of the following laboratory features relates to - thalassemia major:
(A)
Very high hemoglobin F
(B). Very high Hemoglobin S
(C). High hemoglobin A2
(D). High hemoglobin C
(E). High hemoglobin E
Q 18. One of the following features specifically relates to - thalassemia minor:
(A). Very high hemoglobin F
(B). Very high Hemoglobin S
(C)
High hemoglobin A2
(D). High hemoglobin C
(E). High hemoglobin E
Q 19. All of the following features relate to - thalassemia major Except:
(A). Huge splenomegaly
(B)
Splenic atrophy
(C). Bone marrow expansion
(D). Iron overload
(E). Hypochromic microcytic red cells with target & nucleated red cells
4
Professor Anwar Sheikha, MD, FRCP, FRCPath., FACP
One Thousand Hematology MCQ
Q 20. All of the following features relate to Sickle Cell Disease Except:
(A). Splenic atrophy
(B). Painful vaso-occlusive crises
(C). Hemolytic anemia
(D). Very high hemoglobin S
(E)
Positive Coomb’s test
Q 21. One of the following features is specific for Hemophilia A:
(A)
Very low level of Factor VIII:c
(B). Very low level of von Willebrand Factor
(C). Prolonged bleeding time
(D). Severe thrombocytopenia
(E). Very low level of Factor IX
Q 22. One of the following features is specific for Christmas Disease:
(A). Very low level of Factor VIII:c
(B). Very low level of von Willebrand Factor
(C). Prolonged Bleeding Time
(D). Severe thrombocytopenia
(E)
Very low level of Factor IX
Q 23. One of the following features is specific for von Willebrand disease:
(A). Very low level of factor VIII:c
(B)
Very low level of von Willebrand factor
(C). Normal bleeding time
(D). Severe thrombocytopenia
(E). Very low level of Factor IX
Q 24. One of the following features relates to ITP (Immune Thrombocytopenic Purpura):
(A). Very low level of Factor VIII:c
(B). Very low level of von Willebrand Factor
(C). Normal bleeding time
(D)
Severe thrombocytopenia with increased marrow megakaryocytes
(E). Very low level of Factor IX
Q 25. Acute Childhood ITP (Immune Thrombocytopenic Purpura) has all of the
following features EXCEPT:
(A). Usually self-limiting
(B). Usually follows a viral infection
(C)
Usually affects girls
(D). Usually associated with increased marrow megakaryocytes
(E). Usually associated with prolonged bleeding time
Q 26. Prothrombin Time (PT) is prolonged in:
(A). Severe thrombocytopenia
(B). Hemophilia A
(C). Christmas disease
(D). von Willebrand disease
(E)
Overanticoagulation with Warfarin
5
Professor Anwar Sheikha, MD, FRCP, FRCPath., FACP
One Thousand Hematology MCQ
Q 27. Activated Partial Thromboplastin Time (APTT) could be prolonged in all of the
following bleeding disorders EXCEPT:
(A)
Severe thrombocytopenia
(B). Hemophilia A
(C). Christmas disease
(D). Severe von Willebrand disease
(E). Factor XII deficiency
Q 28. Bleeding Time (BT) is prolonged in:
(A)
Severe thrombocytopenia
(B). Hemophilia A
(C). Christmas disease
(D). Factor XII deficiency
(E). Factor VII deficiency
Q 29. Thrombocytopenia could result from any of the following conditions EXCEPT:
(A). Aplastic anemia
(B). Acute myeloid leukemia
(C). Hypersplenism
(D)
Iron deficiency anemia
(E). Breast cancer infiltrating the bone marrow
Q 30. All of the following conditions can cause DIC (Disseminated Intravascular
Coagulation) EXCEPT:
(A)
ITP (Immune Thrombocytopenic Purpura)
(B). Brain injury
(C). ABO incompatible blood transfusion
(D). Severe sepsis
(E). Snake bite
Q 31. One of the following features is specific for chronic myeloid leukemia:
(A)
BCR-ABL transcription gene
(B). Alcohol-induced lymph node pain
(C). Lymphadenopathy
(D). High Neutrophil Alkaline Phosphatase
(E). Low serum B12
Q 32. Anemia of chronic lymphocytic leukemia could result from any of the following
causes EXCEPT:
(A)
Fragmentation of the red cells
(B). Autoimmune hemolysis
(C). Splenomegaly
(D). Bone marrow infiltration
(E). Folate deficiency
Q 33. In the Western communities, the chance of finding a fully histocompatible stem
cell donor for transplantation is:
A.
~100%
B.
~50%
C
~25%
D.
~10%
E.
~05%
6
Professor Anwar Sheikha, MD, FRCP, FRCPath., FACP
One Thousand Hematology MCQ
Q 34. The ideal management for a 34 year old Middle Eastern prince with relapsing
diffuse large B- cell lymphoma presenting with generalized lymphadenopathy,
marrow and blood infiltration is:
A.
Wait and watch policy
B.
Local radiation therapy
C.
Rituximab
D.
R-CHOP
E
Hematopoietic stem cell autotransplantation
Q 35. The ideal management for a 54 year old patient presenting with diffuse large Bcell lymphoma extensively involving the mediastinum, both cervical lymph nodes,
and the spleen is:
A.
Wait and watch policy
B.
Local radiation therapy
C.
Rituximab
D
R-CHOP followed by mediastinal irradiation
E.
Hematopoietic stem cell autotransplantation
Q 36. The ideal management for a 54 year old patient with stage IV diffuse large B- cell
lymphoma presenting with generalized lymphadenopathy, marrow and blood
infiltration is:
A.
Wait and watch policy
B.
Local radiation therapy
C.
Rituximab
D
R-CHOP
E.
Hematopoietic stem cell autotransplantation
Q 37. A 40 year acute myeloid leukemia patient with hyperleukocytosis and poor
karyotypic features failed to go to remission after two courses of induction. He
partially responded to anti CD33 Mylotarg “Ozagamicin”. He has a monozygotic
twin brother, two fully matched siblings and many family members who are one
antigen mismatch. Patient was advised for peripheral stem cell transplantation.
To give the best chance for cure in this patient, what type of stem cell
transplantation you will chose?
A.
Syngeneic transplantation from the monozygotic twin brother
B
Allogeneic transplantation from one of the fully-matched siblings
C.
Autotransplantation after high dose chemotherapy
D.
Cord blood transplantation
E.
No transplantation
Q 38. All but one of the followings is true in one antigen mismatched stem cell
transplantation:
A.
Survival is as good as fully matched transplants in acute myeloid leukemia
B.
There is more chance for GvHD “Graft versus Host Disease”
C.
It is associated with more Graft versus Tumor effect
D
Preferable to fully matched transplants in aplastic anemia
E.
Preferable to matched unrelated donors in chronic myeloid leukemia
7
Professor Anwar Sheikha, MD, FRCP, FRCPath., FACP
One Thousand Hematology MCQ
Q 39. Compared to bone marrow transplantation, peripheral hematopoietic stem cell
transplantation has all but one of the following criteria:
A.
Has more committed progenitor cells
B.
Results in a more rapid engraftment
C.
Associated with a better survival
D.
Easier on the donor
E
Easier on the recipient
Q 40. Washed red cell transfusion is indicated in one of the following situations:
A.
Transfusion-dependent thalassemia major
B.
Sickle cell disease
C.
Aplastic anemia
D.
Marrow or peripheral stem cell transplant patients
E
IgA deficient patient needing blood transfusion
Q 41. All but one of the following features relate to Gastric MALToma:
A.
In >90% of patients association with H. Pylori infection is seen
B
t(11;18) detected in up to 40% of patients usually indicates good response
to anti H. pylori treatment
C.
Initially one or two courses of anti H. pylori could effectively put many
patients into remission
D.
Gastrectomy could be curative if the disease is localized to stomach
E.
R-CHOP is the current treatment of choice for patients needing
chemotherapy
Q 42. All but one of the following features could be manifestations of paroxysmal
nocturnal hemoglobinuria:
A.
Intravascular hemolytic anemia
B
Association with Parvovirus B19 infection
C.
Pancytopenia
D.
Budd-Chiari syndrome
E.
Erectile dysfunction, abdominal pain & esophageal spasm
Q43. Which of the following criteria is a must for the diagnosis of Multiple Myeloma?
(A)
Plasma cell infiltration of the bone marrow
(B). Very high ESR
(C). Light chains in the urine
(D). Osteolytic bone lesions
(E). Renal failure
Q44. Which of the following features is not part of the “POEMS” Syndrome?
(A). Polyneuropathy
(B). Organomegaly
(C). Endocrinopathy
(D)
Myocardial Infarction
(E). Skin changes
8
Professor Anwar Sheikha, MD, FRCP, FRCPath., FACP
One Thousand Hematology MCQ
Q45. All of the following features suggest the diagnosis of Multiple Myeloma rather
than Benign Monoclonal Gammopathy (Monoclonal Gammopathy of
Undetermined significance/ M-GUS) EXCEPT:
(A). Presence of Plasmablasts
(B). Presence of clusters of plasma cells in the marrow biopsy
(C). Osteolytic bone lesions
(D). Hypercalcemia
(E)
Less than 10% plasma cells in the bone marrow
Q46. The major groups of B-cell Non-Hodgkin lymphomas in the new W.H.O.
classification include any of the followings EXCEPT:
(A). Follicular lymphoma
(B). Mantle cell lymphoma
(C). Burkitt lymphoma
(D). Lymphoblastic lymphoma
(E)
Mycosis fungoides/ Sezary syndrome
Q47. The major groups of T-cell non-Hodgkin lymphomas in the new W.H.O.
classification include any of the followings EXCEPT:
(A). Lymphoblastic lymphoma
(B). Peripheral T-cell lymphoma
(C)
Mantle cell lymphoma
(D). Adult T-cell leukemia/lymphoma
(E). Mycosis fungoides/ Sezary syndrome
Q48. Stage I or Low Cell Mass stage of Myeloma has all of the features EXCEPT:
(A)
Renal failure
(B). Hb > 100 gm/L
(C). Calcium equal or < 3 mmol/L
(D). Low rate of paraprotein production
(E). Normal bones or a solitary lesion in the X-ray
Q49. Eosinophilia is a well-known feature of:
(A). Viral infections
(B)
Asthma
(C). Myelofibrosis
(D). Visceral Leishmaniasis
(E). Aplastic anemia
Q50. One of the following features is characteristic for Myelofibrosis:
(A). Very high hemoglobin, because it is a myeloproliferative disorder
(B). Splenic atrophy in one third of patients
(C)
Extramedulary hemopoiesis
(D). Hypochromic microcytic red cells from iron exhaustion
(E). Reduced reticulin fibers in the marrow
Q 51. The least frequent cause of death in hemophiliacs is:
A.
Intracranial hemorrhage
B.
Hepatitis
C.
HIV infection
D.
Bleeding associated with FVIII inhibitor
E
Postpartum hemorrhage
9
Professor Anwar Sheikha, MD, FRCP, FRCPath., FACP
One Thousand Hematology MCQ
Q 52. Anemia of chronic disorders is least likely in one of the following chronic
conditions:
A
Diabetes mellitus
B.
Tuberculsois
C.
Bronchogenic carcinoma
D.
Rheumatoid arthritis
E.
Osteomyelitis
Q 53. Anemia of chronic disorders is least likely in one of the following chronic
conditions:
A.
Bronchiectasis
B.
AIDS
C.
Infective endocarditis
D
Hypertension
E.
Temporal arteritis
Q 54. All but one of the followings is true about hemolytic uremic syndrome “HUS”:
A
It is always due to deficiency of ADAMTS-13
B.
Usually affects children and follows an attack of bloody diarrhea
C.
Shiga toxin from toxigenic E. coli “0157:H7” or Shigella is the usual culprit
D.
In children plasma exchange is usually unnecessary and ineffective
E.
HUS affecting adults are more serious and need plasma exchange
Q 55. One of the followings is not true about thrombotic thrombocytopenic purpura:
A.
Usually due to deficiency of the metalloproteinase enzyme “ADAMTS-13”
B.
There is accumulation of unusually large multimers of vWF “ULvWF” that
causes formation of platelet thrombi
C.
Antibody to ADAMTS-13 could also lead to pile up of ULvWF
D.
Plasma exchange is the treatment of choice with high recovery rate
E
Drugs like Clopidogrel or Ticlopidine usually cause drug-induced
thrombocytopenia rather than TTP
Q 56. Worldwide, the most frequent cause of death in hemophiliacs has been:
A.
Intracranial hemorrhage
B.
Hepatitis
C
HIV infection
D.
Bleeding associated with FVIII inhibitor
E.
Postpartum hemorrhage
Q 57. Prolonged APTT without bleeding history is a feature of:
A.
Factor XII deficiency
B.
Prekallikrein deficiency
C.
HMWK deficiency
D
All of the above
E.
None of the above
Q 58. Bleeding in one of the following clotting factor deficiencies is usually mild:
A.
von Willebrand disease type 2N
B.
Hemophilia A
C.
Christmas disease
D
Factor XI deficiency
E.
Type 3 von Willebrand disease
10
Professor Anwar Sheikha, MD, FRCP, FRCPath., FACP
One Thousand Hematology MCQ
Q 59. Normal clotting screen “PT; APTT & TT” in a patient with convincing history of
bleeding could be due to:
A.
Factor VIII deficiency
B.
Factor IX deficiency
C.
Factor XII deficiency
D
Factor XIII deficiency
E.
Prothrombin deficiency
Q 60. TRALI “Transfusion Related Acute Lung Injury” is least likely to be associated
with transfusion of one of the following blood products:
A.
Whole blood
B
Packed red cells
C.
Platelet concentrate
D.
Cryoprecipitate
E.
FFP “Fresh Frozen Plasma”
Q 61. Normal naïve B-cells differentiate from precursor B- lymphoblasts after
Immunoglobulin VDJ gene rearrangement. Leukemia of these cells is:
A.
Aggressive
B.
SmIg+++
C
CD5 positive
D.
CD23E.
Cyclin D1+
Q 62. Normal naïve B-cells differentiate from precursor B- lymphoblasts after
Immunoglobulin VDJ gene rearrangement. Malignancy of these cells have all but
one of the following features:
A.
Histologically low grade
B.
Clinically indolent
C.
Morphologically mature looking
D
Usually associated with B- symptoms
E.
Often widespread
Q 63. All but one of the following treatment options could be offered to patients
resistant to the standard Imatinib mesylate dose of 400 mg/day:
A.
Dasatinib
B.
Nilotinib
C.
Higher dose of Imatinib
D
Autologous peripheral stem cell transplantation
E.
Allogeneic peripheral stem cell transplantation
Q 64. One of the following infectious agents has been shown to contribute to the
development of Gastric MALToma “Lymphoma of the Mucosa Associated
Lymphoid Tissue”:
A.
CMV “Cytomegalovirus”
B.
EBV “Epstein-Barr virus”
C.
HTLV-1 “Human T- Leukemia Virus type-1”
D
Helicobacter pylori
E.
Mixed bacterial infections
11
Professor Anwar Sheikha, MD, FRCP, FRCPath., FACP
One Thousand Hematology MCQ
Q 65. One of the following infectious agents has been shown to contribute to the
development of Burkitt lymphoma:
A.
CMV “Cytomegalovirus”
B
EBV “Epstein-Barr virus”
C.
HTLV-1 “Human T- Leukemia Virus type-1”
D.
Helicobacter pylori
E.
Mixed bacterial infections
Q66. MACROCYTOSIS could be seen in all of the following EXCEPT:
A.
5 q- syndrome
B.
Aplastic anemia
C
Hereditary spherocytosis
D.
Erythroleukemia
E.
Myelofibrosis
Q67. COOMB'S TEST is positive in:
A.
Hereditary spherocytosis
B
Warm autoimmune hemolytic anemia
C.
Iron deficiency anemia
D.
Glucose-6-Phosphate Dehydrogenase deficiency
E.
Thalassemia minor
Q68. Target cells could be seen in all of the following diseases EXCEPT:
A.
Thalassemia major
B.
Thalassemia minor
C.
Liver disease
D.
Splenic atrophy
E
Acute lymphoblastic leukemia
Q69. All of the following changes could be seen in iron deficiency anemia EXCEPT:
A
Subacute combined degeneration of the spinal cord
B.
Angular stomatitis
C.
Koilonychia
D.
Glossitis
E.
Esophageal web (Plummer-Vinson Syndrome)
Q70. All of the following features are characteristic for pernicious anemia EXCEPT:
A.
Oval macrocytosis
B.
Hypersegmented neutrophils
C.
Howell-Jolly bodies
D
Deficient hemoglobinization of the developing red cells
E.
Active ineffective megaloblastic marrow
Q71. Major criteria to diagnose aplastic anemia include all of the following EXCEPT:
A.
Empty bone marrow
B.
Anemia
C
Jaundice
D.
Neutropenia
E.
Thrombocytopenia
12
Professor Anwar Sheikha, MD, FRCP, FRCPath., FACP
One Thousand Hematology MCQ
Q72. All of the following features relate to hemolytic anemias EXCEPT:
A.
Splenomegaly
B.
Gall Stones
C.
Indirect hyperbilirubinemia
D
Hypochromic microcytic red cells
E.
Reticulocytosis
Q73. One of the following is characteristic for β- thalassemia minor:
A
High Hemoglobin A2
B.
Very high Hemoglobin F
C.
Marked splenomegaly
D.
Severe transfusion-dependent anemia
E.
High Hemoglobin H
Q74. All of the following features are true for sickle cell disease EXCEPT:
A.
Vaso-occlusive crises
B.
Hemolytic anemia
C.
Leg ulcers
D.
Aseptic bone necrosis
E
Splenomegaly in the majority of cases
Q75. Warm autoimmune hemolysis could result from all of the following EXCEPT:
A
Burn
B.
Chronic lymphocytic leukemia
C.
Systemic lupus erythematosus
D.
Drugs
E.
Lymphoma
Q76. Helmet cells could be seen in all of the following diseases EXCEPT:
A.
MAHA (Microangiopathic Hemolytic Anemia)
B.
March hemoglobinuria
C.
TTP (thrombotic thrombocytopenic purpura)
D.
Artificial heart valves
E
Aplastic anemia
Q77. All of the following features relate to G-6-P DH deficiency EXCEPT:
A.
Intravascular hemolysis
B.
Favism
C.
Blister red cells
D.
Heinz bodies
E
Moderate splenomegaly
Q78. One of the following features is not true in Acute Lymphoblastic Leukemia:
A.
It is the commonest cancer of children
B
Blast cells are positive for Sudan Black stain
C.
Results from proliferation of lymphoblasts
D.
Could present with anemia, neutropenia & thrombocytopenia
E.
CNS could be involved
13
Professor Anwar Sheikha, MD, FRCP, FRCPath., FACP
One Thousand Hematology MCQ
Q79. One of the following features is not true in the FAB classification of
Acute Myeloblastic Leukemia :
A.
M1 is AML of the myeloblasts
B
M2 is AML of the lymphoblasts
C.
M3 is AML of the promyelocytes
D.
M5 is AML of the monoblasts
E.
M7 is AML of the megakaryoblasts
Q80. Marked splenomegaly could be seen in any of the following EXCEPT:
A.
Thalassemia major
B.
Chronic myeloid leukemia
C
Immune thrombocytopenic purpura “ITP”
D.
Kala Azar
E.
Malarial splenomegaly or Tropical Splenomegaly Syndrome
Q81. Granulomatous Lymphadenitis could result from any of the following “specific”
conditions EXCEPT:
A.
Tuberculosis
B.
Toxoplasmosis
C.
Sarcoidosis
D
Cushing’s disease
E.
Crohn’s disease
Q82. One of the following features is not true in Nodular Sclerosis Hodgkin’s disease:
A.
Lacunar type of Reed Sternberg cells
B.
Anterior mediastinal mass
C.
No male preponderance
D.
Alcohol intolerance
E
The commonest type in the Middle East
Q83. One of the following conditions cannot cause marked splenomegaly:
A.
Osteopetrosis
B.
Niemann Pick disease
C.
Gaucher’s disease
D.
Hydatid disease of the spleen
E
Celiac disease
Q84. One of the following features is not true in Burkitt’s Lymphoma:
A.
It is associated with Epstein Barr virus infection and malaria
B
It is common at very high altitudes
C.
It usually affects the jaw bones or the viscera
D.
It is one of the most aggressive tumors but responsive to chemotherapy
E.
It has a “Starry-sky” tissue appearance
Q85. Renal failure complicates myeloma because of any one of the following EXCEPT:
A.
Amyloidosis of the Kidney
B.
Nephrocalcinosis from hypercalcemia
C.
Light chain damage to the renal tubules
D
Extensive infiltration of the kidney by myeloma cells
E.
Pyelonephritis secondary to immune paresis & neutropenia
14
Professor Anwar Sheikha, MD, FRCP, FRCPath., FACP
One Thousand Hematology MCQ
Q86. One of the following features is not true in Waldenstrom’s macroglobulinemia:
A.
IgM Gammopathy
B.
Hyperviscosity
C.
Splenomegaly and lymphadenopathy
D.
Cryoglobulinemia
E
Large Osteolytic bone lesions
Q87. One of the following features cannot happen in Myelofibrosis:
A
Splenic atrophy
B.
Nucleated red cells and myelocytes in the blood smear
C.
Hemopoiesis in the liver and spleen
D.
Tear drop red blood cells
E.
Portal hypertension
Q88. One of the following features cannot happen in Essential Thrombocythemia:
A.
Splenic atrophy
B.
GIT bleeding
C.
Thrombosis
D.
Splenomegaly
E
Thrombocytopenia
Q89. One of the following features cannot happen in Polycythemia Rubra Vera:
A.
Hyperviscosity
B.
Hypermetabolism
C.
Hyperuricemia
D
Splenic atrophy
E.
Pruritis
Q90. All of the following Non-Hodgkin’s Lymphomas are intermediate grades in the
“old” Working Formulation EXCEPT:
A
Burkitt’s lymphoma
B.
Follicular Large Cell lymphoma
C.
Diffuse Large Cell Lymphoma
D.
Diffuse Small Cleaved Cell Lymphoma
E.
Diffuse Mixed Cell Lymphoma
Q91. APTT cannot be prolonged in one of the following conditions:
A.
von Willebrand’s Disease
B.
Hemophilia A
C.
Christmas Disease
D.
Hageman Factor (XII) Deficiency
E
Factor VII Deficiency
Q92. All of the following features are true in Acute Childhood ITP EXCEPT:
A.
Marrow megakaryocytosis
B.
Giant platelets with high Mean Platelet Volume (MPV)
C
Low incidence of spontaneous remission
D.
Prolonged bleeding time
E.
Purpuric superficial muco-cutaneous type of bleeding
15
Professor Anwar Sheikha, MD, FRCP, FRCPath., FACP
Q93.
One Thousand Hematology MCQ
Acute childhood ITP is characterized by all of the following features EXCEPT :
A.
It is usually a self limiting disease
B.
MPV “Mean Platelet Volume” could be more than 10 fL (Normal 5-8 fL)
C
No or few megakaryocytes in the bone marrow
D.
Platelet count is usually very low
E.
Good response to intravenous immunoglobulin
Q94. Thrombocytosis can result from any of the following causes EXCEPT :
A.
Hodgkin's Disease
B.
Ulcerative colitis
C.
Splenectomy
D
Aplastic anemia
E.
Steroid administration
Q95. All of the following relate to DIC EXCEPT:
A.
Hypofibrinogenemia
B.
Prolonged PT, APTT and bleeding time
C.
Thrombocytopenia
D.
Increased D-Dimer & Fibrin(ogen) Degradation Products (FDP)
E
DIC means “Death Is Coming”
Q96. One of the following is not true in Acute Hypergranular Promyelocytic Leukemia :
A.
It results from proliferation of promyelocytes
B.
The leukemic cells could be dumb-bell shaped without granules
C.
The chromosome abnormality t(15q+;17q-) is characteristic
D.
Treatment with ATRA could be very effective
E
Characterized by lymphadenopathy, gum hypertrophy & skin infiltration
Q97. All of the following features relate to Chronic Granulocytic Leukemia EXCEPT:
A.
Philadelphia chromosome is positive in the majority of cases
B.
The chronic stage usually lasts about four years
C.
Segmented neutrophils and myelocytes are increased
D
Neutrophil alkaline phosphatase is high
E.
Usually terminates in blast crisis
Q98. One of the following features is not true in Acute Lymphoblastic Leukemia:
A
It is basically the disease of elderly people
B.
It could happen in young adults
C.
It is the best type of children’s leukemia with a cure rate of >60%
D.
It is due to proliferation of lymphoblasts
E.
It could present as CNS leukemia
Q99. One of the following features is true in Chronic Lymphocytic Leukemia:
A
Usually affects elderly people
B.
It is not uncommon in children
C.
It results from uncontrolled proliferation of immature lymphocytes
D.
All patients should be treated immediately
E.
Thrombocytosis carries a poor prognosis
16
Professor Anwar Sheikha, MD, FRCP, FRCPath., FACP
One Thousand Hematology MCQ
Q100. Thrombocytopenia in Chronic Lymphocytic Leukemia could not result from:
A.
Bone marrow infiltration by leukemic cells
B.
Autoimmune antibodies to platelets
C.
Hypersplenism
D
Varicella zoster
E.
Chemotherapy used in the management of the CLL
Q101. Type 2N “N = Normandy” von Willebrand disease is due to:
A.
Quantitative deficiency of von Willebrand Factor “VWF”
B.
Deficiency of the high molecular weight VWF multimers
C.
Increased binding of VWF to platelets causing depletion of high molecular
weight VWF and thrombocytopenia
D.
Decreased binding of VWF to platelets, but with normal VWF multimers
E
Decreased binding of VWF to FVIII causing low plasma Factor VIII:c
Q102. Type 1 von Willebrand disease is due to:
A
Quantitative deficiency of von Willebrand Factor “VWF”
B.
Deficiency of the high molecular weight VWF multimers
C.
Increased binding of VWF to platelets causing depletion of high molecular
weight VWF and thrombocytopenia
D.
Decreased binding of VWF to platelets, but with normal VWF multimers
E.
Decreased binding of VWF to Factor VIII causing low plasma FVIII:C
Q103. Type 2M “M = Multimers” von Willebrand disease is due to:
A.
Quantitative deficiency of von Willebrand Factor “VWF”
B.
Deficiency of the high molecular weight VWF multimers
C.
Increased binding of VWF to platelets causing depletion of high molecular
weight VWF and thrombocytopenia
D.
Decreased binding of VWF to platelets, but with normal VWF multimers
E.
Decreased binding of VWF to Factor VIII causing low plasma FVIII:C
Q104. Type 2A von Willebrand disease is due to:
A.
Quantitative deficiency of von Willebrand Factor “VWF”
B
Deficiency of the high molecular weight VWF multimers
C.
Increased binding of VWF to platelets causing depletion of high molecular
weight VWF and thrombocytopenia
D.
Decreased binding of VWF to platelets, but with normal VWF multimers
E.
Decreased binding of VWF to Factor VIII causing low plasma FVIII:C
Q105. Type 2B von Willebrand disease is due to:
A.
Quantitative deficiency of von Willebrand Factor “VWF”
B.
Deficiency of the high molecular weight VWF multimers
C
Increased binding of VWF to platelets causing depletion of high molecular
weight VWF and thrombocytopenia
D.
Decreased binding of VWF to platelets, but with normal VWF multimer
distribution
E.
Decreased binding of VWF to Factor VIII causing low plasma FVIII:C
17
Professor Anwar Sheikha, MD, FRCP, FRCPath., FACP
One Thousand Hematology MCQ
Q106. One of the following types of von Willebrand disease is autosomal recessive:
A.
Type 1 von Willebrand disease
B.
Type 2A von Willebrand disease
C.
Type 2B von Willebrand disease
D
Type 2N von Willebrand disease
A.
Type 2M von Willebrand disease
Q107. One of the following types of von Willebrand disease is associated with
thrombocytopenia:
A.
Type 1 von Willebrand disease
B.
Type 2A von Willebrand disease
C
Type 2B von Willebrand disease
D.
Type 2N von Willebrand disease
A.
Type 2M von Willebrand disease
Q108. One of the following features does not relate to Type 3 von Willebrand disease:
A
It is autosomal dominant
B.
There is virtual absence of von Willebrand Factor
C.
There is profound deficiency of Factor VIII:c with prolonged APTT
D.
Screening assays show absent VWF:RCo and VWF:Ag
E.
Parents of these patients may have Type 1 vWD with bleeding
Q109. One of the following feature is not true about Type 2N “N = Normandy”
von Willebrand disease:
A
It is autosomal dominant
B.
Mutations selectively inactivate FVIII:C binding site on VWF
C.
The platelet-dependent functions of VWF is intact
D.
Factor VIII:C is usually <10% and APTT is prolonged
E.
Patients usually behave like hemophilia but with autosomal style of
inheritance
Q110. Type 2N “N = Normandy” von Willebrand disease should be suspected in:
A.
Any patient with low FVIII:C in whom a Factor VIII inhibitor is ruled out
B.
Any patient with low FVIII:C in whom X-linked inheritance is not clear
C.
Any patient with low FVIII:C in whom therapy with recombinant or
monoclonal FVIII concentrate gives poor results
D
All of the above
E.
None of the above
Q111. The drug most commonly associated with drug-induced thrombocytopenia is:
A.
Vincrisitne
B.
Steroid
C
Septrin
D.
Aspirin
E.
Warfarin
Q112. Bleeding due to all but one of the followings is extremely rare:
A.
Factor II “Prothrombin” deficiency”
B.
Factor VII deficiency
C
Factor IX deficiency
D.
Factor X deficiency
E.
Factor V deficiency
18
Professor Anwar Sheikha, MD, FRCP, FRCPath., FACP
One Thousand Hematology MCQ
Q113. Median survival for patients with chronic myeloid leukemia who were treated with
Busulphan “Myleran” alone:
A.
Is around one year
B
Is around 45 months
C.
Is around 58 months
D.
Could possibly be decades
E.
Could have normal life expectancy
Q114. Survival of patients with chronic myeloid leukemia who were treated with
Hydroxyurea alone:
A.
Is around one year
B.
Is around 45 months
C
Is around 58 months
D.
Could possibly be decades
E.
Could have normal life expectancy
Q115. Survival of a patient with chronic myeloid leukemia who is treated with Imatinib
“Glivec” alone:
A.
Is around one year
B.
Is around 45 months
C.
Is around 58 months
D
Could possibly be decades
E.
Could have normal life expectancy
Q116. Disease free survival of patients with chronic myeloid leukemia who were
managed successfully with allogeneic peripheral stem cell transplantation:
A.
Is around one year
B.
Is around 45 months
C.
Is around 58 months
D.
Could possibly be decades
E
Could have normal life expectancy
Q117. All but one of the followings features do not apply to Factor XI deficiency:
A.
Usually associated with prolonged APTT
B.
Usually treated with FFP
C
Bleeding is universal and like in hemophilia is predictably proportional
to the degree of FXI deficiency
D.
Usually seen in Ashkenazi “European” Jews
E.
Antifibrinolytic agents like Tranexamic acid or Epsilon–aminocaproic acid
may provide adequate protection for minor procedures
Q118. In a pregnant lady, gestational rather than immune thrombocytopenia is a
possibility when:
A.
The thrombocytopenia was present even prior to the pregnancy
B.
The thrombocytopenia started early in pregnancy
C.
The platelet count is usually less than 40K/uL
D
The thrombocytopenia was present in prior pregnancies
E.
Platelet antibodies could be detected
19
Professor Anwar Sheikha, MD, FRCP, FRCPath., FACP
One Thousand Hematology MCQ
Q119. Acute onset thrombocytopenia in an ICU setting is least likely to be due to:
A.
Thrombotic thrombocytopenia purpura
B.
Sepsis
C.
Massive blood transfusion
D.
DIC
E
Iron deficiency
Q120. Macrocytosis could not happen in one of the following conditions:
A.
Aplastic anemia
B.
Hypothyroidism
C.
Reticulocytosis
D.
Myelodysplastic syndrome
E
Thalassemia major
Q121. All of the following physiological compensatory mechanisms could try to correct
the effect of severe chronic anemia EXCEPT:
A
Increasing the blood pressure
B.
Increasing red cell 2,3 DPG
C.
Increasing cardiac output
D.
Redistribution of blood from skin to brain
E.
Increasing plasma volume
Q122. Spherocytosis is a prominent feature of one of the following conditions:
A.
Sickle cell anemia
B
Warm autoimmune hemolytic anemia
C.
Cold hemagglutinin disease
D.
Fragmentation anemia
E.
Hereditary elliptocytosis
Q123. All of the following features could be seen in megaloblastic anemia EXCEPT:
A.
Hypersegmented neutrophils
B.
Oval macrocytosis
C.
Glossitis with a beefy tongue
D.
Mild thrombocytopenia or neutropenia
E
Target cells
Q124. All of the following cells are regarded as primitive hematological cells EXCEPT:
A
Megakaryocyte
B.
Myeloblast
C.
Pronormoblast
D.
Monoblast
E.
Lymphoblast
Q125. Red cell fragmentation is a feature of all of the following conditions EXCEPT:
A.
March hemoglobinuria
B.
Malignant hypertension
C.
Prosthetic heart valve
D.
Hemolytic Uremic syndrome
E
Warm autoimmune hemolytic anemia
20
Professor Anwar Sheikha, MD, FRCP, FRCPath., FACP
One Thousand Hematology MCQ
Q126. All of the following could be due to Epstein Barr Virus infection EXCEPT::
A.
Infectious mononucleosis
B.
Angiocentric lymphoma
C.
Nasopharyngeal carcinoma
D
Hodgkin’s disease
E.
Burkitt’s lymphoma
Q127. All of the following facts relate to G6PDH deficiency EXCEPT:
A
The anemia is usually extravascular with remarkable splenomegaly
B.
Hemolysis is severe in Mediterranean type (no activity in reticulocytes)
C.
In the A- type, reticulocytes has 10% activity & hemolysis is not severe
D.
The disease is sex-linked
E.
Most female with low enzyme level have extreme lyonization
Q128. One of the following features is not true in Hereditary Elliptocytosis:
A.
It is quite common in Saudi Arabia
B
The hemolytic anemia is even more severe than hereditary spherocytosis
C.
It is autosomal dominant
D.
Homozygous patients can have a very severe hemolytic disease
E.
Could present with jaundice, gall stones, splenomegaly & reticulocytosis
Q129. Thalassemias are common in the indigenous people of these countries EXCEPT:
A
England
B.
Italy
C.
Greece
D.
Saudi Arabia
E.
Kurdistan
Q130. Majority of the leukemic cells are strongly positive for CD10 (CALLA) in which
type of leukemia:
A.
T-CLL
B.
Acute myeloid leukemia
C.
Myelodysplastic syndrome
D
Common ALL
E.
Burkitt type of ALL (B-ALL)
Q131. Sudan Black, Myeloperoxidase & Specific Esterase are positive in:
A
AML without differentiation (M1)
B.
Acute Monoblastic Leukemia (M5)
C.
Acute megakaryoblastic Leukemia (M7)
D.
Acute lymphoblastic Leukemia
E.
Hairy Cell Leukemia
Q132. One of the following features is not true in Acute Promyelocytic Leukemia (M3):
A.
It used to have a very poor prognosis because of DIC following treatment
B.
It is now one of the best types of AML because of treatment with ATRA
C.
Faggot Cells ( Promyelocytes with bundles of Auer Rods) could be seen
D.
Strongly positive Sudan Black stain
E
Very strong TdT positivity
21
Professor Anwar Sheikha, MD, FRCP, FRCPath., FACP
One Thousand Hematology MCQ
Q133. Acute leukemia patients cannot present with:
A.
Fungal infection
B.
Bacterial Infection
C.
Viral Infection
D
Empty bone marrow
E.
Mediastinal widening
Q134. In Acute Leukemia the white blood cell count could be:
A.
High
(100,000/uL)
B.
Very high
(250 000/uL)
C.
Low
(1000/uL)
D.
Normal
(10,000/uL)
E
All of the above
Q135. All of the following are true for chronic myeloid leukemia treatment EXCEPT:
A.
Imatinib mesylate (Glivec) is a new revolutionary oral treatment
B.
Interferon can benefit many patients but side effects are troublesome
C.
Hydroxyurea is still the most widely used treatment
D.
Busulphan is no more used because of its long-term complications
E
Old patients should be given the choice of stem cell or marrow transplant
Q136. Which one is not true for chronic lymphocytic leukemia treatment:
A.
Rai stage IV patients benefit from steroid before proper chemotherapy
B
All Rai stage I & II patients should be immediately treated
C.
Fludarabine, Chlorambucil, Vincristine & Prednisolone are effective
D.
Coomb’s positive hemolytic anemia patients better be given steroid first
E.
Acyclovir should be used liberally for zoster and herpes simplex
Q137. The following are regarded as Chronic lymphoproliferative disorders EXCEPT:
A.
Hairy cell leukemia
B.
Small lymphocytic lymphoma
C.
Waldenstrom’s macroglobulinemia
D
Burkitt lymphoma
E.
Chronic lymphocytic leukemia
Q138. All of the following features relate to Hairy Cell Leukemia EXCEPT:
A.
Splenomegaly
B
Granulocytosis
C.
Dry bone marrow aspiration tap
D.
Tartarate Resistant Acid Phosphatase (TRAP) positive cells
E.
Monocytopenia
Q139. All of the following features are true in Aplastic Anemia management EXCEPT :
A.
Marrow or stem cell transplantation should be offered to all young patients
B.
Anti-lymphocyte Globulin could benefit many patients
C.
Chemotherapy could be effective in patients with marrow lymphocytosis
D.
Blood transfusion from potential marrow donors should be avoided
E
Supportive management with red cells, platelets & antibiotics has no role
22
Professor Anwar Sheikha, MD, FRCP, FRCPath., FACP
One Thousand Hematology MCQ
Q140. All of the following are true for Vitamin B12 EXCEPT:
A
Mainly absorbed in the duodenum
B.
The daily requirement is 1 microgram
C.
The body stores are enough for many years
D.
Deficiency causes megaloblastic anemia
E.
Mostly available in meat products
Q141. All but one of the following features are true about Cord Blood Stem Cell
transplantation:
A.
Histocompatibility mismatch is better tolerated
B
High numbers of stem cells is guaranteed
C.
The median time to availability is short since all supply are typed
D.
Associated with a lower incidence of GvHD
E.
Associated with a higher incidence of graft failure and slow engraftment
Q142. One of the following features is not true in Cord Blood Stem Cell transplantation
A.
A poor source of future Donor Lymphocyte Infusion supply
B.
The cord stem cells could be from a baby with a genetic disorder
C.
Could be the only source in communities with diverse background
D
Engraftment is quick and steady
E.
Low incidence of GvHD
Q143. All but one of the following statements is true about autologous transplantation:
A.
Currently it is the commonest practiced form of stem cell transplantation
B
It could be associated with a modest degree of GvHD
C.
It does not have Graft versus Tumor effect
D.
The biggest drawback is the potential possibility of tumor contamination
E.
It seems to be the treatment of choice for multiple myeloma
Q144. Finding a histocompatible matched unrelated donor “MUD” for a leukemia
patient who needs stem cell transplantation is more difficult in:
A.
Irish Americans
B
African Americans
C.
Japanese
D.
Nordic countries
E.
Korea
Q145. The most probable form of stem cell transplantation for a Kurdish patient with
Philadelphia negative chronic myeloid leukemia who had immigrated to Sweden
and offered the procedure is:
A.
Matched Unrelated Donor transplantation
B.
Autotransplantation
C.
Syngeneic transplantation
D.
Allogeneic transplantation
E
Cord blood transplantation
23
Professor Anwar Sheikha, MD, FRCP, FRCPath., FACP
One Thousand Hematology MCQ
Q146. The most probable form of stem cell transplantation for a Swedish patient with
Philadelphia negative chronic myeloid leukemia, who is the only child in the
family is:
A
Matched Unrelated Donor transplantation
B.
Autotransplantation
C.
Syngeneic transplantation
D.
Allogeneic transplantation
E
Cord blood transplantation
Q147. The most frequent form of stem cell transplantation for patients with multiple
myeloma is:
A.
Matched Unrelated Donor transplantation
B
Autotransplantation
C.
Syngeneic transplantation
D.
Allogeneic transplantation
E.
Cord blood transplantation
Q148. The best form of stem cell transplantation for a patient with aplastic anemia is:
A.
Matched Unrelated Donor transplantation
B.
Autotransplantation
C
Syngeneic transplantation
D.
Allogeneic transplantation
E.
Cord blood transplantation
Q149. The transplant of choice for a patient with thalassemia major is:
A.
Matched Unrelated Donor transplantation
B.
Autotransplantation
C.
Syngeneic transplantation
D
Allogeneic transplantation
E.
Cord blood transplantation
Q150. There is no role for hematopoietic stem cell transplantation in the initial treatment
strategy of one of the following blood diseases:
A
Acute promyelocytic leukemia
B.
Acute myeloid leukemias
C.
Chronic myeloid leukemia
D.
Aplastic anemia
E.
Thalassemia major
Q151. Pronormoblasts are cells with all of the following features EXCEPT:
A.
They are the earliest recognizable erythroid cells
B.
They have deep blue basophilic cytoplasm
C
They do not contain nucleoli
D.
They eventually give rise to normoblasts
E.
Their nuclei are round and consist of finely stippled chromatin
Q152. Promyelocytes are cells with all of the following features EXCEPT:
A
They are the earliest granulocyte precursors
B.
Their nuclei contain few nucleoli
C.
Their cytoplasm contain granules
D.
They will give rise to myelocytes
E.
They may be larger than myeloblasts
24
Professor Anwar Sheikha, MD, FRCP, FRCPath., FACP
One Thousand Hematology MCQ
Q153. Plasma cells have all of the following features EXCEPT:
A.
They are the end stage cells in the lymphoid lineage
B.
They produce immunoglobulins
C.
They are elliptical fried-egg like cells with eccentric nuclei
D.
Their nuclear chromatin is coarse and clumped
E
They can normally be seen in the peripheral blood smear
Q154. Megakaryocytes are characterized by all of the following EXCEPT:
A
They are the largest cells in the peripheral blood smear
B.
They produce platelets
C.
They are usually multinucleated
D.
Their cytoplasm is full of azurophilic granules
E.
They can reach the size of up to 100 micrometer in diameter
Q155. The following features relate to the red cell anomalies :
A.
Anisocytosis indicates difference in shape
B.
Poikilocytosis indicates difference in size
C.
Howell-Jolly bodies are precipitated denatured hemoglobin
D.
Heinz bodies are nuclear remnants
E
Polychromasia usually indicates reticulocytosis
Q156. All of the following hematological values are normal EXCEPT:
A.
A hemoglobin of 10 gm/dL in a three month old child
B.
A hematocrit of 60% in a newborn
C.
An MCV of 100 femtoliter in a newborn
D
Red cell volume of 38 ml/Kg body weight in an adult male
E.
A neutrophil count of 2000/uL in a Saudi medical student
Q157. The followings relate to Glucose-6-Phosphate Dehydrogenase Deficiency :
A.
Usually affects female patients
B.
G6PDH is important in the Embden-Meyerhof (Glycolysis) cycle to provide
reducing power for the protection of the red cells from oxidative injuries
C.
Favism usually happens three days after eating beans
D
Inheritance of the deficient gene is sex-linked
E.
Acute Drug-induced hemolysis happens immediately after taking the drug
Q158. One of the following features is not true in pernicious anemia:
A
It is due Vitamin B12 deficiency from poor intake
B.
It is due to autoantibodies against the gastric parietal cells
C.
Marrow erythropoiesis is active but ineffective
D.
Blood smear shows oval macrocytes and hypersegmented neutrophils
E.
There is severe lack of intrinsic factor
Q159. One of the following features is correct in hereditary spherocytosis :
A.
It is autosomal recessive
B.
Usually associated with marked unconjugated hyperbilirubinemia
C.
Usually associated with marked conjugated hyperbilirubinemia
D.
Splenectomy reverses the red cell shape to biconcave form
E
Severe unexpected anemia could result from Parvovirus B19 infection
25
Professor Anwar Sheikha, MD, FRCP, FRCPath., FACP
One Thousand Hematology MCQ
Q160. One of the following features is not true in Warm Autoimmune Hemolytic Anemia:
A.
It is usually due to IgG antibodies
B
It is usually due to IgM antibodies
C.
Steroid therapy &/or splenectomy usually benefit the patient
D.
The spleen is often enlarged
E.
Coomb's test is usually positive
Q161. The following statements relate to Sickle Cell Trait :
A.
Patients usually present with painful crises
B.
Hemoglobin S is usually more than 50%
C
They are usually protected from Plasmodium falciparum infection
D.
Patients are usually symptomatic from their anemia
E.
Priapism is common
Q162. One of the following features relate to β- thalassemia major :
A
Splenomegaly
B.
Splenic atrophy from repeated vaso-occlusion
C.
High hemoglobin F at around 40%
D.
Reduced iron absorption
E.
Effective erythropoiesis
Q163. All of the following features relate to hemophilia A EXCEPT:
A.
Prolonged APTT
B
Prolonged bleeding time
C.
It is usually a male disease
D.
It is transmitted by female
E.
Joint and deep-seated bleeding is common
Q164. von Willebrand disease is characterized by :
A.
Prolonged prothrombin time
B.
Deficiency of the coagulant portion of factor VIII (VIII:c)
C
Prolonged bleeding time
D.
It only affects male
E.
Hemarthrosis is common
Q165. Thrombocytopenic Purpura could complicate any of the following EXCEPT:
A.
Acute Leukemia
B.
Aplastic anemia
C
Pure red cell aplasia
D.
Myelodysplastic syndromes
E.
Bone marrow metastasis
Q166. Thrombocytosis can result from any of the following conditions EXCEPT:
A.
Severe iron deficiency from a chronically bleeding duodenal ulcer
B.
5 q- Syndrome
C.
Splenectomy
D
Aplastic anemia
E.
Steroid administration
26
Professor Anwar Sheikha, MD, FRCP, FRCPath., FACP
One Thousand Hematology MCQ
Q167. All of the following are true in DIC EXCEPT:
A.
It could complicate snake bite
B.
It could complicate mismatched blood transfusion
C.
Prolonged PT, APTT and bleeding time
D.
It is quite common in obstetric practice
E
Should never be treated with platelet & FFP because fuel is added to the
fire
Q168. One of the following cannot happen in Chronic Granulocytic Leukaemia:
A.
Priapism
B.
Gout
C.
Weight loss from hypermetabolism
D.
Acute abdomen from splenic infarction
E
Thrombocytopenia unless in blast crisis
Q169. Only one of the following features is true in Acute Lymphoblastic Leukemia:
A.
It is basically the disease of elderly people
B.
White cell count is always high in the blood
C.
Bone marrow or stem cell transplantation is the early treatment of choice
D.
Only a minority of patients could be cured of their disease
E
It could present as CNS leukemia, which has a worse prognosis
Q170. All of
A
B.
C.
D.
E.
the following is true in Chronic Lymphocytic Leukemia EXCEPT:
All stages of the disease should be treated immediately
Fludarabine and Chlorambucil are known drugs in the treatment of CLL
Rai and Binet staging systems are utilized in grading the stage of CLL
Could be confused with small lymphocytic lymphoma
Bone marrow or stem cell transplantation do not benefit majority of
patients
EACH OF THE FOLLOWING MULTIPLE CHOICE QUESTIONS IS COMPOSED OF
A STEM AND FIVE STATEMENTS. EACH OF THE STATEMENTS COULD BE TRUE
OR FALSE. IN YOUR ANSWER SHEET, YOU ARE REQUESTED TO FILL THE
CORRECT ONES AND LEAVE THE FALSE ONES EMPTY.
Q171. The following statements relate to hemopoiesis :
A
Hemopoiesis starts from the bone marrow in a two week old embryo
B
Pronormoblast is a red cell precursor which is full of hemoglobin
C
Promyelocyte is a large cell with few nucleoli but no granules
D.
Anisocytosis is a term used to denote variation in red cell size
E
Poikilocytes are irregularly shaped red cells.
Q172. The following statements concern normal hematological values :
A
The normal MCV is between 76 and 96 milliliters
B.
A hemoglobin of 10 gm/dL is normal for a three month old child
C.
A 70 Kg adult male has around ten units of whole blood
D
The red cell life span is between two and three months
E.
The red cell count of a full term infant is less than an adult male
27
Professor Anwar Sheikha, MD, FRCP, FRCPath., FACP
One Thousand Hematology MCQ
Q173. The following statements relate to iron status in the human body :
A
The normal amount of iron in an adult male is about 15 gms
B.
The bone marrow needs about 30 mg of iron daily to produce red cells
C.
A menstruating female requires about three milligrams of iron daily
D
Excessive TEA consumption increases iron absorption
E
Unconjugated iron binding capacity is reduced in iron deficiency anemia
Q174. Macrocytosis could happen in any of the following conditions :
A.
Alcoholic liver cirrhosis
B
Thalassemia
C.
Newborn
D
Iron overload
E.
Folic acid deficiency
Q175. The following statements relate to Sickle Cell Anemia:
A.
It is common in the Asir region of Saudi Arabia
B
It only affects male patients
C.
Sickle cell trait protects against Falciparum malaria
D
It results from reduced production of β-globin chains
E
Patients with low hemoglobin should immediately be transfused
Q176. The following features relate to Sickle Cell Disease:
A.
Splenomegaly at early childhood
B
Hemoglobin S of 40%
C.
Leg ulcers
D.
Hand-foot syndrome
E
Symptomatic anemia
Q177. Sicklers can present with any of the following features:
A.
Biliary colic
B.
Priapism
C
Marked splenomegaly immediately after birth
D.
Salmonella osteomyelitis
E
Splenic atrophy in the neonatal period
Q178. The following features relate to Sickle Cell Trait:
A
Severe hypochromic microcytic anemia
B
Extramedullary hemopoiesis
C.
Splenic syndrome at high altitude
D.
Sickling problems during long flights in unpressurized aircrafts
E
Hemoglobin S of 73%
Q179 The following statements relate to thalassemias:
A.
Patients with low hemoglobin should be transfused
B
β-thalassemia is due to replacement of Glutamic acid by Valine
C
High Hemoglobin F in a neonate indicates β- thalassemia major
D.
Hemoglobin electrophoresis is essential for proper diagnosis
E
Hemoglobin F has a low affinity for oxygen
28
Professor Anwar Sheikha, MD, FRCP, FRCPath., FACP
One Thousand Hematology MCQ
Q180. The following statements relate to β- thalassemia minor :
A
Normal hemoglobin A2
B.
Hypochromic microcytic red cells
C.
Increased numbers of red cells
D
Severe anemia necessitating transfusions
E.
Should be counseled regarding marriage to another β- thalassemia minor
Q181. The following features are seen in β- thalassemia intermedia :
A.
Iron overload
B
Continuous and persistent need for blood transfusion
C.
Paravertebral extramedullary hemopoietic deposits
D.
Deafness from temporal bone involvement
E
Positive Coomb’s test
Q182. The following laboratory results relate to β- thalassemia major:
A.
High Hemoglobin F
B.
Mild unconjugated hyperbilirubinemia
C
Normal hemoglobin A
D.
Hypochromic microcytic red cells with many target & nucleated red cells
E
sickle shaped red cells
Q183. The following statements relate to the laboratory diagnosis of leukemias:
A
Sudan Black is positive in Acute Lymphoblastic Leukemia
B
Periodic Acid Schiff is positive in Acute Myeloid Leukemia
C
Specific (Chloroacetate) Esterase is positive in Acute Monoblastic type
D
Leukocyte Alkaline Phosphatase is high in chronic myeloid leukemia
E.
Acid phosphatase is polar positive in T cell acute lymphoblastic leukemia
Q184. The following statements relate to Acute Monoblastic leukaemia (M5) :
A
It has the best prognosis between the subtypes of AML
B.
Non-specific Butyrate Esterase is positive in the leukemic monoblasts
C
DIC is very common if patients are not heparinized before treatment
D.
Gum hypertrophy, skin involvement and lymphadenopathy are common
E.
Lysozyme enzyme is usually elevated
Q185. The following tests relate to hemostatic tests :
A
Prolonged prothrombin time is usually due to prothrombin deficiency
B.
Prolonged APTT could be due to Factor VIII:c deficiency
C.
Severe hypofibrinogenemia could result in prolonged PT, APTT & TT
D.
Factor X deficiency results in prolonged PT & APTT but not TT
E
Bleeding time is prolonged in Hemophilia A
Q186. The following statements relate to Hemophilia A :
A
It is almost always transmitted from the mother
B
It is due to deficiency of the large part of factor VIII (VIII R:Ag)
C
Activated Partial Thromboplastin Time (APTT) is usually normal
D
Factor VIII level of 20% is regarded as severe hemophilia
E
Menorrhagia and nose bleeding are very common
29
Professor Anwar Sheikha, MD, FRCP, FRCPath., FACP
One Thousand Hematology MCQ
Q187. The following statements are true about von Willebrand Disease :
A.
Prolonged bleeding time
B.
Failure of platelet aggregation to Ristocetin
C.
Menorrhagia
D.
APTT could be normal
E
Sex-linked inheritance
Q188. Thrombin Time (TT) is prolonged in the following situations :
A.
Patients on Heparin therapy
B
Prothrombin deficiency
C
von Willebrand Disease
D.
Severe hypofibrinogenemia
E
Hemorrhagic Disease of the Newborn
Q189. The following statements relate to Glucose-6-Phosphate Dehydrogenase :
A.
200 million people are deficient of this enzyme worldwide
B
Deficiency can cause severe extravascular hemolysis
C
Splenomegaly is a particular feature of Favism
D.
Deficiency is believed to protect against malaria
E.
Hemoglobinemia, hemoglobinuria & hemosiderinuria can occur
Q190. In Cold Hemagglutinin Disease (CHAD) :
A.
Coomb's test could be positive
B
Splenomegaly is usually marked
C.
Could be due to lymphoma
D.
MCV might be very high
E.
Blood samples should be taken into pre-warmed syringes and tubes
Q191. The following statements relate to Paroxysmal Nocturnal Hemoglobinuria :
A
Autosomal recessive
B
Positive Coomb’s test
C.
Extreme sensitivity of the red cell membrane to complement lysis
D
Fragmentation anemia
E.
Budd Chiari syndrome (Hepatic vein thrombosis)
Q192. Intravascular hemolysis could happen in the following diseases: :
A.
CHAD “Cold Hemagglutinin Disease”
B.
PNH “Paroxysmal Nocturnal Hemoglobinuria”
C.
March Hemoglobinuria
D.
Mismatched ABO-incompatible blood transfusion
E.
G-6-P Dehydrogenase deficiency
Q193. Fragmentation anemia could complicate the following conditions :
A.
Prosthetic cardiac valve
B.
Burn
C.
Hemolytic Uremic Syndrome (HUS)
D.
Thrombotic Thrombocytopenic Purpura (TTP)
E.
March hemoglobinuria
30
Professor Anwar Sheikha, MD, FRCP, FRCPath., FACP
One Thousand Hematology MCQ
Select the most appropriate answer:
Q194. One of the following features is not true about Hereditary Elliptocytosis:
A.
It is an autosomal dominant disease
B.
Usually associated with mild splenomegaly
C.
Reticulocytosis is a common feature
D.
Usually results from structural membrane protein defects
E
HbF could be high
Q195. Splenomegaly is not a prominent feature of one of the following hemolytic
anemias :
A.
Hereditary spherocytosis
B.
Sickle cell disease in early childhood
C.
Thalassemia major in adolescents
D
Paroxysmal Nocturnal Hemoglobinuria
E.
Osteopetrosis
Q196. One of the following is not true in the W.H.O. Mantle Cell Lymphoma:
A.
Described as Centrocytic lymphoma in the Kiel classification
B
It has been found that these cells definitely come from the germinal
centers
C.
The lymphoma cells are small to medium sized with cleaved nuclei
D.
Lymphomatous polyposis is a GIT form of Mantle cell lymphoma
E.
This type of lymphoma is usually nodular
Q197. All of the following cell markers are true in Mantle cell lymphoma EXCEPT:
A.
CD5 is commonly present
B.
Pan B cell markers (CD19, CD20, CD22 & CD79a) are positive
C.
CD 23 is lacking
D.
Cyclin D1 expression is common and diagnostically valuable
E
TdT is usually positive
Q198. All of the following feature are true in Mantle cell lymphoma EXCEPT:
A.
t(11;14) is a characteristic cytogenetic abnormality
B
Blood and marrow spread is rare
C.
Prognosis and long term survival is poor
D.
Contrary to old belief, cells do not come from the germinal centers
E.
The growth pattern is commonly nodular or mantle zone pattern
Q199. All of the following features are related to Infectious Mononucleosis EXCEPT:
A
Results from infection with Cytomegalovirus (CMV)
B.
Positive Monospot test
C.
Atypical lymphocytosis
D.
Possible splenic rupture
E.
Development of antibodies to Epstein Barr virus (EBV)
Q200. One of the quantitative blood cell changes is not seen in Hodgkin lymphoma:
A.
Eosinophilia
B
Basophilia
C.
Lymphopenia
D.
Thrombocytosis
E.
Anemia
31
Professor Anwar Sheikha, MD, FRCP, FRCPath., FACP
One Thousand Hematology MCQ
Match the following types of leukemias with their morphological, cytochemical,
immunological and clinical features. Each choice is used once only.
(201).
(202).
(203).
(204).
(205).
(A).
(B).
(C).
(D).
(E).
Pre-B Acute Lymphoblastic Leukemia
Acute Hypergranular Promyelocytic Leukemia (M3)
Chronic Myeloid Leukemia
Acute Myeloblastic Leukemia without Differentiation
T- Acute Lymphoblastic Leukemia
……...
……...
……...
……...
……...
A leukemic patient bleeding from Disseminated Intravascular Coagulation.
Leukemic Promyelocytes with bundles of Auer rods forming “Faggot” cells seen.
There is translocation between chromosomes 15 and 17.
A Middle aged patient with a very high white cell count without blasts.
BCR-ABL fusion gene detected . Spleen is very big.
A child with a very high white cell count, most of which are blasts. Blasts are
positive for acid phosphatase. Chest x-ray showed mediastinal widening.
Immunological studies revealed blasts positive for CD5, CD7 and T- antigen.
A young patient with leukemia and 90% blasts.
Blasts are positive for Sudan Black, Myeloperoxidase and Specific Esterase.
No differentiation to promyelocytes seen.
A five year old child with anemia, neutropenia and thrombocytopenia.
Bone marrow is full of blasts that are positive for CD 10, Cyt.u and TdT.
Child responded nicely to chemotherapy and got cured of his disease.
SELECT THE MOST APPROPRIATE SINGLE ANSWERS:
Q206. Pan B cell markers (CD19, 20, 22 & 79a) are positive in all but one of these:
A
Sezary syndrome
B.
Follicular center cell lymphoma
C.
B- cell small lymphocytic lymphoma
D.
Hairy cell leukemia
E.
Diffuse large B- cell lymphoma
Q207. All of the following features are related to "HYPERSPLENISM" EXCEPT:
A.
Splenomegaly
B.
Anemia
C.
Active bone marrow
D
Thrombocytosis
E.
Neutropenia
Q208. Erythrocyte Sedimentation Rate (ESR) is usually low in one of the following:
A
Polycythemia
B.
Anemia
C.
Multiple Myeloma
D.
Rheumatoid arthritis
E.
Tuberculosis
Q209. All of these epithelial changes could be seen in severe iron deficiency EXCEPT:
A.
Koilonychia
B.
Brittle nails
C.
Dysphagia from esophageal web
D
Subacute combined degeneration of spinal cord
E.
Glossitis
32
Professor Anwar Sheikha, MD, FRCP, FRCPath., FACP
One Thousand Hematology MCQ
Q 210. Vitamin B12 deficiency is related to any of the following conditions EXCEPT:
A.
Pernicious anemia
B
Myelofibrosis
C.
Gastrectomy
D.
Crohn's disease
E.
Veganism
Q211. The following features relate to hereditary spherocytosis :
A.
It is autosomal recessive
B.
Usually associated with marked unconjugated hyperbilirubinemia
C.
Usually associated with marked conjugated hyperbilirubinemia
D.
Splenectomy reverses the red cell shape change to biconcave form
E
Severe unexpected anemia could happen from hemolytic crisis
Q212. One of the following features is not true in Adult T-Cell Leukemia/Lymphoma:
A
Hypocalcaemia
B.
Viral etiology
C.
Common in Japan
D.
Leukemic blood picture
E.
Convoluted T lymphoma cells
Q213. One of the following features is not true in Small Lymphocytic Lymphoma:
A.
Can be confused with Chronic Lymphocytic Leukemia (CLL)
B.
Can terminate in Richter's syndrome
C
Common in children
D.
Hypogammaglobulinemia is common
E.
Although it is of low grade nature, treatment is usually not curative
Q214. Which one of the following is regarded as "HIGH GRADE" lymphoma:
A.
Small lymphocytic Lymphoma
B.
Lymphocyte Rich Hodgkin Lymphoma
C.
Mantle cell Lymphoma
D
Lymphoblastic Lymphoma
E.
Lympho-plasmacytic Lymphoma
Q215. All of the following diseases and features relate to Skin lymphomas EXCEPT:
A.
Sezary Syndrome
B.
Mycosis Fungoides
C
Often of B cell lymphocyte origin
D.
Erythroderma and presence of cerebriform lymphocytes in the blood
E.
Adult T cell Leukaemia/Lymphoma usually present with skin lesions
Q216. One of the following features relate to Mixed cellularity Hodgkin's Disease:
A.
Lacunar Cells
B.
Mediastinal involvement
C.
Alcohol-induced nodal pain
D
The commonest type in Saudi Arabia
E.
Bands of collagen separating cellular lymphoma nodules
33
Professor Anwar Sheikha, MD, FRCP, FRCPath., FACP
One Thousand Hematology MCQ
Q217. All of the following features are true for Sezary Syndrome EXCEPT:
A.
T-Helper cell lymphoma
B
Caused by HTLV-1 virus
C.
Involvement of the skin and the blood
D.
Erythroderma
E.
It usually passes through patch, plaque and tumor stages
Q218. All of the following features are true for Lymphoblastic lymphoma EXCEPT:
A.
Usually affects children
B
"Starry-Sky" tissue appearance
C.
Mediastinal involvement is common
D.
Usually have a leukemic phase
E.
Cells have distinctive convoluted nuclear morphology
Q219. Paraproteins could be seen in all of the following neoplasms EXCEPT:
A
Acute Myeloblastic Leukemia (AML)
B.
Waldenstrom's Macroglobulinemia
C.
Multiple Myeloma
D.
Chronic Lymphocytic Leukemia (CLL)
E.
Lymphomas
Q220. All of the following features could be seen in Multiple Myeloma EXCEPT:
A.
Rouleaux formation in the blood smear
B
Painful osteolytic skull lesions
C.
Osteolytic lesions in the bones
D.
Bence-Jones protein in the urine
E.
An ESR of 150 mm/hr
Match the following hemoglobin electrophoresis pattern with the corresponding
hemoglobinopathy. Each choice is used once only.
(221).
(222).
(223).
(224).
(225).
β- thalassemia major
Sickle cell disease
β - thalassemia minor
Sickle cell trait
α- thalassemia
…….
…….
…….
…….
…….
(A).
(B).
(C).
(D).
(E).
Hemoglobin F of 100%
Hemoglobin S of 100%
Hemoglobin A2 of 8%
Hemoglobin S of 40%
Hemoglobin H is increased.
THE FOLLOWING QUESTIONS ARE RELATED TO HEMATOLOGICAL
EMERGENCIES & BLOOD PRODUCTS. SELECT THE MOST APPROPRIATE.
34
Professor Anwar Sheikha, MD, FRCP, FRCPath., FACP
One Thousand Hematology MCQ
Q226. A BLEEDING CHILD CAN HAVE ANY, BUT ONE, OF THE FOLLOWING
BLOOD DISORDERS:
A.
Aplastic Anemia
B
Multiple Myeloma
C.
Acute Leukemia
D.
Immune Thrombocytopenic Purpura
E.
Hemophilia
Q227. A SEVERELY ANEMIC CHILD ADMITTED ON EMERGENCY BASIS
MIGHT HAVE ANY, BUT ONE, OF THE FOLLOWING CONDITIONS:
A.
Glucose-6-Phosphate Dehydrogenase Deficiency
B.
Acute Lymphoblastic Leukemia
C.
Hemolytic crisis of sickle cell disease
D.
Autoimmune hemolytic anemia
E
Chronic lymphocytic leukemia
Q228. ONE OF THE FOLLOWING FEATURES RELATE TO CHILDHOOD ITP:
A.
Hemarthrosis
B.
Mostly a female disease
C.
Autosomal recessive
D
Self-limiting in the majority of cases
E.
Splenectomy is the therapy of choice
Q229. THE FOLLOWING FACTS RELATE TO PLATELET CONCENTRATE :
A
One unit should be given for each 10 Kg body weight
B.
It provides all the coagulation factors
C.
The shelf life in the refrigerator is 5 days
D.
The shelf life at room temperature is 24 hours
E.
Should be given to every child with immune thrombocytopenic purpura
Q230. ALL, BUT ONE, OF THE FOLLOWING BLOOD PRODUCTS CAN BE GIVEN
WITHOUT CROSS MATCHING:
A
Packed red cells
B.
Albumin
C.
Fresh frozen plasma
D.
Platelet concentrate
E.
Factor VIII concentrate
EACH OF THE FOLLOWING MULTIPLE CHOICE QUESTIONS IS COMPOSED OF
A STEM AND FIVE STATEMENTS. EACH OF THE STATEMENTS COULD BE TRUE
OR FALSE. IN YOUR ANSWER SHEET, YOU ARE REQUESTED TO FILL THE
CORRECT ONES AND LEAVE THE FALSE ONES BLANK.
Q231. ESR IS USUALLY INCREASED IN:
A.
Lymphomas
B
Light chain only multiple myeloma
C.
Anemia
D
Secondary polycythemia
E
Hyopofibrinogenemia
35
Professor Anwar Sheikha, MD, FRCP, FRCPath., FACP
One Thousand Hematology MCQ
Q232. PANCYTOPENIA COULD RESULT FROM THE FOLLOWING CONDITIONS :
A.
Aplastic Anemia
B.
Osteopetrosis
C
Diamond-Blackfan Syndrome
D.
Kala Azar with hypersplenism
E.
Fanconi's anemia
Q233. GROSS SPLENOMEGALY COULD COMPLICATE FOLLOWING DISEASES :
A.
Osteopetrosis
B.
Thalassemia major
C.
Primary Lymphoma of the spleen
D.
Gaucher's disease
E.
Myelofibrosis
Q234. NEUTROPENIA COULD BE A FEATURE OF THE FOLLOWING CONDITIONS:
A.
Chloramphenicol therapy
B.
Typhoid fever
C
Staphylococcal osteomyelitis
D
Chronic Granulocytic Leukemia
E.
Megaloblastic anemia
Q235. MONOCYTOSIS IS A USUAL FEATURE OF THE FOLLOWING DISEASES :
A
Filariasis
B.
Tuberculosis
C
Hairy cell leukemia
D.
Chronic neutropenia
E.
Chronic myelomonocytic leukemia
Q236. THE FOLLOWING STATEMENTS RELATE TO AGRANULOCYTOSIS :
A
Severe neutrophilia
B.
Mouth ulceration
C
Gross splenomegaly
D.
Could result from Phenylbutazone therapy
E.
Hyperpyrexia
Q237. THE FOLLOWING RELATES TO GRANULOCYTIC LEUKEMOID REACTION :
A.
Marked neutrophilia
B
Low Neutrophil Alkaline Phosphatase score
C
Positive Philadelphia chromosome
D.
Can result from severe infection, intoxication or malignancy
E.
Toxic granulation of the neutrophils
Q238. LEUKO-ERYTHROBLASTIC ANEMIA DENOTES :
A.
Presence of nucleated red cells in the peripheral smear
B.
Presence of maturing myeloid cells in the peripheral blood smear
C.
Marrow infiltration by Malignancy, Myelofibrosis or myeloma
D
Drug overdose
E
Presence of blasts in the bone marrow
36
Professor Anwar Sheikha, MD, FRCP, FRCPath., FACP
One Thousand Hematology MCQ
Q239. THE FOLLOWING STATEMENTS RELATE TO MULTIPLE MYELOMA :
A.
Majority of cases have IgG Myeloma
B.
Light chains could be detected in the urine in half of all cases
C.
IgM Myeloma is extremely rare
D
Severe pain in the skull bones is a common finding
E
Bone marrow infiltration by Reed Sternberg cells
Q240. THE FOLLOWING STATEMENTS RELATE TO WALDENSTROM'S
MACROGLOBULINEMIA :
A
Elevated level of polyclonal IgM
B.
Elevated level of monoclonal IgM
C.
Elevated ESR
D.
Hyperviscosity syndrome
E
Elevated IgG
Q241. ANY OF THE FOLLOWING FEATURES FULFILL THE CRITERIA FOR
HIGH CELL MASS MYELOMA ( STAGE III ):
A
Hemoglobin of 120 g/L
B.
Calcium of 3.3 mmol/L
C.
Multiple osteolytic bone lesions
D.
IgG of 60 g/L
E
Bence Jones protein of 2 g/ 24 hour urine
Q242. PURPURA COULD BE MANIFESTATION OF THE FOLLOWING DISORDERS:
A.
Immune Thrombocytopenic Purpura (ITP)
B
Hemophilia
C.
Essential Thrombocythemia
D.
Disseminated Intravascular Coagulopathy (DIC)
E.
Thrombotic thrombocytopenic purpura (TTP)
Q243. THE FOLLOWING FEATURES RELATE TO HODGKIN'S DISEASE :
A.
Painless progressive peripheral lymphadenopathy
B.
Usually affects children in the underdeveloped world
C.
More common in the young adult in the developed world
D.
All age groups in Saudi Arabia
E.
Curable in the majority of cases
Q244. THE FOLLOWING CYTOGENETIC ABNORMALITIES COULD BE SEEN
IN BURKITT'S LYMPHOMA :
A.
t(8;14)
B.
t(2; 8)
C.
t(8;22)
D
t(15;17)
E
t(9;22)
Q245. THE FOLLOWING FEATURES RELATE TO INFECTIOUS MONONUCLEOSIS :
A.
Atypical lymphocytosis
B
The lymphoid cells are basically B- Lymphocytes
C.
B- cells and pharyngeal mucosa have receptors for the virus
D.
Caused by Epstein Barr virus
E.
Monospot test is positive
37
Professor Anwar Sheikha, MD, FRCP, FRCPath., FACP
One Thousand Hematology MCQ
Q246. THE FOLLOWING STATEMENTS RELATE TO DIFFERENT FAB TYPES OF
ACUTE LYMPHOBLASTIC LEUKAEMIA (ALL):
A.
L1 is leukemia of small and monomorphic lymphoblasts
B.
L2 is leukemia of large and heterogeneous lymphoblasts
C
L3 is Burkitt’s lymphoma
D.
Burkitt leukemic cells have vacuolated deeply basophilic cytoplasm
E.
L3 and B-ALL are the same
Q247. THE FOLLOWING RELATE TO T- CELL LARGE GRANULAR LEUKEMIA :
A.
The disease is usually indolent
B.
Patients could be asymptomatic for a long time
C.
Neutropenia and recurrent infections are common
D.
Rheumatoid arthritis is a strong association
E.
Prognosis is usually good
Q248. THE FOLLOWING RELATE TO NK CELL LARGE GRANULAR LEUKEMIA :
A.
The disease is usually more aggressive than T- cell type
B.
Arthritis and symptomatic neutropenia are rare
C.
The leukemia tends to follow an aggressive course
D.
Patients are younger and B- symptoms more common than T-cell type
E.
Hepato-splenomegaly, GIT involvement and coagulopathy are common
Q249. THE FOLLOWING COMPLICATIONS CAN FOLLOW BLOOD TRANSFUSION:
A.
Hemolytic transfusion reaction
B.
Febrile transfusion reaction
C
Iron deficiency anemia
D.
AIDS
E
Ulcerative colitis
Q250. HEMOLYTIC TRANSFUSION REACTION USUALLY PRESENTS WITH THE
FOLLOWING FEATURES :
A
Fever as the only and sole manifestation
B.
Heat feeling in the vein into which the blood has been transfused
C.
Lumbar pain
D.
DIC
E
Hypertension
Q251. THE FOLLOWING DISEASES CAN BE TRANSMITTED THROUGH BLOOD
TRANSFUSION :
A.
Syphilis
B
Gonorrhea
C.
Malaria
D
Hydatid disease
E.
Hepatitis
Q252. THE FOLLOWING SUPPORT THE DIAGNOSIS OF POLYCYTHEMIA RUBRA
VERA (PRV) RATHER THAN SECONDARY POLYCYTHEMIA :
A.
Splenomegaly
B.
Neutrophilia
C.
Associated Gout
D
High ESR
E
Low plasma volume
38
Professor Anwar Sheikha, MD, FRCP, FRCPath., FACP
One Thousand Hematology MCQ
Q253. THE FOLLOWING FEATURES ARE MORE SUGGESTIVE OF ESSENTIAL
THROMBOCYTHEMIA (ET) THAN REACTIVE THROMBOCYTOSIS:
A.
Platelet count of more than a million/uL
B.
Splenic atrophy
C.
Prolonged bleeding time
D.
Abnormal platelet function tests
E
Recent history of splenectomy
Q254. REACTIVE THROMBOCYTOSIS CAN FOLLOW ANY OF THE FOLLOWING :
A.
Iron deficiency
B
Aplastic anemia
C
Kala Azar
D
Acute lymphoblastic leukaemia
E.
Operative surgery
Q255. Which of the following is correct :
A
The average life span of the normal red cell is 60 days
B
The hemoglobin molecule contains two globin chains
C
In normal individuals most red cells are destroyed in the bone marrow
D.
Red cells principally metabolize glucose
E.
The average half-life for decay of radioactive chromium labeled
normal red cells is around 30 days
Q256. Severe iron deficiency anemia may be associated with :
A
Clubbing of the fingers
B.
Pica
C.
Stomatitis
D
Peripheral neuropathy with loss of ankle jerks
E
Hyperemia and ulceration of the lower rectum
Q257. Pernicious anemia could be associated with :
A.
Premature graying of hair
B
Brown eyes
C
Wide jaw bones
D.
Blood Group A
E.
Vitiligo
Q258. Drugs which may be associated with Immune Hemolytic Anemia include:
A
Digoxin
B
Aspirin
C.
Isoniazid
D.
Sulphonamides
E.
Methyl dopa
Q259. Which of the following statements are correct :
A
15% of the male population are Rhesus positive
B
Rhesus positive males are always homozygous
C
Naturally occurring Anti-A agglutinin causes marked hemolysis
D.
The indirect Coomb's test is of value in cross-matching of blood
E
Transfusion of Rhesus incompatible blood is only safe in males
39
Professor Anwar Sheikha, MD, FRCP, FRCPath., FACP
One Thousand Hematology MCQ
Q260. Which of the following diseases may be associated with purpura ?
A
Thyrotoxicosis
B
Typhoid fever
C.
Systemic Lupus Erythematosus
D
Thymic tumor
E.
Typhus
Correlate the following diseases with the most compatible values & features:
A.
Myelofibrosis
B.
Multiple Myeloma
C.
Polycythemia Rubra Vera
D.
Essential Thrombocythemia
E.
Hb
Hypersplenism
WBC
Platelet
Blood Smear
Low
+
Normal
Other
Features
Huge Spleen
High
Normal
Plethoric
Tear drop RBC
Leukoerythroblastic
Increased
marrow reticulin
Rouleaux
Formation
Marrow Plasma
Cells of 60%
261
Low
+
Low
+
262
High
High
+
+++
Diagnosis
++
Low
+
High
+
264
Low
+
Low
+
265
Low
+
263
High
+
Low
+
High
High
+
++++
Full of Platelets
Splenic atrophy
Match the following types of leukemias with their corresponding features.
Put down the correct letter in the blank bracket.
Q266. Acute Myeloid Leukemia
(
) (A). Non-specific Esterase +
Q267. Chronic Myeloid Leukemia
(
) (B). Hairy lymphocytes.
Q268. Hairy cell Leukemia
(
) (C). BCR/-ABL transcription gene
Q269. Acute Monoblastic Leukemia
(
) (D). TdT+/ CD 10+/ HLA-DR+
Q270. Pro-B
(
) (E). Sudan Black, Specific
Acute Lymphoblastic Leukemia
Esterase & Myeloperoxidase +
Match the following blood diseases with the corresponding red cell abnormalities
Q271.
Q272.
Q273.
Q274.
Q275.
Q276.
Q277.
Q278.
Q279.
Q280.
Hereditary elliptocytosis
Uremia
Iron deficiency anemia
Coomb’s positive warm AIHA
Sickle cell anemia
MAHA
Hemoglobin H disease
Myelofibrosis
Megaloblastic anemia
G-6-P DH deficiency
(
(
(
(
(
(
(
(
(
(
)
)
)
)
)
)
)
)
)
)
(A).
(B).
(C).
(D).
(E).
(F).
(G).
(H).
(I).
(J).
Hypochromic cells
Bite or blister cells
Oval macrocytosis
Tear drop cells
Sickle cells
Burr cells
Golf ball red cells
Spherocytosis.
Elliptocytosis
Fragmented cells
40
Professor Anwar Sheikha, MD, FRCP, FRCPath., FACP
One Thousand Hematology MCQ
Match the following hematological diagnoses with their correct criteria.
Q281. Iron deficiency anemia
Q282. Megaloblastic Anemia
Q283. Thalassemia major
Q284. Sickle cell disease
Q285. G6PDH deficiency
Q286. Autoimmune hemolytic Anemia
Q287. Thalassemia minor
Q288. Acute lymphoblastic leukemia
Q289. Acute Myeloblastic leukemia
Q290. Chronic Granulocytic Leukemia (
Q291. Chronic Lymphocytic Leukemia (
Q292. Hairy cell leukaemia
(
Q293. Myelodysplastic syndromes
(
Q294. Hodgkin's Disease
(
Q295. Non-Hodgkin's Lymphoma
(
Q296. Myelofibrosis
(
Q297. PRV
(
Q298. Essential thrombocythemia
(
Q299. Multiple myeloma
(
Q300. Waldenstrom macroglobulinemia (
Q301. Hemophilia A
(
Q302. Hemophilia B
(
Q303. von Willebrand's Disease
(
Q304. DIC
(
Q305. Celiac disease
(
(
(
(
(
(
(
(
(
(
)
)
)
)
)
)
)
)
)
)
)
)
)
)
)
)
)
)
)
)
)
)
)
)
)
A. High hemoglobin A2
B. High hemoglobin S
C. High hemoglobin F
D. Favism
E. Positive Coomb’s test
F. PAS positive blast cells
G. Sudan Black + blasts
H. High MCV
I. Koilonychia
J. Massive splenomegaly/Dry tap
K. High hemoglobin
L. Pre-leukemia
M. Reed Sternberg Cells (RSC)
N. Lymphoma without RSC
O. High mature & maturing WBC
P. High lymphocyte count
Q. Lymphocytosis with hairy cells
R. Splenic atrophy in 1/3 cases
S. Monoclonal IgM
T. Monoclonal IgG
U. Splenic atrophy/ Weight loss
V. Consumption coagulopathy
W. Prolonged APTT/ menorrhagia
X. Reduced factor IX
Y. Reduced factor VIII:c
Correlate the histological type of non-Hodgkin’s Lymphoma with its grade of
aggressiveness. Fill-in the space with “A” for Low Grade type; “B” for Intermediate
Grade and “C” for High Grade type according to the old Working Formulation.
As you know, we are now using the new W.H.O classification, but knowing the aggressiveness
of the disease is still very important.
HISTOLOGICAL TYPE
AS IN WORKING FORMULATION
306
307
308
309
310
311
312
313
314
315
Small lymphocytic Lymphoma
Burkitt’s Lymphoma
Lymphoblastic Lymphoma
Immunoblastic Lymphoma
Follicular Small Cleaved Cell Lymphoma
Follicular Mixed Lymphoma
Diffuse Small Cleaved Cell Lymphoma
Diffuse Mixed Lymphoma
Diffuse large Cell Lymphoma
Follicular Large cell Lymphoma
GRADE
41
Professor Anwar Sheikha, MD, FRCP, FRCPath., FACP
One Thousand Hematology MCQ
EACH ITEM BELOW CONTAINS FOUR SUGGESTED ANSWERS, OF WHICH ONE
OR MORE IS CORRECT. CHOOSE THE CORRECT LETTERED ANSWER
ACCORDING TO THE FOLLOWING FORMAT:
A
if 1,2 and 3
are correct
B
if 1 and 3
are correct
C
if 2 and 4
are correct
D
if 4 alone
is correct
E
if 1,2,3 and 4 are correct
Q316. Patients with Hodgkin’s Disease are immunocompromized because of :(1).
Lymphocytopenia secondary to the advanced nature of the disease.
(2).
Splenectomy as part of the staging procedure.
(3).
Anergy from impaired cell-mediated immunity.
(4).
Chemo-radiotherapy as part of the management plans.
Q317. The histological types of Hodgkin’s Disease include:
(1).
Nodular sclerosis with Lacunar type of Reed Sternberg cells.
(2).
Lymphocyte predominant with the popcorn type of Reed Sternberg cells.
(3).
Mixed cellularity type which is the commonest type in Abha.
(4).
Lymphocyte depleted which is the worst type of the Hodgkin’s Disease.
Q318. The following cells could be seen in Mixed Cellularity Hodgkin’s Disease:
(1).
Reed Sternberg cells.
(2).
Eosinophils.
(3).
Plasma cells.
(4).
Lymphocytes.
Q319. The following types of Non-Hodgkin’s Lymphoma come under the “Low-Grade”
group in the old Working Formulation Classification:
(1).
Burkitt’s Lymphoma
(2).
Lymphoblastic Lymphoma
(3).
Immunoblastic lymphoma
(4).
Small Lymphocytic lymphoma.
Q320. Viruses could be the cause of the following Lymphomas:
(1).
Human T cell Leukemia/Lymphoma.
(2).
Mycosis Fungoides
(3).
Burkitt’s Lymphoma.
(4).
Mediterranean Lymphoma.
Q321. The following types of lymphomas are grouped under the “Intermediate-Group”
according to the old Working Formulation:
(1).
Diffuse Large Cell Lymphoma.
(2).
Follicular Large Cell Lymphoma.
(3).
Diffuse Small Cleaved Cell Lymphoma.
(4).
Diffuse Mixed Cell Lymphoma.
Q322. The following features relate to Burkitt’s Lymphoma:
(1).
Probably the most aggressive human tumor.
(2).
Probably the most homogenous human tumor.
(3).
Starry-sky histological picture.
(4).
The tumor can double its mass in only 24 hours.
42
Professor Anwar Sheikha, MD, FRCP, FRCPath., FACP
One Thousand Hematology MCQ
Q323. The following features relate to Multiple Myeloma:
(1).
Bone marrow Infiltration by plasma cells.
(2). IgM paraproteinemia
(3).
Osteolytic bone lesions.
(4).
Viral etiology
Q324. Waldenstrom’s macroglobulinemia can present with the following features:
(1).
IgG paraproteinemia
(2).
Hyperviscosity.
(3).
Pautrier’s abscesses in the skin
(4).
Lymphoma-like picture with splenomegaly and lymphadenopathy.
Q325. Renal failure commonly complicate myeloma because of the following reasons:
(1).
Hypercalcemia.
(2).
Heavy chain damage to the kidney tubules.
(3).
Secondary amyloid deposition in the glomeruli.
(4).
Light chain damage to the renal tubules.
Q326. The following features strongly support the diagnosis of polycythemia rubra vera:
(1).
High red cell mass (>36 ml/Kg in male & > 32 ml/Kg in female)= A1.
(2).
Normal Oxygen saturation of > 92%= A2.
(3).
Moderate splenomegaly=A3.
(4).
B group of features( Leukocytosis, thrombocytosis, High LAP score & B12).
Select the most appropriate ANSWER.
Q327. Any, but one, of the following can complicate hemolytic transfusion reaction:
A.
Renal failure
B.
DIC
C.
Hypertension.
D.
Spherocytosis
E.
Hemoglobinemia, hemoglobinuria & hemosiderinuria
Q328. In Secondary Polycythemia:
A.
Spleen is usually enlarged
B.
White cells and platelets are also increased
C.
There is tendency for platelet dysfunction and bleeding
D.
Erythropoietin is usually increased.
E.
Thromboembolism is common
Q329. Direct Antiglobulin (Coomb's) Test could be positive in:
A.
Hereditary spherocytosis
B.
Hereditary elliptocytosis
C.
Glucose 6-Phosphate Dehydrogenase Deficiency
D.
Thalassemia
E.
Hypertensive patients on Methyldopa.
Q330. Hemoglobin F is composed of:
A.
α2 β 2
B.
α2 δ 2
C.
α2 γ2 .
D.
γ4
E.
β4
43
Professor Anwar Sheikha, MD, FRCP, FRCPath., FACP
One Thousand Hematology MCQ
Q331. All, but one, of the following RBC abnormalities are grouped as "Poikilocytosis":
A.
Target Cells
B.
Sickle Cells
C.
Tear Drop Cells
D.
Spherocytes .
E.
Elliptocytes
Q332. Which one of the following statements relate to Acute Lymphoblastic Leukemia:
A.
Results from proliferation of Myeloblasts
B.
Mature and maturing white blood cells are increased
C.
It is the commonest cancer in children.
D.
Blasts are positive for Sudan Black
E.
Never cured by chemotherapy alone
Q333. Neutropenia could be seen in all of the following diseases EXCEPT:
A.
Agranulocytosis
B.
Hepatitis
C.
Chronic Myeloid Leukaemia (CML) .
D.
Influenza
E.
Severe megaloblastic anemia
Q334. Paraproteins could be seen in all of the following neoplasms EXCEPT:
A.
Monoclonal Gammopathy of Undetermined significance
B.
Waldenstrom's Macroglobulinemia
C.
IPSID “Immunoproliferative Small Intestinal Disease”
D.
Chronic Lymphocytic Leukaemia (CLL)
E.
Burkitt lymphoma.
Q335. Burkitt Lymphoma could present itself with all, but one, of the following features
A.
Pautrier's abscess formation.
B.
Intestinal obstruction in a child
C.
Tumor necrolysis syndrome after treatment
D.
Extreme sensitivity to Cyclophosphamide
E.
Aggressive nature
Q336. The following relates "HYPOSPLENISM" :
A.
Splenomegaly
B.
Anemia
C.
Active bone marrow
D.
Target, pitted and Heinz-body containing red cells.
E.
Neutropenia
44
Professor Anwar Sheikha, MD, FRCP, FRCPath., FACP
One Thousand Hematology MCQ
Q337. You are provided with a table to comment on the hemostatic test results
for some common bleeding disorders. Write “P” if you think the test is prolonged
and “N” for a normal test result.
To help you, many of the answers are already correctly labeled.
The tests are:
P.T.
A.P.T.T.
meaning
meaning
Prothrombin Time
Activated Partial Thromboplastin Time
B.T.
meaning
Bleeding Time
T.T.
meaning
Thrombin Time
The bleeding diseases are:
vWD
meaning
von Willebrand’s Disease
DIC
meaning
Disseminated Intravascular Coagulation
ITP
meaning
Immune Thrombocytopenic Purpura
Hemophilia
which includes both Hemophilia A & B
DISEASE OR FACTOR
DEFICIENCY
PT
APTT
HEMOPHILIA
TT
N
vWD
N
XII & XI
N
N or P
N
N
VII
N
N
X, V & II
FIBRINOGEN
B.T.
N
P
LIVER FAILURE
P
P
N
P
N
DIC
ITP
Select True (T) and False (F) answers.
Q338. Myelofibrosis is often associated with :
A.
Large spleen.
B.
Lymphadenomegaly
C.
Philadelphia chromosome
D.
Gout.
E.
Portal hypertension.
Q339. The following are true of hemophiliacs :
A.
Genes responsible for the disorder are carried on the Y chromosome
B.
The condition is inherited as sex-linked recessive characteristic.
C.
Patients usually suffer from recurrent dislocation of joints
D.
Continued bleeding from the umbilical cord may be the earliest event.
E.
Bleeding time is grossly prolonged
45
Professor Anwar Sheikha, MD, FRCP, FRCPath., FACP
One Thousand Hematology MCQ
Q340. In Hemolytic Uremic Syndrome :
A.
Purpura is a common feature.
B.
It is much commoner in the USA than in the UK
C.
It appears suddenly in a previously healthy child.
D.
It is associated with leucopenia
E.
Hemoglobinuria may occur.
Q341. Sideroblastic anemia may be associated with :
A.
Children with pulmonary Hemosiderosis
B.
Lead poisoning.
C.
Myelodysplastic syndrome.
D.
Partially treated hemorrhagic anemia
E.
Middle aged patient with a moderate refractory normoblastic anemia.
Q342. In Thrombotic Thrombocytopenic Purpura :
A.
Fragmentation hemolytic anemia occurs.
B.
Mainly males are affected
C.
Hypertension is usual
D.
Neurological manifestations are common.
E.
There is a characteristic petechial rash.
Q343. Hemoglobinuria may occur in :
A.
Incompatible blood transfusion.
B.
Hereditary spherocytosis
C.
Hypersplenism
D.
Sickle cell anemia
E.
Drowning in fresh water.
Q344. Pernicious anemia :
A.
Occurs with equal frequency at all decades over twenty
B.
May be associated with severe glossitis.
C.
May present with mental disturbances.
D.
The spleen is usually palpable
E.
Is usually associated with mild jaundice.
Q345. The following are part of the erythroid series :
A.
Pronormoblast.
B.
Monoblast
C.
Plasma cell
D.
Megakaryocyte
E.
Promyelocyte
Q346. Bone pain is a common symptom in the following :
A.
Multiple myeloma.
B.
Severe iron deficiency anemia
C.
Sickle cell anemia.
D.
Pernicious anemia
E.
Hereditary spherocytosis
46
Professor Anwar Sheikha, MD, FRCP, FRCPath., FACP
One Thousand Hematology MCQ
Q347. In which of the following are chromosome abnormalities common :
A.
Chronic myeloid leukaemia.
B.
Erythroblastosis fetalis
C.
Oroya fever
D.
Acute leukaemia.
E.
Thalassemia
Q348. A decrease in the life span of the red cell is found in the following :
A.
Pernicious anemia.
B.
Immune Thrombocytopenic Purpura ( ITP )
C.
Thalassemia.
D.
Hemophilia
E.
Pregnancy
Q349. Which of the following are often found in the blood after splenectomy ?
A.
Megaloblasts
B.
Target cells.
C.
Howell Jolly bodies.
D.
Thrombocytosis.
E.
Leucopenia
Q350. The following drugs or toxins may be responsible for aplastic anemia :
A.
Quinidine
B.
Penicillamine.
C.
Benzene.
D.
Glibenclamide
E.
Sodium aureothiomalate.
MATCH THE FOLLOWING ABNORMAL RED BLOOD CELL FEATURES WITH
THEIR MOST PROBABLE BLOOD DISEASES.
“YOU ARE ONLY REQUESTED TO FILL IN THE SPACES WITH THE POSSIBLE
LETTERS. DON’T RE-WRITE THE WHOLE RED CELL ABNORMALITY. JUST
INSERT THE POSSIBLE CORRECT LETTER.”
(A). SHISTOCYTES (Fragmented RBC)
(F). HYPOCHROMIC MICROCYTIC RBC
(B). TEAR DROP RED CELLS
(G).
ELLIPTOCYTES
(C). SICKLE CELLS
(H).
HEINZ BODIES
(D). HOWELL-JOLLY BODIES
(I ).
SPHEROCYTES
(E). BASOPHILIC STIPPLING
(J).
BURR CELLS
POSSIBLE RED CELL ABNORMALITY
BLOOD DISEASE
351 UREMIA
352 HEREDITARY SPHEROCYTOSIS
353 G6P DEHYDROGENASE DEFICIENCY
354 HEREDITARY ELLIPTOCYTOSIS
355 MAHA “Microangiopathic Hemolytic Anemia”
356 MYELOFIBROSIS
357 SICKLE CELL ANEMIA
358 IRON DEFICIENCY ANEMIA
359 LEAD POISONING
360 MEGALOBLASTIC ANEMIA
47
Professor Anwar Sheikha, MD, FRCP, FRCPath., FACP
One Thousand Hematology MCQ
Select the most appropriate answers to these morphology questions:
Q361. The following features are true in all of these hematological cells EXCEPT :
A.
Pronormoblasts are relatively large myeloid cells with abundant
cytoplasmic granules and few nucleoli.
B.
Myeloblasts are positive for Sudan Black stain
C.
Lymphoblasts are positive for Periodic Acid Schiff (PAS) stain
D.
Myeloblasts are positive for Specific "Chloroacetate" Esterase stain
E.
Tear drop cells & leuko-erythroblastic feature are seen in myelofibrosis
Q362. All of the following features are correct about red cell anomalies, EXCEPT :
A.
Anisocytosis means variation in size
B.
Poikilocytosis means variation in shape
C.
Howell Jolly Bodies are nuclear remnants
D.
Heinz bodies are precipitated denatured hemoglobin debris
E.
Polychromasia indicates reticulocytopenia.
Q363. Reticulocytes have all of the following features EXCEPT :
A.
They are young red blood cells with fine basophilic reticulum
B.
Their number in the blood is between 10 and 100 billion per liter
C.
Their ratio in the blood is between 0.2 to 2 percent
D.
They can best be demonstrated by Supravital stains
E.
After a week in the blood they give rise to mature erythrocytes.
Q364. The following features relate to Target Cells:
A.
Large target cells are usually due to gain of red cell membrane
and characteristically seen in thalassemia major
B.
Small target cells are usually due to defective hemoglobinization
and characteristically seen in liver failure
C.
They are markedly increased after splenectomy
D.
They are markedly increased in Hemoglobin SC disease.
E.
They are usually hyperchromic cells
Q365. Siderocytes have all, but one, of the following features:
A.
They are erythrocytes containing iron granules
B.
They are Pappenheimer bodies as seen by Romanowsky stains
C.
They are best seen when stained by Prussian-blue stains
D.
Ring sideroblasts are characteristic for sideroblastic anemias
E.
They are seen in iron deficiency anemia but not in thalassemia.
Q366. Poikilocytosis could mean any one of the following EXCEPT :
A.
Blister cells
B.
Prickle cells
C.
Pear shaped cells
D.
Spherocytes.
E.
Crenated cells
48
Professor Anwar Sheikha, MD, FRCP, FRCPath., FACP
One Thousand Hematology MCQ
Q367. The following abnormalities are seen in megaloblastic anemias EXCEPT:
A.
Giant metamyelocytes in the bone marrow
B.
Hypersegmented neutrophils in the peripheral blood
C.
Defective hemoglobinization of the developing red cells.
D.
Hypercellular bone marrow
E.
Howell Jolly bodies in the red cells
Q368. All, but one, of the following abnormalities are seen in iron deficiency anemias:
A.
Hypochromic microcytic red cells
B.
Defective hemoglobinization of the developing red cells
C.
Pencil shaped red cells
D.
Ring sideroblasts in the marrow nucleated red cells.
E.
Absent stainable iron stores
Q369. All, EXCEPT one, of the following relate to the clinical features of hemolysis :
A. The patient may not be anemic
B. Splenomegaly is a common feature
C. Biliary colic may occur
D. Mild jaundice is a usual feature
E. Gall stones at young age is an absolute indication of hemolytic anemia.
Q370. All, EXCEPT one, of the following relate to laboratory features of hemolysis:
A. Reticulocytosis is a usual finding
B. Polychromasia usually detected
C. Coomb’s test is positive in all patients.
D. Cross-matching might be difficult if autoantibodies are present
E. Mild unconjugated hyperbilirubinemia is usual
LEUKEMIA IMMUNOPHENOTYPING EXMINATION:
There are five main immunological types of Acute Lymphoblastic Leukemias:
Common ALL; T-ALL; Pro-B ALL; Pre-B ALL and B-ALL. Each of these types of ALL is
characteristically positive for certain markers. The main immunological markers used
are TdT; HLA-DR; CD10; Cyt.; SmIg; CD 19 to 22, which are Pan B cell markers and
CD 2 to 8 markers, which are T cell markers.
In the following table, you are requested to fill in the positive tests with the plus (+) sign
and the negative tests with the minus (-) sign.
For your convenience, I have already marked few of the tests.
ALL
Q371
Q372
Q373
Q374
Q375
Pro-B
cALL
Pre-B
B-ALL
T-ALL
TdT
CD10
Cyt. 
SmIg CD19-22
(BLineage)
CD 2-8
(T-Lineage)
-
49
Professor Anwar Sheikha, MD, FRCP, FRCPath., FACP
One Thousand Hematology MCQ
Select the True (T) & False (F) answers:
Q376. The following compensatory mechanisms occur in severe chronic anemia:
A
Arterial PO2 is decreased
B.
Red cell 2,3 diphosphoglycerate increases
C.
There is an increase in cardiac output at rest
D.
The concentration of erythropoietin in the urine is often increased
E
Whole blood viscosity is increased
Q377. The following statements relate to the gain & loss of iron by the body:
A
Absorption takes place only in the lower part of the small intestine
B
In adult males at least 2 mg of iron is lost from the body each day
C
The average iron loss during each menstrual period is 120-150 mg
D.
Iron requirements in pregnancy are increased
E.
Adult males absorb about 1 mg of iron daily although intake is 10 mg
Q378. The following relate to the causes of iron deficiency:
A.
Partial gastrectomy may be followed by iron deficiency
B.
Iron deficiency anemia in adult male is quite worrisome
C
Iron deficiency is most frequently caused by dietary lack of iron
D.
Hemosiderin can be excreted in large quantities in the urine
E.
Women of child-bearing age are more common to develop iron deficiency
than adult males
Q379. Which of the following are consistent with diagnosis of chronic iron deficiency ?
A.
Hemoglobin of 60 gm/L, MCV of 60 fl, MCH of 15.5 pg
B.
Absent stainable iron in the marrow
C
A reticulocyte count of 3.7 %
D.
Tendency to crave for strange things like chalk and ice
E.
A platelet count of 540 thousand / uL
Q380. The following are statements about sideroblastic anemia :
A.
There is increased marrow iron with ring sideroblasts
B.
There is impaired hemoglobin synthesis
C.
A dimorphic blood picture is often found
D
The anemia is due to erythroid hypoplasia in the marrow
E.
In many patients the etiology of acquired sideroblastic anemia is obscure
Q381. The following are statements about Erythrocyte Sedimentation Rate (ESR):
A.
The rate of sedimentation in one hour at 200 C is about 4 mm in men
B.
It is raised in anemia
C
It is high in defibrinated blood
D
It can be used to exclude presence of underlying organic disease
E.
The ESR is useful in following the progress of diseases such as
tuberculosis and rheumatoid arthritis
Q382. The following statements relate to the Anemia of Chronic Disorders :
A
This type of anemia is uncommon
B.
The blood picture is usually normocytic normochromic
C.
In the bone marrow, stainable iron stores are normal or increased
D
The serum iron and total iron binding capacity ( TIBC ) are raised
E.
The ESR is usually elevated
50
Professor Anwar Sheikha, MD, FRCP, FRCPath., FACP
One Thousand Hematology MCQ
Q383. The following are statements concerning some of the hematological
findings during pregnancy :
A.
The hemoglobin concentration decreases
B.
The red cell mass increases
C
The ESR is lower
D.
The concentration of many coagulation factors is increased
E.
A total white cell count of 20 x 109/l following delivery is common
Q384. The following concern vitamin B12 :
A.
It is plentiful in a mixed diet, so inadequate intake is rare
B
The daily requirement of vitamin B12 is about 100 microgram
C.
The body stores can last 2-3 years
D.
Pernicious anemia is due to lack of intrinsic factor production
E
Vitamin B12 is absorbed in the upper jejunum
Q385. Common features of megaloblastic anemias are as follows :
A.
A raised MCV with oval macrocytosis in the peripheral blood smear
B.
There is hyperplastic erythropoiesis in the bone marrow
C
Pancytopenia is rare
D.
Hypersegmented neutrophils may be present in the peripheral blood
E.
The typical megaloblastic bone marrow and macrocytic red cells
may be masked if concomitant iron deficiency exists
Q386. The following are related to the diagnosis of pernicious anemia (PA) :
A.
The serum B12 is always reduced
B.
The red cell folate is low in 60% of patients
C
A patient with a megaloblastic anemia, histamine fast achlorhydria
and low serum B12 must have PA
D
A low radioactive vitamin B12 excretion in the urine ( Schilling
test ) uncorrected by the oral administration of IF is usual
E
The presence of IF antibodies is essential to confirm the diagnosis
Q387. The following are statements concerning β-thalassemia major :
A.
Iron overload is the major cause of death
B
Severe anemia occurs in the neonatal period
C.
Gross hepatosplenomegaly is common
D.
There is often evidence of endocrine disease
E.
A pancytopenia may develop
Q388. The following concern a peripheral blood neutrophil leukocytosis :
A
If neutrophils comprise 80% of a total WBC of 8,000/uL, then,
by definition, a neutrophil leukocytosis exists
B.
It is commonly caused by an acute pyogenic infection
C
A postoperative neutrophilia indicates associated infection
D.
It often follows severe hemorrhage
E.
It is characteristic of most myeloproliferative diseases
51
Professor Anwar Sheikha, MD, FRCP, FRCPath., FACP
One Thousand Hematology MCQ
Q389. The following concern aplastic anemia:
A.
There is commonly a pancytopenia
B.
A bone marrow aspirate can appear to be of normal cellularity
C
Splenomegaly is a consistent feature at the time of diagnosis
D.
The plasma erythropoietin concentration is usually high
E
The reticulocyte count is usually greater than 2%
Q390. The following concern the presenting features of Acute Leukemia:
A.
The presenting symptoms are due to bone marrow failure
B.
Fever can occur without obvious infection
C
Enlargement of the spleen below the umbilicus is common
D.
Bone pain can be a feature
E
Widespread lymphadenomegaly is a diagnostic feature
Q391. The following are statements concerning Acute Lymphoblastic Leukemia:
A
The lymphoblasts in culture have a rapid cell cycle
B.
Immunophenotyping of the lymphoblasts is essential
C.
There is sometimes an associated mediastinal mass
D
Lymphoblasts are always found in the peripheral blood
E.
A high WBC & moderate splenomegaly are poor prognostic features
Q392. The following concern Acute Myeloblastic Leukemia ( AML ) :
A
The absence of physical signs excludes the diagnosis of AML
B.
There are distinct morphological variants
C.
AML is less common in children than ALL
D.
Cytogenetic studies suggest the presence of an abnormal cell
line in many patients
E.
Some patients present with severe bleeding due to coagulation
factor deficiencies
Q393. The following are statements about the clinical findings of
Chronic Granulocytic Leukemia (CGL ) :
A.
Night sweats and weight loss are common
B.
Splenomegaly is the major clinical sign
C.
Patients can present with priapism
D.
Bruising may occur despite a normal platelet count
E
Lymphadenomegaly is common at presentation
Q394. The following relate to Chronic Lymphocytic Leukemia :
A.
It is discovered as an incidental finding in 25% of cases
B.
Hepatosplenomegaly may occur without lymph node enlargement
C.
Thrombocytopenia indicates an advanced stage
D.
Hemolytic anemia may complicate the disease
E.
Herpes zoster infections are quite common
Q395. Infiltration of the bone marrow by myeloma cells could cause the following :
A.
Osteolytic bone lesions
B.
A normocytic normochromic anemia
C
Circulating nucleated RBCs & immature myeloid cells in 60% of cases
D.
The appearance of plasma cells in the peripheral blood
E
Hypocalcaemia
52
Professor Anwar Sheikha, MD, FRCP, FRCPath., FACP
One Thousand Hematology MCQ
Q396. The following relate to the treatment of myelomatosis :
A.
Alkylating agents are the mainstay of treatment
B
Therapeutic irradiation is contraindicated
C.
Hypercalcemia must be treated urgently
D.
Prophylactic antibiotics may be indicated
E.
Treatment is best monitored by serial paraprotein measurements
Q397. The following concern patients with Polycythemia :
A.
The red cell mass estimation was a must in the past for diagnosis
B.
The total blood volume is usually increased in Polycythemia Rubra Vera
C
Blood viscosity remains unaltered in Polycythemia Rubra Vera
D
Platelet function in PRV is usually normal
E.
Pruritus could be from Hyperhistaminemia secondary to basophilia
Q398. The following statements relate to Myelofibrosis :
A.
Reticulin fibers are increased in the bone marrow
B.
The development of myelofibrosis can be preceded by PRV
C.
Extramedullary hemopoiesis is a cardinal feature of the disease
D
The disease is often found incidentally
E.
Palpable splenomegaly is usual
Q399. The following concern the recognition of splenic enlargement :
A.
There is a left hypochondrial mass that moves down on respiration
B
The notch on the anteromedial border of the spleen is rarely felt
C
Percussion is unhelpful in recognizing the presence of splenomegaly
D.
A plain X-ray of the abdomen is useful
99Tcm colloid can be used to detect enlargement
E.
Q400. The following blood cell changes may be seen after splenectomy:
A.
Target cells
B.
Howell-Jolly bodies
C
Thrombocytopenia
D.
Transient neutrophilia
E.
Red cells containing hemosiderin
Q401. The following features relate to Hodgkin lymphoma:
A.
The commonest presentation is painless cervical lymphadenopathy
B
A chest X-ray is rarely helpful
C.
Patients with constitutional symptoms have a poorer prognosis
D.
Although Pruritus is not a B-symptom, prognostically it is as important
E.
A normocytic normochromic anemia is common
Q402. The following factors inhibit thrombus formation:
A.
Nitrous oxide
B.
Plasmin
C.
Antithrombin III
D
Stasis
E.
Decreased concentration of coagulation factors
53
Professor Anwar Sheikha, MD, FRCP, FRCPath., FACP
One Thousand Hematology MCQ
Q403. The following factors potentiate thrombus formation:
A
Prostacyclin (PGI2)
B.
Thrombocythemia
C.
Damaged vascular endothelium
D.
Depressed RE function
E.
Inhibitors of fibrinolysis
Q404. The following facts concern abnormal bleeding due to thrombocytopenia :
A.
Superficial bleeding into the skin is typical
B.
Bleeding from a superficial skin cut is often excessive
C.
Gastrointestinal hemorrhage can occur
D
Usually associated with joint bleeding
E.
A history of drug ingestion is important
Q405. The following diseases could cause of thrombocytopenic purpura:
A
Henoch-Schonlein purpura
B.
Aplastic anemia
C.
Systemic lupus Erythematosus (SLE)
D.
Disseminated Intravascular Coagulation (DIC)
E
Hemorrhagic Disease of the Newborn (HDN)
Q406. The following conditions predispose to thromboembolism:
A.
Antiphospholipid antibody syndrome
B.
Homocystinuria
C.
(HIT) Heparin-induced thrombocytopenia
D.
Factor XII deficiency
E.
Disseminated Intravascular Coagulation
Q407. The followings concern Hemophilia:
A.
The severity of bleeding is related to the factor VIII concentration
B
Chronic arthritis is uncommon
C.
Hematomas can compress peripheral nerves
D.
It occurs more commonly than Christmas disease
E.
If a hemophiliac marries a normal female, all their daughters will be
carriers
Q408. The followings concern von Willebrand disease :
A
It is inherited as an sex-linked disease
B
The bleeding time is usually normal
C.
The factor VIII activity measured by clotting assay is usually low
D
Factor VIII-related antigen is usually normal
E.
Platelet function test is abnormal
Q409. The followings concern the use of anticoagulants :
A.
During Warfarin therapy factors II, VII, IX and X are reduced
B
A single loading dose of 30mg Warfarin is the best way of initiating
therapy
C.
Barbiturates affect the action of Warfarin
D
The best way of using heparin is by intermittent parenteral dosage
E.
Subcutaneous heparin is helpful in prevention of postoperative
thrombosis
54
Professor Anwar Sheikha, MD, FRCP, FRCPath., FACP
One Thousand Hematology MCQ
Q410. The following concerns Disseminated Intravascular Coagulation (DIC) :
A
Thrombosis is rare
B.
Bleeding can be a major feature
C.
Fragmentation of the red cells may be observed on a blood film
D.
Thrombin time may be prolonged despite an adequate fibrinogen
E
Treatment with heparin is to prevent thrombosis is usually life-saving
Q411. The following are general statements about blood transfusion and blood groups:
A
Donor blood is usually collected into heparin which acts as an
anticoagulant
B.
Most of ABO-incompatible transfusions are due to failure to check identity
C.
Anti-A and Anti-B are often absent from the sera of group O babies
D
The following blood groups (ABO, Rh(D) and MN) are from a mother,
child and putative father :
mother:
A Rh(D) positive MN
child :
O Rh(D) negative NN
putative father :
A Rh(D) positive NN
The result exclude paternity
E.
Care should be exercised in transfusing chronic anemias to avoid
overload
Q412. Requirements for blood donors are :
A
The hemoglobin concentration is above 140 gm/l
B.
The donor has not had malaria in the recent past
C.
The donor has not been pregnant within the previous year
D
The donor has never been jaundiced
E.
The donor's blood is free from evidence of active syphilis
Q413. The followings are complications of rapid massive blood transfusion:
A.
Electrocardiographic abnormalities may develop
B
Metabolic acidosis is common
C.
Abnormal bleeding
D.
Hypothermia
E.
Altered hemoglobin function
Q414. The followings concern basic immunological reactions in blood group serology:
A.
Anti-A agglutinate group A red cells when they are suspended in saline
B.
Addition of albumin improves the sensitivity of some serological tests.
C.
Antiglobulin (Coomb's) test detects antibodies on the erythrocyte
membrane which fail to agglutinate the red cells when suspended in
saline
D.
The direct Coomb’s test detects in vivo sensitization of RBC with antibody
E.
The Antiglobulin test is used in the standard cross-match procedure
55
Professor Anwar Sheikha, MD, FRCP, FRCPath., FACP
One Thousand Hematology MCQ
Q415. The following statements relate to unwanted reactions of blood transfusion:
A.
Complement-mediated red cell lysis follows incompatible blood
transfusion
B.
Renal failure may follow the transfusion of ABO incompatible blood
C
Febrile transfusion reaction is usually due to Rhesus incompatibility
D.
Sudden extreme hypotension in the presence of warm extremities
could indicate that the transfused blood was contaminated with bacteria
E.
One cause of urticarial & other allergic reactions is anti-IgA in the
recipient
Q416. The following statements concern other unwanted effects of blood transfusion :
A.
A patient with aplastic anemia requiring 24 units of blood per annum will
accumulate 6 gm of iron during that year
B.
A rapid fall of hemoglobin together with jaundice 5 to 7 days following
transfusion suggests the development of delayed transfusion reaction
C
Post-transfusion hepatitis has been eliminated by routine HBsAg
screening
D.
Dyspnea accompanied by a dry cough can be an early sign
of pulmonary edema due to circulatory overload
E
Thrombophlebitis of the vein cannulated for transfusion is common
Q417. The followings are statements about blood and blood products:
A.
The only indication for whole blood transfusion might be massive bleeding
B
Packed RBC has inferior post-transfusion survival compared to whole
blood
C.
Cryoprecipitate contains fibrinogen
D
Blood product replacement therapy in patients with DIC is contraindicated
E.
Febrile neutropenic patients may benefit from the infusion of white cells
from a patient with chronic granulocytic leukemia (CGL)
Q418. The following are further statements about blood products:
A.
Albumin is useful in the treatment of nephrotic syndrome
B
Specific immunoglobulins prepared from donor plasma in the
convalescent phase of infectious disease can be used to confer active
immunity
C
The administration of plasma protein fraction (PPF) is accompanied
by the risk of transmitting serum hepatitis
D.
Coagulation deficiencies in liver disease can be treated with FFP
E.
Factor VIII concentrates has a half-life of nearly 12 hours in the
circulation
Q419. The following statements concern Hemolytic Disease of the Newborn (HDN) :
A
Rhesus HDN is common in Rh(D) positive babies born to primiparous
Rh(D) negative mothers
B.
Passively administered anti-D to Rh(D) negative mothers postpartum has
reduced the incidence of Rh HDN
C.
High concentration of unconjugated bilirubin can cause death
D
A jaundiced newborn child whose blood group is O and whose
mother is group A could have HDN due to ABO incompatibility
E
Exchange transfusion should always be performed in any newborn
with HDN if the cord hemoglobin concentration is below 15 g/dl
56
Professor Anwar Sheikha, MD, FRCP, FRCPath., FACP
One Thousand Hematology MCQ
Q420. The following disorders are inherited as autosomal dominant traits:
A.
Hereditary elliptocytosis
B.
Hereditary spherocytosis
C
Christmas disease (Hemophilia B)
D.
von Willebrand's disease
E
Glucose-6-Phosphate Dehydrogenase deficiency
Q421. Methemoglobinemia is a recognized complication of ingestion of or exposure to:
A.
Potassium chlorate
B.
Nitroglycerine
C.
Aniline dyes
D
Aspirin
E
Ascorbic acid
Q422. Henoch-Schonlein syndrome :
A.
May be complicated by intussusception
B.
May run a relapsing course of as long as a year
C.
Could be associated with nephropathy
D.
Typically follows infection with β- hemolytic streptococci
E
Is characterized by painful erythematous, 3 cm diameter, shin lesions
Q423. Thromboembolism is a possible complication of the following factor deficiency:
A
Factor VIII
B
Factor XI
C
Factor X
D.
Factor XII
E
Factor XIII
Q424. The following inherited problems can predispose to venous thromboembolism:
A.
Factor V Leiden
B.
Protein C deficiency
C.
Protein S deficiency
D.
Elevated factor VIII
E.
Antithrombin III deficiency
Q425. The following disorders are inherited as X-linked recessive traits:
A.
Wiskott-Aldrich syndrome
B.
Glucose-6-Phosphate Dehydrogenase deficiency
C
Acute intermittent porphyria
D.
Complete testicular feminization
E
Homocystinuria
Q426. Causes of macrocytosis in the blood with normoblastic erythropoiesis include :
A.
Hypothyroidism
B.
Pregnancy
C
Jejunal diverticulosis
D.
Hepatic cirrhosis
E.
Alcoholism
57
Professor Anwar Sheikha, MD, FRCP, FRCPath., FACP
One Thousand Hematology MCQ
Q427. Characteristic results of severe folate deficiency include :
A
Steatorrhoea
B
A leuko-erythroblastic anemia
C
Subacute combined degeneration of the spinal cord
D.
Hypersegmentation of the nuclei of the polymorphonuclear leukocytes
E
An increases in the total iron binding capacity of the blood
Q428. The bleeding time is characteristically prolonged in:
A
Hemophilia
B
Christmas disease
C.
von Willebrand's disease
D
Factor XII deficiency
E.
Immune thrombocytopenic purpura
Q429. Recognized complications of Hodgkin lymphoma include:
A.
Generalized pruritus
B.
Paraplegia
C.
Hemolytic anemia
D.
Cryptococcal meningitis
E.
Generalized varicella-zoster infection
Q430. Target cell is a recognized finding in blood films:
A.
In hemoglobin S disease
B.
In iron deficiency anemia
C.
In thalassemia
D.
In cirrhosis
E
In microangiopathic hemolytic anemia
Q431. Recognized causes of discrete osteolytic lesions in a two-year-old child include:
A
Dietary rickets
B.
Histiocytosis X
C
Infectious mononucleosis
D.
Acute lymphoblastic leukemia
E
Multiple myeloma
Q432. The following statements about Burkitt lymphoma are correct :
A.
A virus is etiologically important
B.
The northernmost part of African continent is relatively little affected
C
It is resistant to treatment with alkylating agents
D.
The head and neck are most often involved in Africa but viscera is a
common site in Saudi Arabia
E.
There are some climatic similarities in areas with the highest incidence
Q433. In patients with subclinical folate deficiency, megaloblastic anemia may be
precipitated by the administration of :
A.
Co-trimoxazole
B.
Methotrexate
C.
Phenytoin
D
Chloroform
E.
Metformin
58
Professor Anwar Sheikha, MD, FRCP, FRCPath., FACP
One Thousand Hematology MCQ
Q434. In the anticoagulant clinic, one should be careful with the effect of the following
on the enhancement of Warfarin effect on INR:
A.
Vitamin K
B.
Rifampicin
C.
Antiepileptics Carbamazepine & Phenobarbitone
D.
Indulgence in episodic eating of green leafy vegetables & citreous fruits
E.
Oral contraceptives
Q435. In the anticoagulant clinic, one should be careful with the effect of the following
on the enhancement of warfarin effect on INR:
A.
Large amounts of alcohol
B.
Ulcer-healing drugs Cimetidine & Omeprazole
C.
Thyroxine
D.
Ampicillin
E.
Antifungals fluconazole, itraconazole & ketoconazole
Q436. Low Molecular weight Heparins are favored on unfractionated heparin because:
A.
Higher bioavailability
B.
No need for monitoring the anticoagulant effect with clotting tests
C.
Much lower incidence of Heparin Induced Thrombocytopenia (HIT)
D.
No effect on function of the platelets
E
Much cheaper
Q437. The following clinical features indicate a more aggressive disease state in
patients with Hodgkin lymphoma:
A.
Fever of 390 C
B.
Drenching nocturnal sweats
C.
Weight loss of more than 10 Kg in a 50 Kg body weight patient
D.
Extensive Pruritus
E
Alcohol-induced lymph node pain
Q438. The following criteria is more common in Hodgkin lymphoma than in nonHodgkin lymphoma:
A
Bone marrow involvement
B
Extra-nodal disease
C.
Usually affects young adults, especially in the developed countries
D.
Unifocal in origin with a predictable contagious lymphatic spread
E
Most cases present with advanced stage
Q439. The following features relate to Immuno-proliferative Small Intestinal Disease:
A.
Most cases are reported from the Middle East
B.
Patients usually present with malabsorption
C.
Also known as Mediterranean lymphoma
D.
Alpha heavy chain might be detected in the serum
E.
Small intestinal biopsy reveals diffuse infiltration of the lamina propria with
plasma cells and lympho-plasmacytoid cells
59
Professor Anwar Sheikha, MD, FRCP, FRCPath., FACP
One Thousand Hematology MCQ
Q440. Acute Graft versus Host Disease (GvHD) basically affects the following organs:
A.
Skin
B.
Gut
C
Heart
D.
Liver
E
Brain
Q441. Allogeneic bone marrow transplantation could be done in the following diseases:
A.
Acute myeloid leukemia
B.
Chronic myeloid leukemia
C.
Aplastic anemia
D
Hemophilia
E.
Wiscott-Aldrich syndrome
Q442. Bone marrow fibrosis could be increased in the following diseases:
A.
Tuberculosis involving the marrow
B.
Lymphocyte depleted Hodgkin’s disease
C.
Myelofibrosis
D.
Acute megakaryoblastic leukemia
E
Thalassemia
Q443. Busulphan has been effective in the treatment of the following diseases:
A.
Pre-conditioning for bone marrow transplantation
B.
Chronic myeloid leukemia
C.
Myeloproliferative disorders
D
Burkitt’s leukemia
E
Burkitt’s lymphoma
Q444. Hydroxyurea is used with great success in the following blood diseases:
A.
Sickle cell disease
B
Thalassemia
C.
Chronic myeloid leukemia
D.
Polycythemia rubra vera
E
Secondary polycythemia
Q445. Prednisolone is an important part of the treatment in the following diseases:
A.
Immune thrombocytopenic purpura
B.
Chronic lymphocytic leukemia
C.
Autoimmune hemolytic anemia
D.
Lymphomas
E.
Acute lymphoblastic leukemia
Q446. Chlorambucil is effective in the management of the following blood diseases:
A.
Lymphomas
B.
Chronic lymphocytic leukemia
C
Acute myeloid leukemia
D.
Severe refractory cold hemagglutinin disease
E
Essential thrombocythemia
60
Professor Anwar Sheikha, MD, FRCP, FRCPath., FACP
One Thousand Hematology MCQ
Q447. Melphalan is the Gold standard in the management of the following diseases:
A
Burkitt lymphoma
B
M-GUS “Monoclonal Gammopathy of Undetermined significance”
C.
Multiple myeloma
D
IPSID “Immunoproliferative Small Intestinal Disease”
E
Acute leukemias
Q448. Aspirin should be considered in the prophylaxis of thrombotic tendency in:
A.
Polycythemia rubra vera
B.
Essential thrombocythemia
C.
Antiphospholipid antibody syndrome
D.
Ischemic heart disease
E.
Behcet’s syndrome
Q449. “Wait and watch” policy is warranted in managing the following malignancies:
A
Stage I Nodular sclerosis Hodgkin lymphoma
B.
Stage III low grade non-Hodgkin lymphoma
C
Stage I low grade non-Hodgkin lymphoma
D
Stage III high grade non-Hodgkin lymphoma
E
Stage I high grade non-Hodgkin lymphoma
Q450. The following relate to the half-life of different coagulation factors:
A.
Half-life of Factor VII is less than 5 hours
B.
Half-life of Factor V is around 15 hours
C.
Half-life of Factor VIII is around 10 hours
D.
Half-life of Factor IX is around 25 hours
E.
Half-life of Factor XI is around 45 hours
Select the most appropriate answer:
Q451. All, but one, of the followings relate to Homocystinuria :
A.
Predisposes to venous thromboembolism
B.
Could also predispose to arterial thrombosis
C.
The disease could manifest itself in childhood or even early in infancy
D.
Inborn error of metabolism due to deficiency of cystathionine synthase
E
Marfan- features, ectopia lentis and mental retardation are not seen
Q452. All, except one, of the following features relate to Factor V Leiden :
A.
It has been found to be strongly associated with venous thromboembolism
B.
Usually there is resistance to Active Protein C
C.
First described by Dahlback from Sweden
D.
>40% of Swedish anticoagulant clinic patients have this deficiency
E
The incidence is much lower than all of AT III, Protein C & S deficiency
Q453. All, but one, of the followings could contribute to venous thromboembolism:
A.
Immobility, surgery & myocardial infarction
B.
Pregnancy and puerperium
C
von Willebrand disease
D.
Obesity
E.
Previous attacks of deep vein thrombosis
61
Professor Anwar Sheikha, MD, FRCP, FRCPath., FACP
One Thousand Hematology MCQ
Q454. All, except one, of the followings could contribute to venous thromboembolism :
A.
PNH “Paroxysmal Nocturnal Hemoglobinuria”
B.
PRV “Polycythemia Rubra Vera”
C.
Behcet’s syndrome
D
Myeloma
E.
Contraceptive pills
Q455 All, but one, of the followings could contribute to thrombophilia :
A
Statin group of cholesterol lowering drugs
B.
Malignancy
C.
Homocystinuria
D.
Nephrotic syndrome
E.
Factor XII deficiency
Q456. Venous thromboembolism could result from any of the following genetic defects
EXCEPT :
A.
Protein C Deficiency
B.
Protein S Deficiency
C.
AT III Deficiency
D.
Factor XII Deficiency
E
Factor V Deficiency
Q457. End-stage chronic renal failure patients bleed because of:
A.
Burr red cells
B.
Inhibitors to coagulation factors
C.
Marked coagulation factor deficiency
D.
Marked thrombocytopenia
E
Platelet dysfunction
Q458. All, but one, of the following statements are true about Factor XIII Deficiency:
A
It is an autosomal dominant disease
B.
It is associated with poor wound healing
C.
Bleeding could be severe and protracted
D.
Weekly cryoprecipitate or factor XIII concentrate are effective treatments
E.
It is a rare disease
Q459. All, but one, of the following statements are true about Factor XI Deficiency:
A.
It is basically a disease of the Ashkenazi Jews
B
There is a strong association between Factor XI level & severity of
bleeding
C.
APTT is prolonged but PT is normal
D.
The half life of Factor XI is 60 hours
E.
Factor XI concentrate treatment could be associated with thrombosis
Q460 All, but one, of the following statements are true about Factor VII Deficiency:
A.
It is a rare autosomal recessive bleeding disorder
B.
Bleeding manifestations is like what is seen in hemophiliacs
C
Clotting screen shows prolonged PT and APTT
D.
Severe bleeding only follows absent or very low level of Factor VII
E.
Factor VII half-life is very short at 3 hours, so surgery needs frequent
doses
62
Professor Anwar Sheikha, MD, FRCP, FRCPath., FACP
One Thousand Hematology MCQ
Q461. All, but one, of the followings are true about True homozygous female
hemophilia A :
A.
It is exceedingly rare
B.
It usually results from marriage of a hemophiliac to a carrier female cousin
C
Menorrhagia is never a problem in these patients
D.
It must be differentiated from extreme unfavorable lyonization in a career
E.
It has been described in a female with Turner’s syndrome
Q462. One of the followings is important in the diagnosis of iron deficiency anemia :
A.
Coomb’s test
B
Serum ferritin
C.
Ring sideroblasts in the marrow
D.
Hemoglobin electrophoresis
E.
Empty bone marrow trephine biopsy
Q463. One of the followings is important in the diagnosis of - thalassemia major:
A.
Coomb’s test
B.
Serum ferritin
C.
Ring sideroblasts in the marrow
D
Hemoglobin electrophoresis
E.
Empty bone marrow trephine biopsy
Q464. One of the followings is important in the diagnosis of sideroblastic anemia:
A.
Coomb’s test
B.
Serum ferritin
C
Ring sideroblasts in the marrow
D.
Hemoglobin electrophoresis
E.
Empty bone marrow trephine biopsy
Q465. One of these is important in the diagnosis of autoimmune hemolytic anemia:
A
Coomb’s test
B.
Serum ferritin
C.
Ring sideroblasts in the marrow
D.
Hemoglobin electrophoresis
E.
Empty bone marrow trephine biopsy
Q466. One of the followings is important in the diagnosis of aplastic anemia:
A.
Coomb’s test
B.
Serum ferritin
C.
Ring sideroblasts in the marrow
D.
Hemoglobin electrophoresis
E
Empty bone marrow trephine biopsy
Q467. Fragmented red cells could be seen in all of the following diseases EXCEPT:
A.
Thrombotic Thrombocytopenic Purpura/ Hemolytic Uremic Syndrome
B.
Malignant Hypertension
C
Hereditary spherocytosis
D.
MAHA “Micro-Angiopathic Hemolytic Anemia”
E.
March Hemoglobinuria
63
Professor Anwar Sheikha, MD, FRCP, FRCPath., FACP
One Thousand Hematology MCQ
Q468. Sickle cells could be seen in the blood smear of one of the following diseases:
A.
Thalassemia
B
Sickle cell disease
C.
Sickle cell trait
D.
Anemia of chronic disorders
E.
Megaloblastic anemias
Q469. Spherocytes are not seen in which one of the following blood diseases:
A.
Hereditary spherocytosis
B.
Warm autoimmune hemolytic anemia
C.
Evans’ syndrome
D.
Incompatible blood transfusion
E
Megaloblastic anemia
Q470. One of these is more specific to sickle cell disease than - thalassemia major:
A.
Massive splenomegaly
B.
Very high Hemoglobin F
C.
Hypochromic microcytic red cells
D
Splenic atrophy
E.
Need for regular blood transfusions
Q471. All, but one, of the following features is more in favor of - thalassemia major
than sickle cell disease:
A.
Massive splenomegaly
B.
Predominant Hemoglobin F
C.
Hypochromic microcytic red cells
D
Splenic atrophy
E.
Need for regular blood transfusions
Q472. One of the following hemoglobin electrophoretic patterns is NOT true:
A.
Very high Hb F is in favor of the diagnosis of - thalassemia major
B.
Hemoglobin C migrates in the same position as Hemoglobin A2
C.
Hemoglobin E migrates in the same position as Hemoglobin A2
D
Hemoglobin S in sickle cell trait is usually around 60%
E.
Hb A2 of ~ 5% is in favor of the diagnosis of - thalassemia minor
Q473. All, but one, of the following disorders can cause Anemia of Chronic Disorders:
A.
Pulmonary tuberculosis
B.
Disseminated malignancy
C.
End stage renal disease
D
Epilepsy patients on Epanutin
E.
Connective tissue diseases
Q474. All, but one, of the followings are true about - thalassemias:
A.
Loss of four -chains is not compatible with life
B.
Loss of three -chains cause Hemoglobin H disease
C.
Loss of two -chains cause - thalassemia minor
D.
Loss of one -chain might not cause any clinical problem
E
-thalassemia gene in a population gives protection against severity of thalassemia
64
Professor Anwar Sheikha, MD, FRCP, FRCPath., FACP
One Thousand Hematology MCQ
Q475. Working formulation for clinical usage WAS a classification of:
A.
Hodgkin Disease
B.
Chronic lymphocytic leukemia
C.
Myelodysplastic syndromes
D
Malignant Non-Hodgkin lymphoma
E.
Acute non-lymphocytic leukemia
Q476. Rai and Binet systems are used for staging:
A.
Hodgkin lymphoma
B
Chronic lymphocytic leukemia
C.
Myelodysplastic syndromes
D.
Non-Hodgkin lymphoma
E.
Acute myeloid leukemia
Q477. Ann Arbor system is basically used for staging:
A
Hodgkin lymphoma
B.
Chronic lymphocytic leukemia
C.
Myelodysplastic syndromes
D.
Non-Hodgkin malignant lymphoma
E.
Acute myeloid leukemia
Q478. The new W.H.O. classification of hematological malignancies embraces all
but one of the following disease groups:
A.
Hodgkin Lymphoma
B.
Chronic lymphocytic leukemia/ small lymphocytic lymphoma
C.
Myelodysplastic syndromes
D.
Non-Hodgkin malignant lymphoma
E
Osteopetrosis
Q479. All, but one, of the followings could cause secondary polycythemia:
A.
Cyanotic congenital heart disease
B.
Smoking
C
Alcoholism
D.
Chronic Obstructive Airway Disease (COAD)
E.
High altitude
Q480. One of these conditions is least likely to predispose to acute leukemia:
A.
Down syndrome
B.
Ataxia telangiectasia
C.
Myelodysplastic syndromes
D.
Aplastic anemia
E
Pure red cell aplasia
Q481. Reticulocyte count of 3% is usual in one of the following anemias:
A
pernicious anemia
B.
Aplastic anemia
C.
Pure red cell aplasia
D.
Fanconi’s anemia
E.
Diamond Blackfan syndrome
65
Professor Anwar Sheikha, MD, FRCP, FRCPath., FACP
One Thousand Hematology MCQ
Q482. Gaucher’s disease is due to :
A
β- glucocerebrosidase deficiency
B.
Sphingomyelinase deficiency
C.
Hexosaminidase A deficiency
D.
Cystathione synthase deficiency
E.
Glucose-6-Phosphate Dehydrogenase deficiency
Q483. Niemann-Pick disease is due to :
A.
β- glucocerebrosidase deficiency
B
Sphingomyelinase deficiency
C.
Hexosaminidase A deficiency
D.
Cystathione synthase deficiency
E.
Glucose-6-Phosphate Dehydrogenase deficiency
Q484. Tay-Sachs disease is due to :
A.
β- glucocerebrosidase deficiency
B.
Sphingomyelinase deficiency
C
Hexosaminidase A deficiency
D.
Cystathione synthase deficiency
E.
Glucose-6-Phosphate Dehydrogenase deficiency
Q485. Favism is due to :
A.
β- glucocerebrosidase deficiency
B.
Sphingomyelinase deficiency
C.
Hexosaminidase A deficiency
D.
Cystathione synthase deficiency
E
Glucose-6-Phosphate Dehydrogenase deficiency
Q486. Homocystinuria could be due to :
A.
β- glucocerebrosidase deficiency
B.
Sphingomyelinase deficiency
C.
Hexosaminidase A deficiency
D
Cystathione synthase deficiency
E.
Glucose-6-Phosphate Dehydrogenase deficiency
Q487. All, but one, of the following features are true about Pelger-Hüet anomaly:
A.
It is autosomal dominant trait
B.
Neutrophil function is intact & it does not have adverse effect on health
C.
The acquired form could be seen in myelodysplastic syndromes
D
Neutrophils are usually multi-segmented with toxic granulation
E.
Usually indicates poorer prognosis if seen in myelodysplastic syndromes
Q488. Malarial anemia could be from any, but one, of the following causes:
A.
Hypersplenism especially in tropical splenomegaly syndrome
B.
Bone marrow suppression
C.
Extravascular destruction of the parasitized red cells
D.
Blackwater fever from intravascular destruction of parasitized red cell
E
Dysplasia of the blood cell precursors in the bone marrow
66
Professor Anwar Sheikha, MD, FRCP, FRCPath., FACP
One Thousand Hematology MCQ
Q489. All, but one, of the following cancers can produce erythropoietin and cause
secondary polycythemia:
A.
Hypernephroma
B.
Hepatoma
C
Leukemia
D.
Bronchial carcinoma
E.
Pheochromocytoma
Q490. Basophilic stippling is a characteristic feature of one of the following red cell
enzymopathies:
A.
Glucose 6 Phosphate Dehydrogenase deficiency
B.
Pyruvate Kinase deficiency
C.
Phosphofructokinase deficiency
D.
Triose Phosphate isomerase deficiency
E
Pyrimidine 5 nucleotidase deficiency
Q491. Type II CDA “Congenital Dyserythropoietic Anemia” has all, but one, of the
following features:
A
It is the least common of the three types of CDA
B.
It is transmitted as an autosomal recessive disease
C.
Red cells could be hemolysed by acidified serum, hence called HEMPAS
D.
Smear shows aniso-poikilocytosis, and marrow erythroid multinuclearity
E.
Patients have anemia, mild jaundice and evidence of iron overload
Q492. A child from Tihama, villages of the southern Saudi Arabia, admitted with a high
fever and organomegaly, could need all of the following investigations EXCEPT:
A.
Bone marrow examination to rule out Burkitt lymphoma
B
PET scan of the abdomen for hot areas of non-Hodgkin lymphoma lesions
C.
Splenic puncture to rule out visceral Leishmaniasis
D.
Immunophenotyping of blast cells “if present” for CD41 and CD61
E.
Enzyme studies to rule out storage diseases
Q493. Osmotic fragility test is helpful in identifying one of the following anemias:
A.
Glucose-6-Phosphate Dehydrogenase deficiency
B.
Sickle cell disease
C
Hereditary spherocytosis
D.
Aplastic anemia
E.
Diamond Blackfan syndrome
Q494 Patients with CHAD “Cold Hemagglutinin Disease” might have all, but one,
of the following features:.
A.
Acrocyanosis in cold weather due to red cell agglutination
B.
It is usually a benign chronic elderly disease, though could be distressing
C.
The hematology analyzer could record a very high red cell MCV
D
It is never associated with lymphoma
E.
It could follow infection with Mycoplasma pneumoniae
67
Professor Anwar Sheikha, MD, FRCP, FRCPath., FACP
One Thousand Hematology MCQ
Q495. All, but one, of the following features should raise the possibility of PNH
“Paroxysmal Nocturnal Hemoglobinuria”:
A.
Intravascular hemolysis with negative direct Coomb’s test
B.
Hemolysis with low platelet and/or white blood cell count
C.
Hemoglobinuria, Budd-Chiari syndrome or recurrent abdominal pain
D.
Pancytopenia with marrow failure
E
Splenomegaly and hypersplenism
Q496. Match the following hematological data & clinical features with the corresponding
blood disease:
Biodata:
Age: 44 years
Sex: Male
Presentation:
Two years history of anemia not responding to treatment
CBC:
Hb: 8.0 gm/dL
WBC: 8,000/uL
Platelet: 200,000/uL
Differential:
Normal
Blood Smear:
Mildly macrocytic anemia
Marrow aspiration: Active trilineage hemopoiesis
Others:
No evidence of megaloblastic anemia
A.
Acute lymphoblastic leukemia
B.
Fanconi’s anemia
C.
Refractory anemia with excessive blasts
D.
Chronic myelomonocytic leukemia
E
Refractory anemia
Q497. Match the following hematological data & clinical features with the corresponding
blood disease:
Biodata:
Age: 80 years
Sex: Male
Presentation:
Left hypochondrial fullness
CBC:
Hb: 10.0 gm/dL WBC: 80,000/uL
Platelet: 200,000/uL
Differential:
Neutrophils:60% Monocytes:10% Myelocytes:10% Blasts:4%
Blood Smear:
Neutrophilia and monocytosis
Marrow aspiration: Hypercellular with some dysplastic features
On Examination:
Marked splenomegaly
A.
Acute lymphoblastic leukemia
B.
Fanconi’s anemia
C.
Refractory anemia with excessive blasts
D
Chronic myelomonocytic leukemia
E.
Refractory anemia
68
Professor Anwar Sheikha, MD, FRCP, FRCPath., FACP
One Thousand Hematology MCQ
Q498. Match the following hematological data & clinical features with the corresponding
blood disease:
Biodata:
Presentation:
CBC:
Differential:
Blood Smear:
Marrow aspiration:
Others:
A.
B
C.
D.
E.
Age: 8 years
Sex: Male
Anemia, infections & bruising
Hb: 8.0 gm/dL
WBC: 1,000/uL
Platelet: 25,000/uL
Neutrophils = 80%
Oligopenic
Nearly empty
Horse-shoe shaped kidney; extra thumb; café-au-lait spots.
Acute lymphoblastic leukemia
Fanconi’s anemia
Refractory anemia with excessive blasts
Chronic myelomonocytic leukemia
Refractory anemia
Q499. Match the following hematological data & clinical features with the corresponding
blood diseases:
Biodata:
Presentation:
CBC:
Differential:
Blood Smear:
Marrow aspiration:
Others:
A.
B.
C
D.
E.
Age: 76 years
Sex: Male
Anemia, infection and bleeding
Hb: 6.0 gm/dL
WBC: 2,000/uL
Platelet: 60,000/uL
Blasts = 12%
Neutrophils = 44%
Hypogranular neutrophils; macrocytosis
Trilineage dysplasia; marrow blasts = 18%
Patient died six months later from an intracranial bleed.
Blasts were still 12%
Acute lymphoblastic leukemia
Fanconi’s anemia
Refractory anemia with excessive blasts
Chronic myelomonocytic leukemia
Refractory anemia
Q500. Match the following hematological data & clinical features with the corresponding
blood diseases:
Biodata:
Presentation:
CBC:
Differential:
Blood Smear:
Marrow aspiration:
Others:
A
B.
C.
D.
E.
Age: 6 years
Sex: Male
Anemia, infection and bleeding
Hb: 6.0 gm/dL
WBC: 60,000 /uL Platelet: 60,000 /uL
Blasts = 44%
Neutrophils = 8 Others= 48%
Blast cells with high nuclear/cytoplasmic ratio
Solidly cellular with blast cells constituting majority of the cells
Blast cells were positive for CD10; TdT & HLA-DR
Pro-B acute lymphoblastic leukemia
Burkitt leukemia
Refractory anemia with excessive blasts
Chronic myelomonocytic leukemia
T- acute lymphoblastic leukemia
69
Professor Anwar Sheikha, MD, FRCP, FRCPath., FACP
One Thousand Hematology MCQ
ONE THOUSAND
HEMATOLOGY
MULTIPLE CHOICE QUESTIONS EXAMINATION BANK
Professor Anwar Sheikha
MD, FRCP, FRCPI, FRCPA, FCAP, FRCPath., FACP
“501 to 1000”
Select the most appropriate answer:
Q501. Presence of “Elliptocytes” is not a usual finding in one of the following blood
diseases:
A.
Megaloblastic anemia
B.
Myelofibrosis
C.
Hereditary elliptocytosis
D
Glucose-6-Phosphate Dehydrogenase deficiency
E.
Severe iron deficiency anemia
Q502. Spherocytosis is not a usual feature in one of the following hemolytic anemias:
A.
Warm auto-immune hemolytic anemias
B.
Hereditary spherocytosis
C
Glucose-6-Phosphate Dehydrogenase deficiency
D.
Hemolytic disease of the newborn
E.
Clostridial septicemia
Q503. Spherocytes could hardly be seen in one of the following conditions:
A.
Microangiopathic hemolytic anemia
B.
After transfusion of stored blood
C.
Severe sepsis with disseminated intravascular hemolysis
D
Aplastic anemia
E.
Blood film prepared from blood allowed to stand before making the smear
Q504. Anisochromasia (various populations of red blood cells with different hemoglobin
content) is not a feature of:
A.
Sideroblastic anemia
B.
Transfused iron deficient anemia patients
C.
Treated iron deficient anemia patients
D.
A previously normal patient after one month of bleeding
E
Acute post-hemorrhagic anemia
70
Professor Anwar Sheikha, MD, FRCP, FRCPath., FACP
One Thousand Hematology MCQ
Q505. In severe polycythemia rubra vera:
A
Marrow iron stain is almost always absent
B.
Philadelphia chromosome is almost always positive
C.
Hemoglobin is almost always high even after repeated venesections
D.
Splenomegaly is almost always present
E.
ESR “Erythrocyte Sedimentation Rate” is almost always high
Q506. In severe polycythemia rubra vera:
A.
Platelet count is usually above a million per uL
B.
White cell count is always in hundreds of thousands per uL
C
Red cell count is always high
D.
Patients always have bleeding and thrombosis at the same time
E.
All patients are symptomatic at presentation
Q507. Megakaryocyte hyperplasia with abnormal morphology is not a feature of:
A.
Essential thrombocythemia
B.
Acute megakaryoblastic leukemia (M7)
C.
Myelofibrosis
D.
5q- syndrome
E
Iron deficiency anemia
Q508. Marrow megakaryocyte hyperplasia is not a feature of:
A.
Immune thrombocytopenic purpura
B.
Hypersplenism causing thrombocytopenia
C.
Polycythemia rubra vera
D.
Chronic myeloid leukemia
E
Drug-induced non-immune thrombocytopenia
Q509. When treating patients with iron deficiency anemias:
A.
Iron should be given as a second-line measure because blood transfusion
is more effective and a quicker mode of therapy
B.
Iron should be given until the hemoglobin is corrected
C.
Iron should not be given beyond the correction of the anemia to avoid iron
overload
D
Iron should be continued beyond anemia correction to replenish the stores
E.
Parenteral iron is a quicker, better and safer mode of therapy
Q510. One of the following features is not true in thalassemia minor patients:
A.
They are not infrequently diagnosed as iron deficiency and given iron
B
High RDW is helpful in differentiating it from iron deficiency anemia
C.
Patients are usually asymptomatic from their disease
D.
Offspring could develop thalassemia major if they get married to a
similar patient
E.
High HbA2 is the best test to diagnose β- thalassemia minor
Q511. One of the followings is true about Red Cell Distribution Width (RDW):
A.
RDW is normal in iron deficiency anemia
B
RDW is usually normal in thalassemia minor
C.
Low MCV & high RDW favor the diagnosis of thalassemia minor
D.
The normal RDW should not drop below 14.5% in the Coulter hemogram
E.
RDW does not have any role in differentiating iron deficiency from
thalassemia
71
Professor Anwar Sheikha, MD, FRCP, FRCPath., FACP
One Thousand Hematology MCQ
Q512. One of the followings is not correct in relation to spleen size in sickle cell disease:
A
Spleen is atrophic at birth
B.
By 6 months of age 1/3rd of patients have splenomegaly
C.
By 12 months of age 2/3rd of patients have splenomegaly
D.
By 2 years of age 3/4th of patients have splenomegaly
E.
Most patients will develop splenic atrophy when they get older
Q513. All but one of the following primary measures are effective in sickle cell anemia:
A.
Hydroxyurea
B
Blood transfusion for all anemic patients
C.
Partial exchange transfusion for special circumstances like major surgery
D.
Analgesia and morphia for painful vaso-occlusive crises
E.
Strict control of infection
Q514. All but one of the following features relate to thalassemia minor patients:
A.
They could have a normal hemoglobin concentration!
B.
Differs from iron deficiency anemia by having a more normal RDW
C.
Usually asymptomatic clinically
D.
They should be advised to avoid marriage with similar carriers
E
High altitude could trigger pain, hemolysis and anemia
Q515. One of the following features is not true in relations to Heinz bodies:
A.
They are red cell membrane-bound denatured hemoglobin
B
They are nuclear remnants
C.
They could be seen in unstable hemoglobins
D.
Blister cells are thought to be from splenic removal of Heinz bodies
E.
They are best detected using Supravital rather than Romanowsky stains
Q516. One of the following features is true in relation to Heinz bodies:
A
It could be detected in Glucose-6-Phosphate Dehydrogenase deficiency
B.
It could be detected in Evans’ syndrome
C.
It is one of the features of megaloblastic anemias
D.
It could be seen in sideroblastic anemias
E.
It is a feature of Pyrimidine-5-Nucleotidase deficiency
Q517. Intravascular hemolysis is a well-known complication of any of the followings
EXCEPT:
A.
ABO incompatible blood transfusion
B.
PNH “Paroxysmal Nocturnal Hemoglobinuria”
C.
CHAD “Cold Hemagglutinin Disease”
D.
G-6-P DH “Glucose-6-Phosphate Dehydrogenase” deficiency
E
Systemic lupus erythematosus with autoimmune hemolytic anemia
Q518. Patients are plethoric with ruddy complexion and suffused conjunctiva in:
A.
Essential thrombocythemia
B.
Myelofibrosis
C
Polycythemia rubra vera
D.
Myelodysplastic syndromes
E.
Chronic myeloid leukemia
72
Professor Anwar Sheikha, MD, FRCP, FRCPath., FACP
One Thousand Hematology MCQ
Q519. All of followings relate to LAP “Leukocyte Alkaline Phosphatase” score” EXCEPT:
A.
Could range between “0” & “400”
B.
Normal score should be between “10” and a “100”
C
It is characteristically high in CML “Chronic Myeloid Leukemia”
D.
It is usually high in leukemoid reactions
E.
It is high in myelofibrosis
Q520. LAP “Leukocyte Alkaline Phosphatase” score” is best performed on:
A
Fresh finger-stick blood smears
B.
EDTA blood
C.
Heparinized blood
D.
Citrate blood
E.
Clotted blood
Q521. All but one of the followings are true about Anemia of Chronic Disorders “ACD”:
A.
The anemia is usually normocytic normochromic
B.
The anemia could be hypochromic
C
Megaloblastic marrow changes are common
D.
Erythropoietin is usually inadequate
E.
ESR is usually raised
Q522. Anemia of chronic disorders could be seen in all but one of the following
conditions:
A
Chronic atrial fibrillation
B.
Tuberculosis
C.
Malignancy
D.
Uremia
E.
Cirrhosis
Q523. All of the followings are true about iron in Anemia of Chronic Disorders EXCEPT:
A.
Serum iron is usually low
B
Serum ferritin is usually low
C.
TIBC “Total Iron Binding Capacity” is usually low
D.
Marrow iron stores are increased or normal
E.
No siderotic granules are seen in the developing marrow erythroblasts
Q524. The cause of anemia & particularly hypochromia in anemia of chronic disorders
is:
A
Reticulo-endothelial block from high hepcidin level preventing iron delivery
from stores to the erythron
B.
Iron deficiency
C.
Ring sideroblasts with mitochondrial iron imprisonment
D.
Poor globin chain synthesis
E.
Pyridoxine deficiency
Q525. All of the followings are management options in aplastic anemia EXCEPT:
A
Hematinics (iron, B12, Folate, etc)
B.
Blood transfusions
C.
Bone marrow transplantation
D.
Anti-lymphocyte globulin
E.
Cyclosporine & Anti-lymphocyte globulin
73
Professor Anwar Sheikha, MD, FRCP, FRCPath., FACP
One Thousand Hematology MCQ
Q526. The least used therapy for the management of Anemia of Chronic Disorders is:
A.
Treatment of the underlying condition
B.
Erythropoietin
C
Immediate prescription of iron
D.
Blood transfusion
E.
Management & correction of associated causes of the anemia if present
Q527. Spherocytosis could be seen in the following diseases:
A.
Evans’ syndrome
B.
Hereditary spherocytosis
C.
Severe burn
D
All of the above
E.
None of the above
Q528. All of the followings are true in pure red cell aplasia EXCEPT:
A
The bone marrow is empty (i.e. hypocellular or acellular)
B.
The congenital form the disease is called Diamond-Blackfan syndrome
C.
It is a well-known complication of hemolytic anemias
D.
Parvovirus B19 is the usual culprit when it occurs in sickle cell disease
E.
Thymoma should always be ruled out
Q529. The followings are mandatory initial investigations in evaluation of hemolytic
anemias:
A.
Complete Blood Count “CBC”
B.
Reticulocyte count
C
Bone marrow examination
D.
Serum bilirubin
E.
Serum haptoglobin
Q530. In Wilson’s disease, the hemolytic anemia is caused by:
A
Oxidant damage to the red cells caused by excess Copper
B.
Autoantibody against the red cells
C.
Complement damage to the red cells because of membrane sensitivity
D.
Immune-complex damage to the red cells
E.
Liver sequestration of the red cells
Q531. Increased marrow iron stores without siderotic granules in erythroblasts is a
feature of:
A.
Sideroblastic anemia
B.
Thalassemia major
C
Anemia of Chronic Disorders
D.
Hemochromatosis
E.
Multiply transfused aplastic anemia
Q532. Increased marrow iron stores with ring sideroblasts in more than 15% of
developing erythroblasts is a feature of:
A
Sideroblastic anemia (Refractory Anemia with Excessive Ring
Sideroblasts)
B.
Thalassemia
C.
Anemia of Chronic Disorders
D.
Hemochromatosis
74
Professor Anwar Sheikha, MD, FRCP, FRCPath., FACP
E.
One Thousand Hematology MCQ
Iron deficiency anemia
Q533. Ham’s Test (Acidified Serum Test) could be positive in:
A
Congenital Dyserythropoietic Anemia Type II (HEMPAS)
B.
Hereditary spherocytosis
C.
Hereditary elliptocytosis
D.
Sickle cell disease
E.
Thalassemia
Q534. Ham’s Test (Acidified Serum Test) is positive in:
A.
Autoimmune hemolytic anemia
B.
Immune thrombocytopenic purpura
C.
Evans’ syndrome
D.
Systemic lupus erythematosus
E
Paroxysmal nocturnal hemoglobinuria
Q535. Basophilic stippling (punctate basophilia) is seen in:
A.
Pyrimidine 5 nucleotidase deficiency
B.
Lead poisoning
C.
Congenital Dyserythropoietic Anemia (CDA)
D
All of the above
E.
None of the above
Q536. All but one of the following diseases are X-linked:
A.
Hemophilia A
B.
Christmas disease
C.
Glucose-6-Phosphate Dehydrogenase deficiency
D.
Wiskott-Aldrich syndrome
E
Hereditary spherocytosis
Q537. Red cell fragmentation is seen in all but one of the following conditions:
A.
TTP
“Thrombotic Thrombocytopenic Purpura”
B.
HUS
“Hemolytic Uremic Syndrome”
C.
MAHA “Micro-Angiopathic Hemolytic Anemia”
D
AA
“Aplastic Anemia”
E.
Gram negative septicemia
Q538. Red cell fragmentation is seen in all but one of the following conditions
A.
March hemoglobinuria
B.
Artificial heart valves
C.
Malignant hypertension
D.
Mucin-secreting adenocarcinoma
E
Hereditary spherocytosis
Q539. All of the followings apply to iron status in hemolytic anemias EXCEPT:
A.
Iron storage is usually increased in chronic extravascular hemolytic
anemias
B.
Iron storage could be reduced in chronic intravascular hemolytic anemias
C.
Iron absorption is usually increased
D.
Ferritin is usually high
E
Marrow usually contains lots of ring sideroblasts
75
Professor Anwar Sheikha, MD, FRCP, FRCPath., FACP
One Thousand Hematology MCQ
Q540. All of the following features might be helpful in differentiating
hemoglobinuria of intravascular hemolytic anemias from hematuria EXCEPT:
A.
Red plasma
B
Red urine
C.
Presence or lack of red cells in centrifuged urine
D.
High serum LDH “Lactate Dehydrogenase”
E.
Low serum haptoglobin
Q541. All of the following clinical features should raise the possibility of PNH
“Paroxysmal Nocturnal hemoglobinuria” EXCEPT:
A.
Intravascular hemolysis
B.
Hemolysis with low platelet or white cell counts
C.
Hemoglobinuria
D.
Recurrent abdominal pain
E
Huge splenomegaly
Q542. PNH “Paroxysmal Nocturnal hemoglobinuria” should be suspected in all of
the following conditions EXCEPT:
A
Dapsone- induced intravascular hemolytic anemia
B.
Budd-Chiari syndrome
C.
Arterial thrombosis anywhere in the body in young patients
D.
Coomb’s negative intravascular hemolysis
E.
Aplastic anemia
Q543. “Prickle” red cells are frequently seen in one of the following hemolytic anemias:
A.
Sickle cell anemia
B.
Glucose-6-Phosohate Dehydrogenase deficiency hemolytic anemia
C
Pyruvate Kinase deficiency hemolytic anemia
D.
Hereditary spherocytosis
E.
Hereditary elliptocytosis
Q544. Bone marrow or peripheral stem cell transplantation is a main mode of
management in all but one of the following hematological diseases:
A
Chronic Lymphocytic Leukemia
B.
Chronic Myeloid Leukemia
C.
Aplastic Anemia
D.
Thalassemia
E.
Relapsing Acute Myeloid Leukemia
Q545. The following hematological conditions could give protection against malaria:
A.
Sickle cell trait
B.
Glucose 6 Phosphate Dehydrogenase deficiency
C.
Hereditary elliptocytosis
D
All of the above
E.
None of the above
Q546. All of the following diseases are very rare or mild in female patients EXCEPT:
A.
Hemophilia A
B.
Christmas disease
C.
Glucose-6-Phosphate Dehydrogenase deficiency
D.
Wiskott-Aldrich syndrome
76
Professor Anwar Sheikha, MD, FRCP, FRCPath., FACP
E
One Thousand Hematology MCQ
Hereditary spherocytosis
Q547. All but one of the following features are possible in PNH
“Paroxysmal Nocturnal Hemoglobinuria”:
A.
Normocytic anemia
B.
Macrocytic anemia
C.
Hypochromic anemia
D
All of the above
E.
None of the above
Q548. All but one of the following features are possible in PNH
“Paroxysmal Nocturnal hemoglobinuria”:
A.
Reticulocytosis
B.
Reticulocytopenia
C.
Thrombocytopenia
D
Increased marrow fibrosis
E.
Leucopenia
Q549. All but one of the following features are possible about iron status in PNH
“Paroxysmal Nocturnal hemoglobinuria”:
A
Increased iron stores in the bone marrow
B.
Low serum iron
C.
High TIBC “Total Iron Binding Capacity”
D.
Low serum Ferritin
E.
Iron deficiency anemia
Q550. All but one of the following features are possible in PNH
“Paroxysmal Nocturnal hemoglobinuria”:
A.
Active bone marrow
B.
Aplastic bone marrow
C.
Positive acidified serum test “Ham’s test”
D.
Positive sugar water test
E
Positive osmotic fragility test
Q551. All but one of the following features are possible during the hemolytic episodes
in PNH “Paroxysmal Nocturnal hemoglobinuria” :
A
High Haptoglobin
B.
High LDH
C.
Hemoglobinuria
D.
Hemoglobinemia
E.
Hemosiderinuria
Q552. One of the following features does not relate to Wilson’s Syndrome:
A
It usually complicate alcoholism
B.
Serum Copper level is high
C.
There is reduced serum Caeruloplasmin
D.
Liver functions tests are usually abnormal
E.
There is toxic accumulation of copper in liver, red cells, cornea, kidney
and brain
77
Professor Anwar Sheikha, MD, FRCP, FRCPath., FACP
One Thousand Hematology MCQ
Q553. One of the following features is not true in Wilson’s Syndrome:
A.
It is an autosomal recessive disorder of copper metabolism
B.
It is also known as hepato-lenticular degeneration
C.
It could well be associated with hemolytic anemia
D.
Ophthalmoscopy shows Keyser-Fleischer rings
E
Coomb’s test is positive.
Q554. All but one of the following associations are true about Wilson’s syndrome:
A.
Copper damage to liver could cause cirrhosis
B
Copper damage to red cells could cause Coomb’s positive hemolytic
anemia
C.
Copper damage to kidney could cause Fanconi’s renal tubular acidosis
D.
Copper damage to brain could cause progressive neurological damage
E.
Copper damage to cornea could cause Keyser-Fleischer rings
Q555. In Wilson’s syndrome:
A
Hemolysis might antedate liver failure
B.
Ham’s test is usually positive
C.
Coomb’s test is usually positive
D.
Osmotic fragility is usually positive
E.
Sugar water test is usually positive
Q556. The following immunizations are recommended prior to splenectomy
for hemolytic anemias:
A.
Pneumovax
B.
Hib “Hemophilus influenza type b”
C.
Meningococcal vaccine
D
All of eth above
E.
None of the above
Q557. In the new W.H.O. classification of Myelodysplastic syndromes “MDS”,
All of the following are true about 5q- syndrome EXCEPT:
A.
Usually affects female patients
B.
Platelet count could be high.
C.
The anemia is usually macrocytic
D.
Dysmegakarypoiesis in the marrow
E
Prognosis is poor and survival is worse then other types of MDS
Q558. All, but one of the following are true about ALIP
“Abnormal Localization of Immature precursors”:
A.
It is an important feature of Myelodysplastic syndromes
B.
It is more frequent in RAEB “Refractory Anemia with Excess Blasts”
C
Pathognomonic for Myelodysplastic syndromes
D.
Usually inter-trabecular rather para-trabecular
E.
It is a marrow trephine biopsy feature
Q559. All, but one, of the following support the diagnosis of Myelodysplastic syndromes:
A
Childhood onset
B.
Dyserythropoiesis
C.
Peripheral cytopenia
D.
ALIP “Abnormal Localization of Immature precursors”
78
Professor Anwar Sheikha, MD, FRCP, FRCPath., FACP
One Thousand Hematology MCQ
E.
Hypercellular marrow
Q560. One of the following features is true in Myelodysplastic syndromes:
A.
Universal progression to acute leukemia
B.
Marked increase of CD34 positive cells
C
Flow Cytometry usually shows myeloid markers CD13/ CD15 & CD33
D.
Ring sideroblasts are only detected in Refractory anemia with Ring
Sideroblasts
E.
Demonstration of Auer Rods has no prognostic value
Q561. One of the following diseases is autosomal dominant:
A.
Hemophilia A
B.
Christmas disease
C.
Glucose-6-Phosphate Dehydrogenase deficiency
D.
Wiskott-Aldrich syndrome
E
Hereditary spherocytosis
Q562. MCHC “Mean Cell Hemoglobin Concentration” is increased in:
A.
Thalassemia
B.
Aplastic anemia
C.
Sideroblastic anemia
D.
Megaloblastic anemia
E
Hereditary spherocytosis
Q563. Hemoglobinemia could be seen in all but one of the following conditions:
A.
Runners
B.
Joggers
C.
Violent exercisers
D.
Marching soldiers
E
Car racers
Q564. Hemoglobinuria is a well-recognized feature of one of the following conditions:
A
Karate players
B.
Air pilots
C.
Hemolytic disease of the newborn
D.
Hereditary spherocytosis
E.
Thalassemia
Q565. Hemoglobinemia is a well known complication of the following blood diseases
Except:
A.
PNH “Paroxysmal Nocturnal Hemoglobinuria”
B.
PCH “Paroxysmal Cold Hemoglobinuria”
C.
CHAD “Cold Hemagglutinin Disease”
D
Evans’ syndrome
E.
Blackwater fever from Falciparum malaria
Q566. Hemoglobinemia is a possible complication of any of the following scenarios
Except:
A.
Incompatible blood transfusion of Group A to O
B.
Incompatible blood transfusion of Group B to O
C.
Incompatible blood transfusion of Group AB to O
D
Incompatible blood transfusion of Group O to AB
E.
Incompatible blood transfusion of Group Bombay AB
79
Professor Anwar Sheikha, MD, FRCP, FRCPath., FACP
One Thousand Hematology MCQ
Q567. Normal serum haptoglobin level is around:
A
~ 2 gm/L
B.
~ 2 mg/L
C.
~ 2 gm/dL
D.
~ 2 mg/dL
E.
~ Undetectable
Q568. Normal serum hemopexin level is:
A
0.5 – 1.0 gm/L
B.
0.5 – 1.0 mg/L
C.
0.5 – 1.0 gm/dL
D.
0.5 – 1.0 mg/dL
E.
Undetectable
Q569. Haptoglobin could be reduced in all but one of the following conditions:
A.
PNH
B
Hereditary spherocytosis
C.
Hepatic failure
D.
Congenital ahaptoglobulinemia
E.
Extensive tissue hemorrhage
Q570. One of the following is true in the management of a thrombotic event in a patient
with APA “Antiphospholipid Antibody Syndrome”:
A.
Warfarin anticoagulation should aim at an INR of less than 2
B
Warfarin anticoagulation should aim at an INR of more than 2
C.
Only three months of treatment is recommended
D.
INR is always a reliable test and is not affected by the lupus anticoagulant
E.
Concomitant Aspirin is not recommended
Q571. The least likely cause of a combined venous and arterial thromboembolism is:
A.
PNH “Paroxysmal Nocturnal Hemoglobinuria”
B.
PRV “Polycythemia Rubra Vera”
C.
APA “Antiphospholipid Antibody” Syndrome
D.
HIT “heparin Induced thrombocytopenia”
E
Factor V Leiden
Q572. The most likely cause of a combined venous and arterial thromboembolism is:
A.
Protein C deficiency
B.
Protein S deficiency
C.
Antithrombin deficiency
D.
Prothrombin gene mutation
E
APA “Antiphospholipid Antibody Syndrome”
Q573. Pure venous thromboembolism is a well-known complication of:
A.
Hypertension
B.
Diabetes mellitus
C.
Smoking
D.
Atherosclerosis
E
Puerperium
80
Professor Anwar Sheikha, MD, FRCP, FRCPath., FACP
One Thousand Hematology MCQ
Q574. Arterial thromboembolism is a well-known complication of:
A
Homocysteinemia
B.
Immobilization
C.
Congestive heart failure
D.
Cancer “Trousseau’s syndrome”
E.
Surgery
Q575. A preferred strategy for supporting pregnancies in APA
“Antiphospholipid Antibody Syndrome”- associated fetal loss and abortion is:
A.
Intravenous heparin with an APTT ratio of 2.5
B
Subcutaneous heparin prophylaxis plus low dose Aspirin
C.
Aspirin alone
D.
Hirudin
E.
No intervention at all
Q576. All, but one, of the following coagulation tests could be used in detection of
lupus anticoagulant in APA “Antiphospholipid Antibody” Syndrome:
A.
APTT
“Active Partial Thromboplastin Time”
B.
KCCT
“Kaolin Cephalin Clotting Time”
C.
DRVVT
“Dilute Russell’s Viper-Venom Test”
D.
Tapian venom test
E
Active protein C resistance test
Q577. All of the following neutrophil changes could be seen in severe sepsis EXCEPT:
A.
Toxic granulation
B.
Left shifting
C.
Cytoplasmic vacuolation
D
All of the above
E.
None of the above
Q578. The following neutrophil changes could be seen in severe sepsis:
A.
Neutrophilia
B.
High Neutrophil Alkaline Phosphatase score
C.
Dohle bodies
D
All of the above
E.
None of the above
Q579. The following neutrophil changes could be seen in severe sepsis:
A.
Right-shifting
B.
Low Neutrophil Alkaline Phosphatase score
C.
Pseudo-Pelger neutrophils
D.
All of the above
E
None of the above
Q580. Howell-Jolly bodies could be seen in all but one of the followings:
A.
Megaloblastic anemia
B.
Splenic atrophy
C.
Celiac disease
D
Hypersplenism
E.
Splenectomy
81
Professor Anwar Sheikha, MD, FRCP, FRCPath., FACP
One Thousand Hematology MCQ
Q581. The following morphological abnormalities could be seen in myelodysplastic
syndromes:
A.
Megaloblastoid erythropoiesis
B.
Myeloid & erythroid nucleocytoplasmic asynchrony
C.
Dysmorphic megakaryocytes
D
All of the above
E.
None of the above
Q582. One of the following von Willebrand Disease types is autosomal recessive:
A.
Type 1
B.
Type 2A
C.
Type 2B
D
Type 2M
E
Type 3
Q583. To diagnose refractory anemia in the W.H.O. classification of
myelodysplastic syndromes, the anemia should be present for at least:
A.
1 month
B.
3 months
C
6 months
D.
1 year
E.
5 years
Q584. The medial survival of refractory anemia & refractory anemia with ring
sideroblasts in the W.H.O. classification of myelodysplastic syndromes is:
A.
3 months
B.
< 1 year
C.
~ 5 years
D
~ 10 years
E.
Normal life span
Q585. The median survival of refractory anemia with excessive blasts in the W.H.O.
classification of myelodysplastic syndromes is:
A.
3 months
B
< 1 year
C.
~ 5 years
D.
~ 10 years
E.
Normal life span
Q586. TdT is extremely useful in differentiating:
A
Acute Lymphoblastic Leukemia from Acute Myeloid Leukemia
B.
Hodgkin’s Disease from Non-Hodgkin’s lymphomas
C.
Chronic Myeloid Leukemia from other myeloproliferative disorders
D.
Mantle cell lymphoma from other types of lymphomas
E.
Leukemias from neuroblastomas
Q587. Cyclin D1 positivity is extremely useful in differentiating:
A.
Acute Lymphoblastic Leukemia from Acute Myeloid Leukemia
B.
Hodgkin’s Disease from Non-Hodgkin’s lymphomas
C.
Chronic Myeloid Leukemia from other myeloproliferative disorders
D
Mantle cell lymphoma from other types of lymphomas
82
Professor Anwar Sheikha, MD, FRCP, FRCPath., FACP
E.
One Thousand Hematology MCQ
Leukemias from neuroblastomas
Q588. In immunophenotyping of Acute Lymphoblastic Leukemias “ALL”,
Burkitt-type ALL type is:
A.
Positive for TdT, CD3 & CD7
B
Negative for TdT but strongly positive for SmIg “Surface membrane
immunoglobulin
C.
Positive for TdT and cytoplasmic Ig μ
D.
Positive for TdT & HLA-DR but negative for CD10 or cytoplasmic Ig μ
E.
Positive for TdT, CD10 but negative for cytoplasmic Ig μ
Q589. The following morphological abnormalities could be seen in myelodysplastic
syndromes:
A.
Pseudo-Pelger neutrophils
B.
Basophilic stippling
C.
Agranular neutrophils
D
All of the above
E.
None of the above
Q590. Positive reaction with the pan-leukocyte marker CD45 is extremely useful in
differentiating:
A.
Acute Lymphoblastic Leukemia from Acute Myeloid Leukemia
B.
Hodgkin’s Disease from Non-Hodgkin’s lymphomas
C.
Chronic myeloid leukemia from other myeloproliferative disorders
D.
Mantle cell lymphoma from other types of lymphomas
E
Leukemias from neuroblastomas
Q591. Reed-Sternberg cells in the tissue sections is extremely useful in differentiating:
A.
Acute Lymphoblastic Leukemia from Acute Myeloid Leukemia
B
Hodgkin’s Disease from Non-Hodgkin’s lymphomas
C.
Chronic Myeloid Leukemia from other myeloproliferative disorders
D.
Mantle cell lymphoma from other types of lymphomas
E.
Leukemias from neuroblastomas
Q592. The cytogenetic abnormality t(9:22)/BCR-ABL fusion gene is extremely useful in
differentiating:
A.
Acute Lymphoblastic Leukemia from Acute Myeloid Leukemia
B.
Hodgkin’s Disease from Non-Hodgkin’s lymphomas
C
Chronic myeloid leukemia from other myeloproliferative disorders
D.
Mantle cell lymphoma from other types of lymphomas
E.
Leukemias from neuroblastomas
Q593. In immunophenotyping of Acute Lymphoblastic Leukemias “ALL”,
Pro-B ALL type is:
A.
Positive for TdT, CD3 & CD7
B.
Negative for TdT but strongly positive for SmIg “Surface membrane
immunoglobulin
C.
Positive for TdT and cytoplasmic Ig μ
D.
Positive for TdT & HLA-DR but negative for CD10 or cytoplasmic Ig μ
E
Positive for TdT, CD10 but negative for cytoplasmic Ig μ
83
Professor Anwar Sheikha, MD, FRCP, FRCPath., FACP
One Thousand Hematology MCQ
Q594. In immunophenotyping of Acute Lymphoblastic Leukemias “ALL”, T- ALL is:
A
Positive for TdT, CD3 & CD7
B.
Negative for TdT but strongly positive for SmIg “Surface membrane
immunoglobulin”
C.
Positive for TdT and cytoplasmic Ig μ
D.
Positive for TdT & HLA-DR but negative for CD10 or cytoplasmic Ig μ
E.
Positive for TdT, CD10 but negative for cytoplasmic Ig μ
Q595. In immunophenotyping of Acute Lymphoblastic Leukemias “ALL”,
Pre-B ALL type is:
A.
Positive for TdT, CD3 & CD7
B.
Negative for TdT but strongly positive for SmIg “Surface membrane
immunoglobulin”
C
Positive for TdT and cytoplasmic Ig μ
D.
Positive for TdT & HLA-DR but negative for CD10 or cytoplasmic Ig μ
E.
Positive for TdT, CD10 but negative for cytoplasmic Ig μ
Q596. In immunophenotyping of hematological malignancies, positive reaction with anti
MPO “Myeloperoxidase” is highly suggestive for:
A.
B-cell malignancy
B.
T-cell malignancy
C
Myeloid leukemias
D.
Monoblastic leukemia
E.
Neuroblastoma
Q597. In immunophenotyping of hematological malignancies, positive reaction with
CD14 & anti-Lysozyme antibodies is highly suggestive for:
A.
B-cell malignancy
B.
T-cell malignancy
C.
Myeloid leukemias
D
Monoblastic leukemia
E.
Neuroblastoma
Q598. In immunophenotyping of hematological malignancies,
negative reaction with CD45 suggests:
A.
B-cell malignancy
B.
T-cell malignancy
C.
Myeloid leukemias
D.
Monoblastic leukemia
E
Neuroblastoma
Q599. In immunophenotyping of hematological malignancies,
positive reaction with CD79a is highly specific for:
A
B-cell malignancy
B.
T-cell malignancy
C.
Myeloid leukemias
D.
Monoblastic leukemia
E.
Neuroblastoma
84
Professor Anwar Sheikha, MD, FRCP, FRCPath., FACP
One Thousand Hematology MCQ
Q600. In immunophenotyping of hematological malignancies,
positive reaction with CD3 is highly specific for:
A.
B-cell malignancy
B
T-cell malignancy
C.
Myeloid leukemias
D.
Monoblastic leukemia
E.
Neuroblastoma
Q601. One of the following inherited abnormalities is a risk factor for both venous and
arterial thrombosis rather than venous thrombophilia alone:
A.
Factor V Leiden mutation
B.
Prothrombin G20210A mutation
C
Hyperhomocystinemia
D.
Protein C deficiency
E.
Protein S deficiency
Q602. One of the following conditions is a risk factor for both venous and arterial
thrombosis rather than venous thrombophilia alone
A.
Malignancy
B.
Contraceptive pills
C
Antiphospholipid syndrome
D.
Antithrombin deficiency
E.
Increased Factor VIII activity
Q603. Coomb’s test is used for investigation of one of the following anemias:
A
AIHA
“autoimmune hemolytic anemia”
B.
PNH
“Paroxysmal Nocturnal Hemoglobinuria”
C.
HS
“Hereditary Spherocytosis”
D.
Hemoglobinopathies
“Thalassemias & Sickle Cell Anemias”
E.
RARS
“Refractory Anemia with Ring Sideroblasts”
Q604. Ham’s test is used for investigation of one of the following anemias:
A.
AIHA
“autoimmune hemolytic anemia”
B
PNH
“Paroxysmal Nocturnal Hemoglobinuria”
C.
HS
“Hereditary Spherocytosis”
D.
Hemoglobinopathies
“Thalassemias & Sickle Cell Anemias”
E.
RARS
“Refractory Anemia with Ring Sideroblasts”
Q605. Osmotic fragility test is used for investigation of one of the following
anemias:
A.
AIHA
“autoimmune hemolytic anemia”
B.
PNH
“Paroxysmal Nocturnal Hemoglobinuria”
C
HS
“Hereditary Spherocytosis”
D.
Hemoglobinopathies
“Thalassemias & Sickle Cell Anemias”
E.
RARS
“Refractory Anemia with Ring Sideroblasts”
Q606. Perls’s or Prussian-blue stain is used for investigation of :
A.
AIHA
“autoimmune hemolytic anemia”
B.
PNH
“Paroxysmal Nocturnal Hemoglobinuria”
C.
HS
“Hereditary Spherocytosis”
D.
Hemoglobinopathies
“Thalassemias & Sickle Cell Anemias”
85
Professor Anwar Sheikha, MD, FRCP, FRCPath., FACP
E
RARS
One Thousand Hematology MCQ
“Refractory Anemia with Ring Sideroblasts”
Q607. Bone marrow examination might be needed for investigation of
one of the following hemolytic anemias:
A.
AIHA
“autoimmune hemolytic anemia”
B.
PNH
“Paroxysmal Nocturnal Hemoglobinuria”
C.
HS
“Hereditary Spherocytosis”
D.
Hemoglobinopathies
“Thalassemias & Sickle Cell Anemias”
E
HEMPAS
“Hereditary Erythroid Multinuclearity with Positive
Acidified Serum Test” which is also known as:
Congenital Dyserythropoietic Anemia Type II
Q608. Hemoglobin electrophoresis is used for investigation of one of the followings:
A.
AIHA
“autoimmune hemolytic anemia”
B.
PNH
“Paroxysmal Nocturnal Hemoglobinuria”
C.
HS
“Hereditary Spherocytosis”
D
Hemoglobinopathies
“Thalassemias & Sickle Cell Anemias”
E.
RARS
“Refractory Anemia with Ring Sideroblasts”
Q609. When two persons with β- thalassemia minor get married,
the chance of having children with β- thalassemia major is:
A.
1 in 2
B
1 in 4
C.
1 in 8
D.
All the children
E.
None of the children
Q610. When a normal person gets married to a patient with β- thalassemia minor,
the chance of having children with β- thalassemia major is:
A.
1 in 2
B.
1 in 4
C.
1 in 8
D.
All the children
E
None of the children
Q611. When a normal person gets married to a patient with β- thalassemia minor,
the chance of having children with β- thalassemia minor is:
A
1 in 2
B.
1 in 4
C.
1 in 8
D.
All the children
E.
None of the children.
Q612. Iron overload in β- thalassemia major could cause any of the following EXCEPT:
A.
Cirrhosis when deposited in the liver
B
Wilson disease when deposited in liver, eyes and brain
C.
Diabetes mellitus when deposited in pancreas
D.
Hyperpigmentation when deposited in skin
E.
Growth retardation when deposited in the endocrine organs
86
Professor Anwar Sheikha, MD, FRCP, FRCPath., FACP
One Thousand Hematology MCQ
Q613. One of the following patterns in hemoglobin electrophoresis supports the
diagnosis of homozygous Sickle Cell Disease:
Answer Age
Hb MCV HbA HbF Hb Hb
HbS HbH HbC HbE
Barts
gm/dL
A2
A.
5 yr 8
80
6%
2%
92%
B
2 yr 6
60
98% 2%
C
18 yr 8
66
70%
2%
↑
20%
D
8 yr 8
64
12%
80%
E
12 yr 10
80
4%
44%
40%
Q 614. One of the following patterns in hemoglobin electrophoresis supports the
diagnosis of Sickle Cell Trait:
Answer Age
Hb MCV HbA HbF Hb Hb
HbS HbH HbC HbE
Barts
gm/dL
A2
A.
24 yr 12
82
56%
2%
42%
B
18 yr 11
66
92% 2%
6%
C
16 yr
8
60
80% 16% 4%
D
E
44 yr
Cord
10
12
66
54
98%
2%
↑↑↑
Remarks
Golf ball RBC
Remarks
↑ RCC
Ectopic
marrow
↓ α/β chain
Dead
Newborn
Q 615. One of the following patterns in hemoglobin electrophoresis supports the
diagnosis of β- thalassemia major:
Answer Age
Hb MCV HbA HbF Hb Hb
HbS HbH HbC HbE
Barts
gm/dL
A2
A
5 yr 8
80
6%
2%
92%
B.
2 yr 6
60
98% 2%
C
18 yr 8
66
70%
2%
↑
20%
D
8 yr 8
64
12%
80%
E
12 yr 10
80
4%
44%
40%
Remarks
Golf ball RBC
Q 616. One of the following patterns in hemoglobin electrophoresis supports the
diagnosis of β- thalassemia minor:
Answer
Age
Hb
MCV
HbA
HbF
gm/dL
A
B.
C
24 yr
18 yr
16 yr
12
11
8
82
66
60
56%
92%
80%
D
E
44 yr
Cord
10
12
66
54
98%
2%
16%
Hb
A2
2%
6%
4%
Hb
Barts
HbS
HbH
HbC
HbE
Remarks
42%
↑ RCC
Ectopic
marrow
↓ α/β chain
2%
↑↑↑
Dead
Newborn
87
Professor Anwar Sheikha, MD, FRCP, FRCPath., FACP
One Thousand Hematology MCQ
Q 617. One of the following patterns in hemoglobin electrophoresis supports the
diagnosis of β- thalassemia intermedia:
Answer Age
Hb MCV HbA HbF Hb Hb
HbS HbH HbC HbE
Barts
gm/dL
A2
A
24 yr 12
82
56%
2%
42%
B
18 yr 11
66
92% 2%
6%
C.
16 yr
8
60
80% 16% 4%
D
E
44 yr
Cord
10
12
66
54
98%
2%
↑↑↑
Q 618. One of the following patterns in hemoglobin electrophoresis supports the
diagnosis of α- thalassemia minor:
Answer Age
Hb MCV HbA HbF Hb Hb
HbS HbH HbC HbE
Barts
gm/dL
A2
A
24 yr 12
82
56%
2%
42%
B
18 yr 11
66
92% 2%
6%
C
16 yr
8
60
80% 16% 4%
D.
E
44 yr
Cord
10
12
66
54
98%
2%
↑↑↑
Q 619. One of the following patterns in hemoglobin electrophoresis supports the
diagnosis of Hemoglobin Barts Hydrops Fetalis:
Answer Age
Hb MCV HbA HbF Hb Hb
HbS HbH HbC HbE
Barts
gm/dL
A2
A
24 yr 12
82
56%
2%
42%
B
18 yr 11
66
92% 2%
6%
C
16 yr
8
60
80% 16% 4%
D
E.
44 yr
Cord
10
12
66
54
98%
2%
↑↑↑
Q 620. One of the following patterns in hemoglobin electrophoresis supports the
diagnosis of Hemoglobin H disease:
Answer Age
Hb MCV HbA HbF Hb Hb
HbS HbH HbC HbE
Barts
gm/dL
A2
A
5 yr 8
80
6%
2%
92%
B
2 yr 6
60
98% 2%
C.
18 yr 8
66
70%
2%
↑
20%
D
8 yr 8
64
12%
80%
E
12 yr 10
80
4%
44%
40%
Remarks
↑ RCC
Ectopic
marrow
↓ α/β chain
Dead
Newborn
Remarks
↑ RCC
Ectopic
marrow
↓ α/β chain
Dead
Newborn
Remarks
↑ RCC
Ectopic
marrow
↓ α/β chain
Dead
Newborn
Remarks
Golf ball RBC
88
Professor Anwar Sheikha, MD, FRCP, FRCPath., FACP
One Thousand Hematology MCQ
Q 621. One of the following patterns in hemoglobin electrophoresis supports the
diagnosis of Hemoglobin SC disease:
Answer Age
Hb MCV HbA HbF Hb Hb
HbS HbH HbC HbE
Barts
gm/dL
A2
A
5 yr 8
80
6%
2%
92%
B
2 yr 6
60
98% 2%
C
18 yr 8
66
70%
2%
↑
20%
D
8 yr 8
64
12%
80%
E.
12 yr 10
80
4%
44%
40%
Q 622. One of the following patterns in hemoglobin electrophoresis supports the
diagnosis of Hemoglobin E Disease in a Bangladeshi child:
Answer Age
Hb MCV HbA HbF Hb Hb
HbS HbH HbC HbE
Barts
gm/dL
A2
A
5 yr 8
80
6%
2%
92%
B
2 yr 6
60
98% 2%
C
18 yr 8
66
70%
2%
↑
20%
D.
8 yr 8
64
12%
80%
E
12 yr 10
80
4%
44%
40%
Remarks
Golf ball RBC
Remarks
Golf ball RBC
Q623. Thalassemias are common in all of the following world regions EXCEPT:
A.
Middle East
B.
Mediterranean countries
C.
India
D.
South East Asia
E
Sweden
Q624. Hemoglobin Barts is composed of:
A.
α2β2
B.
α2γ2
C.
α2δ2
D.
β4
E
γ4
Q625. Hemoglobin H is composed of:
A.
α2β2
B.
α2γ2
C.
α2δ2
D.
β 4.
E.
γ4
Q626. All of the following red cell changes are expected in β- thalassemia major
EXCEPT:
A.
Marked anisocytosis
B.
Marked poikilocytosis
C
Macrocytosis
D.
Target cells
89
Professor Anwar Sheikha, MD, FRCP, FRCPath., FACP
E.
One Thousand Hematology MCQ
Numerous nucleated red cells
Q627. HbF approaching 100% is a typical feature of one of the following diseases:
A.
β- thalassemia minor
B
β- thalassemia major
C.
α- thalassemia minor
D.
HbH Disease
E.
Sickle Cell disease
Q628. Mild elevation of HbF is expected in all but one of the following conditions:
A.
Diamond Blackfan disease
“congenital pure red cell aplasia”
B.
Fanconi’s anemia
“congenital aplastic anemia”
C.
MDS
“Myelodysplastic Syndromes”
D.
JCML
“Juvenile Chronic Myeloid Leukemia”
E
Aplastic anemia
Q629. Normal range for HbA2 is:
A
1.5 to 3.2%
B.
< 1%
C.
97%
D.
Undetectable
E.
100%
Q630. Normal range for HbA is:
A.
1.5 to 3.2%.
B.
< 1%
C
97%
D.
Undetectable
E.
100%
Q631. Normal range for HbF in an adult is:
A
1.5 to 3.2%
B.
< 1%.
C.
97%
D.
Undetectable
E.
100%
Q632. All of the following changes are expected in β- thalassemia minor EXCEPT:
A.
Hypochromic red cells
B.
Microcytic red cells
C.
Basophilic stippling
D.
Target red cells
E
Very high RDW “red cell distribution width”
Q633. In HbH disease:
A.
No
α chain is missing
B.
one α chain is missing
C.
Two α chains are missing
D
Three α chains are missing
E.
All of the four α chains are missing
90
Professor Anwar Sheikha, MD, FRCP, FRCPath., FACP
One Thousand Hematology MCQ
Q634. In Hb Barts Hydrops Fetalis :
A.
No
α chain is missing
B.
one α chain is missing
C.
Two α chains are missing
D.
Three α chains are missing
E
All of the four α chains are missing
Q635. In α- thalassemia minor:
A.
No
α chain is missing
B.
one α chain is missing
C
Two α chains are missing
D.
Three α chains are missing
E.
All of the four α chains are missing
Q636. Crenated red cells “Echinocytes” are usually seen:
A.
Hereditary spherocytosis
B.
Hereditary elliptocytosis
C.
Iron deficiency anemia
D.
Thalassemia
E
Blood films prepared from old blood “storage artifact”
Q637. All of the following features relate to Echinocytes “Crenated red cells” EXCEPT:
A.
Red cells with numerous short, regular projections from their surface
B
Red cells with small number of spicules of variable length, thickness
& shape on their surface
C.
Few crenated red cells could be seen in blood films from healthy people
D.
Numerous crenated red cells in freshly-made & well-stained blood smear
is a significant observation
E.
Usually seen in uremic patients “burr cells”
Q638. All of the following features relate to Acanthocytes EXCEPT:
A.
They are red cells with numerous short, regular membrane projections .
B
They are red cells with small number of spicules of variable length,
Thickness & shape on their surface
C.
Acanthocytes are often associated with abnormal lipid metabolism
D.
Seen in red cells that lack Kell antigen “McLeod blood Group”
E.
Could be seen in splenectomized patients or in splenic atrophy
Q639. Pure Poikilocytosis is not common in one of the following diseases:
A.
Megaloblastic anemia
B.
Iron deficiency anemia
C.
Thalassemia
D.
Myelofibrosis
E
Hereditary spherocytosis
Q640. One of the following is more in favor of Iron deficiency anemia than thalassemia
minor:
A.
Basophilic stippling
B.
Target cells
C
High RDW “red cell distribution width”
D.
High HbA2
91
Professor Anwar Sheikha, MD, FRCP, FRCPath., FACP
E.
One Thousand Hematology MCQ
Normal iron stores
Q641. Nucleated red cells could be seen in all of the following anemias EXCEPT:
A
Aplastic anemia
B.
Thalassemia major
C.
Hemolytic disease of Newborn
D.
Autoimmune hemolytic anemias
E.
G6P Dehydrogenase deficiency
Q642. Nucleated red cells is a prominent feature of one of the following diseases:
A.
Diamond Blackfan disease “congenital pure red cell aplasia”
B.
Fanconi’s anemia “congenital aplastic anemia”
C.
Polycythemia rubra vera
D
Myelofibrosis
E.
Essential thrombocythemia
Q643. Red cell osmotic fragility could be increased in all of the following diseases
EXCEPT:
A.
Hereditary spherocytosis
B.
Severe autoimmune hemolytic anemia
C
Thalassemia
D.
Hereditary elliptocytosis
E.
Evans’ syndrome
Q644. Leptocytosis “thin red cells”
is seen in one of the following blood diseases:
A
Severe iron deficiency anemia
B.
A β-lipoproteinemia
C.
MAHA “Micro-angiopathic hemolytic anemia”
D.
Artifactual
E.
AIHA “Autoimmune hemolytic anemia”
Q645. Acanthocytosis is seen in one of the following blood diseases:
A.
Severe iron deficiency anemia
B
A β-lipoproteinemia
C.
MAHA “Micro-angiopathic hemolytic anemia”
D.
Artifactual
E.
AIHA “Autoimmune hemolytic anemia”
Q646. Spherocytosis is seen in one of the following blood diseases:
A.
Severe iron deficiency anemia
B.
A β-lipoproteinemia
C.
MAHA “Micro-angiopathic hemolytic anemia”
D.
Artifactual
E
AIHA “Autoimmune hemolytic anemia”
Q647. Schistocytosis “fragmented red cells” are seen in one of the following blood
diseases:
A.
Severe iron deficiency anemia
B.
A β-lipoproteinemia
C
MAHA “Micro-angiopathic hemolytic anemia”
D.
Artifactual
92
Professor Anwar Sheikha, MD, FRCP, FRCPath., FACP
E.
One Thousand Hematology MCQ
AIHA “Autoimmune hemolytic anemia”
Q648. Echinocytosis “Crenated red cells” are usually:
A.
Severe iron deficiency anemia
B.
A β-lipoproteinemia
C.
MAHA “Micro-angiopathic hemolytic anemia”
D
Artifactual
E.
AIHA “Autoimmune hemolytic anemia”
Q649. All of the following features relate to Leptocytosis EXCEPT:
A.
They are unusually thin red cells in which the cells are stained as rings of
hemoglobin surrounding a large almost unstained central area
B.
Seen in severe iron deficiency anemia
C.
Seen in thalassemia major
D
Seen frequently in hereditary spherocytosis
E.
It is the result of severe lack of hemoglobin synthesis
Q650. Hemoglobin S is composed of molecules of hemoglobin in which :
A
Glutamic acid is replaced by Valine at position 6 of the β- globin chain
B.
Valine is replaced by Glutamic acid at position 6 of the β- globin chain
C.
Glutamic acid is replaced by Lysine at position 6 of the β- globin chain
D.
Glutamic acid is replaced by Lysine at position 26 of the β- globin chain
E.
Glutamic acid is replaced by Valine at position 26 of the β- globin chain
Q651. Hemoglobin C is composed of molecules of hemoglobin in which :
A.
Glutamic acid is replaced by Valine at position 6 of the β- globin chain
B.
Valine is replaced by Glutamic acid at position 6 of the β- globin chain
C
Glutamic acid is replaced by Lysine at position 6 of the β- globin chain
D.
Glutamic acid is replaced by Lysine at position 26 of the β- globin chain
E.
Glutamic acid is replaced by Valine at position 26 of the β- globin chain
Q652. Hemoglobin E is composed of molecules of hemoglobin in which :
A.
Glutamic acid is replaced by Valine at position 6 of the β- globin chain
B.
Valine is replaced by Glutamic acid at position 6 of the β- globin chain
C.
Glutamic acid is replaced by Lysine at position 6 of the β- globin chain
D
Glutamic acid is replaced by Lysine at position 26 of the β- globin chain
E.
Glutamic acid is replaced by Valine at position 26 of the β- globin chain
Q653. Ring Sideroblasts are seen in:
A
Sideroblastic anemia
B.
Glucose 6 Phosphate Dehydrogenase Deficiency
C.
Lead poisoning
D.
Megaloblastic anemias
E.
Malaria
Q654. Punctate basophilia is a well-known feature of:
A.
Sideroblastic anemia
B.
Glucose 6 Phosphate Dehydrogenase Deficiency
C
Lead poisoning
D.
Megaloblastic anemias
E.
Malaria
93
Professor Anwar Sheikha, MD, FRCP, FRCPath., FACP
One Thousand Hematology MCQ
Q655. Heinz bodies are usually seen in:
A.
Sideroblastic anemia
B
Glucose 6 Phosphate Dehydrogenase Deficiency
C.
Lead poisoning
D.
Megaloblastic anemias
E.
Malaria
Q656. Howell-Jolly bodies could be seen in:
A.
Sideroblastic anemia
B.
Glucose 6 Phosphate Dehydrogenase Deficiency
C.
Lead poisoning
D
Megaloblastic anemias
E.
Malaria
Q657. Neutrophil hypersegmentation is said to be present in all of the following
situations EXCEPT:
A.
Any neutrophil with more than 6 segments
B.
If >5% of the neutrophils have more than 5 segments
C.
If more than 15% of neutrophils have more than 4 segments
D
Whenever Pseudo-Pelger cells are abundant
E.
When the majority of the neutrophil have more than 5 segments
Q658. Neutrophil hypersegmentation is an expected feature of all of the following
diseases EXCEPT:
A
Pelger-Huet Anomaly
B.
Megaloblastic anemia
C.
Patients receiving Hydroxyurea treatment
D.
Uremia
E.
Very occasionally as a familial benign abnormality
Q659. Pelger-Huet or Pseudo-Pelger cells could be seen in:
A.
AML with dysplasia
B.
Pelger-Huet anomaly
C.
MDS “Myelodysplastic Syndromes”
D
All of the above
E.
None of the above
Q660. High Hemoglobin F approaching 100% is mainly seen in:
A.
β- thalassemia minor
B
β- thalassemia major
C.
α- thalassemia major (Hemoglobin Bart’s Hydrops fetalis)
D.
α- thalassemia minor
E.
Hemoglobin H disease
Q661. High hemoglobin A2 is a diagnostic feature of :
A
β- thalassemia minor
B.
β- thalassemia major
C.
α- thalassemia major (Hemoglobin Bart’s Hydrops fetalis)
D.
α- thalassemia minor
E.
hemoglobin H disease
94
Professor Anwar Sheikha, MD, FRCP, FRCPath., FACP
One Thousand Hematology MCQ
Q662. Golf- ball appearing red cells are seen in:
A.
β- thalassemia minor
B.
β- thalassemia major
C.
Hemoglobin Bart’s Hydrops fetalis
D.
α- thalassemia minor
E
Hemoglobin H disease
Q663. A dead fetus with a big placenta and severely hypochromic microcytic red cells
could have:
A.
β- thalassemia minor
B.
β- thalassemia major
C
Hemoglobin Bart’s Hydrops fetalis
D.
α- thalassemia minor
E.
Hemoglobin H disease.
Q664. Mild elevation of Hemoglobin F is expected in all of the following diseases
EXCEPT:
A.
Diamond Blackfan Disease
B.
Fanconi’s anemia
C.
Myelodysplastic syndrome
D.
Juvenile Chronic Myeloid Leukemia
E
Evans’ syndrome
Q665. The normal Hb A2 is:
A.
97%
B
1.5 – 3.2%
C.
< 1%
D.
Undetectable
E.
100%
Q666. The normal Adult Hemoglobin (Hb A) is:
A
97%
B.
1.5 – 3.2%
C.
< 1%
D.
Undetectable
E.
100%
Q667. Hemoglobin F in an adult male patient is usually:
A.
97%
B.
1.5 – 3.2%
C
< 1%
D.
Undetectable
E.
100%
Q668. Hemoglobin E is common in:
A.
West Africa
B.
Tropical Africa
C
South East Asia
D.
Mediterranean countries
E.
Europe
95
Professor Anwar Sheikha, MD, FRCP, FRCPath., FACP
One Thousand Hematology MCQ
Q669. Hemoglobin C is common in:
A
West Africa
B.
Tropical Africa
C.
South East Asia
D.
Mediterranean countries
E.
Europe
Q670. β- thalassemia is common in:
A.
West Africa
B.
Tropical Africa
C.
United States of America
D
Mediterranean countries
E.
Europe
Q671. Eosinophilia is a known feature of all of the following conditions EXCEPT:
A
Prolonged steroid therapy
B.
Asthma
C.
Filariasis
D.
Hodgkin’s Disease
E.
Hay fever
Q672. All of the following white cell abnormalities are expected in
severe bacterial infection “sepsis” EXCEPT:
A.
Neutrophilia
B.
Toxic granulation of neutrophils
C.
High Neutrophil Alkaline Phosphatase
D.
Left shifting of neutrophil maturation
E
Low CD4 positive lymphocyte count
Q673. In patients with AIDS “Acquired Immunodeficiency Syndrome”
a remarkable diagnostic finding is:
A.
Neutrophilia
B.
Toxic granulation of neutrophils
C.
High Neutrophil Alkaline Phosphatase
D.
Left shifting of neutrophil maturation
E
Low CD4 positive lymphocyte count
Q674. All of the following are expected in severe bacterial infection “sepsis” EXCEPT:
A
Right shifting of neutrophil maturation
B.
Döhle bodies
C.
Hyperpyrexia
D.
Neutrophil cytoplasmic vacuoles
E.
Leukemoid reaction
Q675. The life-span of platelets is usually around:
A.
~120 days
B
~ 10 days
C.
~ 7 hours
D.
~ 20 days
E.
Could reach years!
96
Professor Anwar Sheikha, MD, FRCP, FRCPath., FACP
One Thousand Hematology MCQ
Q676. The life span of normal red cells is usually around:
A
~120 days
B.
~ 10 days
C.
~ 7 hours
D.
~ 20 days
E.
Could reach years!
Q677. The life-span of irreversibly sickle cells is usually around:
A.
~120 days
B.
~ 10 days
C.
~ 7 hours
D
~ 20 days
E.
Could reach years!
Q678. The life-span of memory B lymphocytes is:
A.
~120 days
B.
~ 10 days
C.
~ 7 hours
D.
~ 20 days
E
Could reach years!
Q679. The circulation half-life of neutrophils is usually around:
A.
~120 days
B.
~ 10 days
C
~ 7 hours
D.
~ 20 days
E.
Could reach years!
Q680. All of the following could be early manifestations of intravascular hemolysis
EXCEPT:
A.
Hemoglobinemia
B.
Hemoglobinuria
C
Hemosiderinuria
D.
Low Haptoglobin
E.
Methemalbuminemia
Q681. One week after an episode of intravascular hemolytic anemia,
the only left-over manifestation of the hemolysis might be:
A.
Hemoglobinemia
B.
Hemoglobinuria
C
Hemosiderinuria
D.
Low Haptoglobin
E.
Methemalbuminemia
Q682. Type 1 myelodysplastic syndrome “Refractory Anemia” could present itself as:
A.
Refractory anemia
B.
Neutropenia
C.
Thrombocytopenia
D
All of the above
E.
None of the above
97
Professor Anwar Sheikha, MD, FRCP, FRCPath., FACP
One Thousand Hematology MCQ
Q683. In myelodysplastic syndromes, isolated refractory macrocytic anemia is called:
A
Refractory anemia
B.
Refractory anemia with ring sideroblasts
C.
Refractory anemia with excessive blasts
D.
Chronic myelomonocytic leukemia
E.
5 q- syndrome
Q684. In myelodysplastic syndromes, marrow blast cells of 18% indicates:
A.
Refractory anemia
B.
Refractory anemia with ring sideroblasts
C
Refractory anemia with excessive blasts
D.
Chronic myelomonocytic leukemia
E.
5 q- syndrome
Q685. In myelodysplastic syndromes,
presence of 33% ring sideroblasts in the marrow erythroid cells indicates:
A.
Refractory anemia
B
Refractory anemia with ring sideroblasts
C.
Refractory anemia with excessive blasts
D.
Chronic myelomonocytic leukemia
E.
5 q- syndrome
Q686. In myelodysplastic syndromes, a dimorphic blood picture is a unique feature of :
A.
Refractory anemia
B
Refractory anemia with ring sideroblasts
C.
Refractory anemia with excessive blasts
D.
Chronic myelomonocytic leukemia
E.
5 q- syndrome
Q687. In myelodysplastic syndromes, the worst prognosis is usually associated with:
A.
Refractory anemia
B.
Refractory anemia with ring sideroblasts
C
Refractory anemia with excessive blasts
D.
Chronic myelomonocytic leukemia
E.
5 q- syndrome
Q688. In myelodysplastic syndromes, the highest incidence of leukemia transformation
is in:
A.
Refractory anemia
B.
Refractory anemia with ring sideroblasts
C
Refractory anemia with excessive blasts
D.
Chronic myelomonocytic leukemia
E.
5 q- syndrome
Q689. In myelodysplastic syndromes, thrombocytosis is a unique feature of:
A.
Refractory anemia
B.
Refractory anemia with ring sideroblasts
C.
Refractory anemia with excessive blasts
D.
Chronic myelomonocytic leukemia
E
5 q- syndrome
98
Professor Anwar Sheikha, MD, FRCP, FRCPath., FACP
One Thousand Hematology MCQ
Q690. In myelodysplastic syndromes, monocytosis is a unique feature of:
A.
Refractory anemia
B.
Refractory anemia with ring sideroblasts
C.
Refractory anemia with excessive blasts
D
Chronic myelomonocytic leukemia
E.
5 q- syndrome
Q691. In myelodysplastic syndromes, ALIP “Abnormal Localization of Immature
Precursors” is mostly seen in:
A.
Refractory anemia
B.
Refractory anemia with ring sideroblasts
C
Refractory anemia with excessive blasts
D.
Chronic myelomonocytic leukemia
E.
5 q- syndrome
Q692. In the new W.H.O. classification of hematological malignancies, t(15;17)
is linked to which type of leukemia:
A.
M2
“Acute myeloid leukemia with differentiation
B
M3
“Acute hypergranular promyelocytic leukemia
C.
M4-Eo “Acute myelomonocytic leukemia with Eosinophilia
D.
5q- syndrome
E.
Chronic myeloid leukemia
Q693. In the new W.H.O. classification of hematological malignancies, t(8;21)
is linked to which type of leukemia:
A
M2
“Acute myeloid leukemia with differentiation
B.
M3
“Acute hypergranular promyelocytic leukemia
C.
M4-Eo “Acute myelomonocytic leukemia with Eosinophilia
D.
5q- syndrome
E.
Chronic myeloid leukemia
Q694. In the new W.H.O. classification of hematological malignancies, inv(16)
is linked to which type of leukemia:
A.
M2
“Acute myeloid leukemia with differentiation
B.
M3
“Acute hypergranular promyelocytic leukemia
C
M4-Eo “Acute myelomonocytic leukemia with Eosinophilia
D.
5q- syndrome
E.
Chronic myeloid leukemia
Q695. In the W.H.O. classification of hematological malignancies, t(9;22) is linked to:
A.
M2
“Acute myeloid leukemia with differentiation
B.
M3
“Acute hypergranular promyelocytic leukemia
C.
M4-Eo “Acute myelomonocytic leukemia with Eosinophilia
D.
5q- syndrome
E
Chronic myeloid leukemia
Q696. Bone marrow blast cells in CMML “chronic myelomonocytic leukemia” is:
A
0 to 20%
B.
20 to 30%
C.
> 30%
D.
Almost 100%
99
Professor Anwar Sheikha, MD, FRCP, FRCPath., FACP
E.
One Thousand Hematology MCQ
None of the above
Q697. In the W.H.O. classification of hematological malignancies, deletion of the long
arm of one of the chromosomes associated with thrombocytosis is called:
A.
M2
“Acute myeloid leukemia with differentiation
B.
M3
“Acute hypergranular promyelocytic leukemia
C.
M4-Eo “Acute myelomonocytic leukemia with Eosinophilia
D
5q- syndrome
E.
Chronic myeloid leukemia
Q698. In myelodysplastic syndromes, the highest chance of leukemia transformation is
seen in:
A.
Refractory anemia
B.
Refractory anemia with ring sideroblasts
C
Refractory anemia with excessive blasts
D.
Refractory cytopenia with multilineage dysplasia
E.
Chronic myelomonocytic leukemia
Q699. Management of myelodysplastic syndromes, especially in elderly, usually
concentrates on all of the following EXCEPT:
A.
Supportive care
B.
Erythropoietin
C.
G-CSF
D.
GM-CSF
E
Aggressive chemotherapy for all patients
Q700. In myelodysplastic syndromes, presence of Auer rods in the peripheral blood
indicates:
A.
Refractory anemia
B.
Refractory anemia with ring sideroblasts
C.
Refractory anemia with excessive blasts
D.
Chronic myelomonocytic leukemia
E
None of the above
Q701. A patient with isolated neutropenia and generalized dysplastic marrow has:
A.
Refractory anemia
B.
Refractory anemia with ring sideroblasts
C.
Refractory anemia with excessive blasts
D
Refractory cytopenia with multilineage dysplasia
E.
Chronic myelomonocytic leukemia
Q702. All of the followings are related to Acute Promyelocytic Leukemia EXCEPT:
A.
Responds very well to the differentiating drug ATRA
B.
Disseminated intravascular coagulation is an expected complication
C.
t(15;17) is a diagnostic cytogenetic abnormality
D
Gum hypertrophy is a well-known association
E.
It has been moved from M3 of FAB classification to the new group of
AML with cytogenetic abnormalities in the W.H.O. classification
100
Professor Anwar Sheikha, MD, FRCP, FRCPath., FACP
One Thousand Hematology MCQ
Q703. The following dysplastic features are associated with myelodysplastic
syndromes:
A.
Dyserythropoiesis
B.
Dysgranulopoiesis
C.
Dysmegakaryopoiesis
D
All of the above
E.
None of the above
Q704. The least likely chance for leukemia transformation in myelodysplastic
syndromes is in:
A.
Refractory anemia
B
Refractory anemia with ring sideroblasts
C.
Refractory anemia with excessive blasts
D.
Refractory cytopenia with multilineage dysplasia
E.
Chronic myelomonocytic leukemia
Q705. The longest survival in myelodysplastic syndromes is in:
A.
Refractory anemia
B
Refractory anemia with ring sideroblasts
C.
Refractory anemia with excessive blasts
D.
Refractory cytopenia with multilineage dysplasia
E.
Chronic myelomonocytic leukemia
Q706. Normal score for Neutrophil Alkaline Phosphatase (NAP) is:
A
15 to 100
B.
100 to 200
C.
200 to 300
D.
300 to 400
E.
400 to 1000
Q707. Neutrophil Alkaline Phosphatase (NAP) score is low in:
A
PNH “Paroxysmal Nocturnal Hemoglobinuria”
B.
Aplastic anemia
C.
Hodgkin lymphoma
D.
Leukemoid reactions
E.
Polycythemia rubra vera
Q708. Neutrophil Alkaline Phosphatase (NAP) score is low in:
A
CML “Chronic Myeloid Leukemia”
B.
Infections
C.
Newborns
D.
Children
E.
Pregnant ladies
Q709. Periodic Acid Schiff “PAS” is usually positive in blasts of:
A.
ALL “Acute Lymphoblastic Leukemia”
B.
Erythroleukemia
C.
Megakaryoblastic leukemia
D
All of the above.
E.
None of the above
101
Professor Anwar Sheikha, MD, FRCP, FRCPath., FACP
One Thousand Hematology MCQ
Q710. Ham’s test is:
A.
Positive in PNH “Paroxysmal Nocturnal Hemoglobinuria”
B.
Could be positive in HEMPAS “Hereditary Erythroid Multinuclearity
with Positive Acidified Serum Test”
C.
Negative in AIHA “Autoimmune Hemolytic Anemia”
D
All of the above
E.
None of the above
Q711. Localized polar cytoplasmic positivity for Acid Phosphatase in blast cells favors
the diagnosis of:
A.
Hairy cell leukemia
B.
AML (M5)
“Acute monoblastic leukemia”
C.
AML (M1)
“Acute myeloid leukemia without maturation”
D.
B-ALL
“B- Acute Lymphoblastic leukemia”
E
T-ALL
“T- Acute Lymphoblastic Leukemia”
Q712. Cytoplasmic block positivity for PAS “Periodic Acid Schiff” in blast cells favors the
diagnosis of:
A.
Hairy cell leukemia
B.
AML (M5)
“Acute monoblastic leukemia”
C.
AML (M1)
“Acute myeloid leukemia without maturation”
D
B-ALL
“B- Acute Lymphoblastic leukemia”
E.
T-ALL
“T- Acute Lymphoblastic Leukemia”
Q713. Myeloperoxidase positive marrow blast cells of more than 3% favor diagnosis of:
A.
Hairy cell leukemia
B.
AML (M5)
“Acute monoblastic leukemia”
C
AML (M1)
“Acute myeloid leukemia without maturation”
D.
B-ALL
“B- Acute Lymphoblastic leukemia”
E.
T-ALL
“T- Acute Lymphoblastic Leukemia”
Q714. Non-specific Butyrate Esterase positivity with discrete scattered positivity for
Sudan Black in blasts favors the diagnosis:
A.
Hairy cell leukemia
B
AML (M5)
“Acute monoblastic leukemia”
C.
AML (M1)
“Acute myeloid leukemia without maturation”
D.
B-ALL
“B- Acute Lymphoblastic leukemia”
E.
T-ALL
“T- Acute Lymphoblastic Leukemia”
Q715. Tartarate Resistant Acid Phosphatase positivity favors the diagnosis of:
A
Hairy cell leukemia
B.
AML (M5)
“Acute monoblastic leukemia”
C.
AML (M1)
“Acute myeloid leukemia without maturation”
D.
B-ALL
“B- Acute Lymphoblastic leukemia”
E.
T-ALL
“T- Acute Lymphoblastic Leukemia”
Q716. All but one of the following features is part of dyserythropoiesis:
A.
Basophilic stippling
B.
Cytoplasmic bridging between divided red cell precursors
C.
Inter-nuclear bridging between divided red cell precursors
D
Pelger-Huet like cells
102
Professor Anwar Sheikha, MD, FRCP, FRCPath., FACP
E.
One Thousand Hematology MCQ
Red cell precursor multinuclearity
Q717. One of the following features could result from dysgranulopoiesis:
A.
Basophilic stippling
B.
Cytoplasmic bridging between divided red cell precursors
C.
Inter-nuclear bridging between divided red cell precursors
D
Pelger-Huet like cells
E.
Red cell precursor multinuclearity
Q718. All of the following features relate to Congenital Dyserythropoietic Anemia
(CDA) Type I EXCEPT:
A.
Anisopoikilocytosis in the blood smear with many macrocytes
B.
Active marrow with erythroid hyperplasia
C.
Inter-nuclear bridging of the red cell precursors in the marrow
D
Ring Sideroblasts in the bone marrow of more than 15%
E.
Basophilic stippling
Q719. All of the following features relate to Congenital Dyserythropoietic Anemia
(CDA) Type II EXCEPT:
A.
Binuclearity of the red cell precursors in the marrow
B.
Multinuclearity of the red cell precursors in the marrow
C
Ring Sideroblasts in the bone marrow of more than 15%
D.
Negative Coomb’s test
E.
Possible positive Ham’s test
Q720. HEMPAS “Hereditary Erythroid Multinuclearity with Positive Acidified Serum
Test” or Congenital Dyserythropoietic Anemia (CDA) Type II is:
A.
A sex linked disease
B.
An autosomal dominant disease
C.
An acquired disease
D.
All of the above
E
None of the above
Q721. All of the following features relate to Congenital Dyserythropoietic Anemia
(CDA) Type III EXCEPT:
A.
Binuclearity of the red cell precursors in the marrow
B.
Multinuclearity of the red cell precursors in the marrow
C.
Gigantoblasts from super-sized multinucleated red cell precursors
D.
Macrocytosis
E
Positive Ham’s test
Q722. Basophilic stippling is a well-recognized dyserythropoietic feature in all but
one of the following conditions:
A.
Congenital Dyserythropoietic Anemia (CDA) Type I
B.
Congenital Dyserythropoietic Anemia (CDA) Type II
C.
Congenital Dyserythropoietic Anemia (CDA) Type III
D.
Severe megaloblastic anemia
E
Hereditary spherocytosis
103
Professor Anwar Sheikha, MD, FRCP, FRCPath., FACP
One Thousand Hematology MCQ
Q723. In Myelodysplastic syndromes, trilineage dysplasia (dyserythropoiesis,
dysmyelopoiesis and dysmegakaryopoiesis) is a more marked feature of one of
the following subtypes:
A.
Refractory anemia
B.
Refractory anemia with ring sideroblasts
C
Refractory anemia with excessive blasts
D.
Chronic myelomonocytic leukemia
E.
5q- syndrome
Q724. In Myelodysplastic syndromes, blast cell count of less than 5% could be seen in
the bone marrow of all of the following subtypes EXCEPT:
A.
Refractory anemia.
B.
Refractory anemia with ring sideroblasts
C
Refractory anemia with excessive blasts
D.
Chronic myelomonocytic leukemia
E.
5q- syndrome
Q725. According to the new W.H.O. classification of hematological malignancies,
22% blast cells in the bone marrow indicates:
A.
Refractory anemia
B.
Refractory anemia with ring sideroblasts
C.
Refractory anemia with excessive blasts
D
Acute myeloid leukemia
E.
5q- syndrome
Q726. In chronic myelomonocytic leukemia, blast cells in the bone marrow could be:
A.
1%
B.
5%
C.
15%
D
All of the above
E.
None of the above
Q727. Siderocytes or red cell Pappenheimer bodies are usual in one of the followings:
A.
Anemia of chronic disorders
B
Sideroblastic anemia
C.
Iron deficiency anemia
D.
Paroxysmal nocturnal hemoglobinuria with prolonged active intravascular
hemolysis
E.
Glucose-6-Phosphate Dehydrogenase deficiency with prolonged active
intravascular hemolysis
Q728. Ring sideroblasts is a common feature of:
A.
Anemia of chronic disorders
B.
Sideroblastic anemia.
C.
Iron deficiency anemia
D.
PNH with prolonged active intravascular hemolysis
E.
G6PDH deficiency with prolonged active intravascular hemolysis
104
Professor Anwar Sheikha, MD, FRCP, FRCPath., FACP
One Thousand Hematology MCQ
Q729. In immunophenotyping of the acute leukemias, one set of the following markers
are positive in T-ALL:
A.
CD19, CD10 and Cytoplasmic CD22 & CD79a
B
CD2, CD7 and Cytoplasmic CD3
C.
CD13, CD33, CD117 and cytoplasmic MPO “Myeloperoxidase”
D.
CD41, CD42 & CD61
E.
CD79A, CD22, CD19 and CD13, CD33 & MPO
Q730. In immunophenotyping of the acute leukemias, one set of the following markers
are positive in B-ALL:
A
CD19, CD10 and Cytoplasmic CD22 & CD79a
B.
CD2, CD7 and Cytoplasmic CD3
C.
CD13, CD33, CD117 and cytoplasmic MPO “Myeloperoxidase”
D.
CD41, CD42 & CD61
E.
CD79A, CD22, CD19 and CD13, CD33 & MPO
Q731. In immunophenotyping of the acute leukemias, one set of the following markers
are positive in Acute Megakaryoblastic Leukemia (AML M7):
A.
CD19, CD10 and Cytoplasmic CD22 & CD79a
B.
CD2, CD7 and Cytoplasmic CD3
C.
CD13, CD33, CD117 and cytoplasmic MPO “Myeloperoxidase”
D
CD41, CD42 & CD61
E.
CD79A, CD22, CD19 and CD13, CD33 & MPO
Q732. In immunophenotyping of the acute leukemias, one set of the following markers
are positive in AML:
A.
CD19, CD10 and Cytoplasmic CD22 & CD79a
B.
CD2, CD7 and Cytoplasmic CD3
C
CD13, CD33, CD117 and cytoplasmic MPO “Myeloperoxidase”
D.
CD41, CD42 & CD61
E.
CD79A, CD22, CD19 and CD13, CD33 & MPO
Q733. In immunophenotyping of the acute leukemias, one set of the following markers
are positive in Acute Biphenotypic Leukemia (Mixed Lineage Leukemia):
A.
CD19, CD10 and Cytoplasmic CD22 & CD79a
B.
CD2, CD7 and Cytoplasmic CD3
C.
CD13, CD33, CD117 and cytoplasmic MPO “Myeloperoxidase”
D.
CD41, CD42 & CD61
E
CD79A, CD22, CD19 and CD13, CD33 & MPO
Q734. Acute Biphenotypic Leukemias (Mixed Lineage Leukemia) constitute:
A.
< 1% of all the acute leukemias
B
~5% of all the acute leukemias
C.
>20% of all the acute leukemias
D.
>30% of all the acute leukemias
E.
~80% of all the acute leukemias
105
Professor Anwar Sheikha, MD, FRCP, FRCPath., FACP
One Thousand Hematology MCQ
Q735. One of the following criteria is not related to β-thalassemia minor:
A.
Hemoglobin A2 of 5%
B.
Hypochromic microcytic red cells
C.
Presence of target cells
D
Hemoglobin F of 100%
E.
High red cell count
Q736. One of the following criteria is related to β-thalassemia minor:
A
Hemoglobin A2 of 6%
B.
Low MCV, MCH and high MCHC
C.
Huge spleen
D.
Hemoglobin F of 100%
E.
Severe anemia
Q737. One of the following criteria is not related to β -thalassemia major:
A
Hemoglobin A2 of 6%
B.
Low MCV, MCH and MCHC
C.
Huge spleen
D.
Hemoglobin F of 100%
E.
Severe anemia
Q738. The normal adult male hemoglobin is:
A.
12.0 to 15.0 gm/dL
B
13.0 to 17.0 gm/dL
C.
13.5 to 19.5 gm/dL
D.
9.5 to 13.5 gm/dL
E.
9.5 to 19.5 gm/dL
Q739. The normal adult female hemoglobin is:
A
12.0 to 15.0 gm/dL
B.
13.0 to 17.0 gm/dL
C.
13.5 to 19.5 gm/dL
D.
9.5 to 13.5 gm/dL
E.
9.5 to 19.5 gm/dL
Q740. The normal hemoglobin in newborns could range between:
A.
12.0 and 15.0 gm/dL
B.
13.0 and 17.0 gm/dL
C
13.5 and 19.5 gm/dL
D.
9.5 and 13.5 gm/dL
E.
9.5 and 19.5 gm/dL
Q741. The normal hemoglobin in a two to three months old child could range between:
A.
12.0 and 15.0 gm/dL
B.
13.0 and 17.0 gm/dL
C.
13.5 and 19.5 gm/dL
D
9.5 and 13.5 gm/dL
E.
9.5 and 19.5 gm/dL
106
Professor Anwar Sheikha, MD, FRCP, FRCPath., FACP
One Thousand Hematology MCQ
Q742. One of the following values is abnormal in cord blood:
A.
Hemoglobin of 180 gm/L
B.
Hemoglobin of 140 gm/L
C
MCV of 70 fL
D.
Hematocrit of 60%
E.
Reticulocyte count of 5%
Q743. One of the following values is normal in adult female:
A.
Hemoglobin of 180 gm/L
B
Hemoglobin of 140 gm/L
C.
MCV of 106 fL
D.
Hematocrit of 60%
E.
Reticulocyte count of 5%
Q744. One of the following values is normal in a ten week old child:
A.
Hemoglobin of 180 gm/L
B
Hemoglobin of 100 gm/L
C.
MCV of 106 fL
D.
Hematocrit of 60%
E.
Reticulocyte count of 5%
Q745. In Hereditary spherocytosis:
A.
MCV is low
B.
MCH is low
C.
MCHC is high
D
All of the above
E.
None of the above
Q746. In Iron deficiency anemia:
A.
MCV is low
B.
MCH is low
C.
MCHC is low
D
All of the above
E.
None of the above
Q747. In megaloblastic anemia:
A.
MCV is low
B.
MCH is low
C.
MCHC is low
D.
All of the above
E
None of the above
Q748. Symptomatic sickle cell anemia patients usually start to develop
painful vaso-occlusive crises:
A.
When they are newborns
B.
In the first six months of life
C
In the second six months of life
D.
In early childhood and adolescent period
E.
When they become adults
107
Professor Anwar Sheikha, MD, FRCP, FRCPath., FACP
One Thousand Hematology MCQ
Q749. β- thalassemia major patients start to become severely anemic:
A.
When they are newborns
B.
In the first six months of life
C
In the second six months of life
D.
In early childhood and adolescent period
E.
When they become adults
Q750. A dimorphic blood picture could be seen in one of the following diseases:
A
Sideroblastic anemia
B.
Aplastic anemia
C.
Pure red cell aplasia
D.
Acute leukemia
E.
Lymphoma
Q751. Hemoglobin S in sickle cell trait is usually:
A.
75 to 100%
B
Little less than 50%
C.
10%
D.
1%
E.
Undetectable
Q752. Hemoglobin S in sickle cell anemia patients is usually:
A
75 to 100%
B.
Little less than 50%
C.
10%
D.
1%
E.
Undetectable
Q753. Hemoglobin S in normal adult is usually:
A.
75 to 100%
B.
Little less than 50%
C.
10%
D.
1%
E
Undetectable
Q754. When collecting and processing blood samples from patients,
the technologist must follow all these rules, EXCEPT:
A.
Always recap the syringe before its disposal
B.
Wear disposable gloves
C.
Never re-use the same syringe for another patient
D.
Seek immediate attention if injured or contaminated by blood
E
Do not dispose sharp objects like needles, syringes and
lancets with other things
Q755. To avoid hemolysis during blood sampling, all EXCEPT one of these
rules should be followed by the phlebotomist:
A.
Withdraw the blood gently
B
Use very fine needles
C.
Deliver the blood gently to the tubes
D.
Avoid frothing during blood withdrawal
E.
Gently mix the blood with the anticoagulant
108
Professor Anwar Sheikha, MD, FRCP, FRCPath., FACP
One Thousand Hematology MCQ
Q756. All of the following blood diseases are sex-linked EXCEPT:
A.
Hemophilia A
B.
Christmas disease
C
Thalassemia
D.
Wiskott-Aldrich syndrome
E.
G6PDH deficiency
Q757. A dimorphic blood picture could be seen in all but one
of the following diseases or situations:
A.
Mixed deficiency anemia (iron and folate for example)
B.
Transfused thalassemia patients
C.
Transfused iron deficiency anemia patients
D.
An iron deficiency anemia patient who has been on iron
E
A thalassemia major patient who has been on iron for a month
Q758. Busulphan is a chemotherapeutic agent that used to be used mainly in
management of:
A.
Chronic lymphocytic leukemia
B
Chronic myeloid leukemia
C.
Hairy cell leukemia
D.
Myelofibrosis
E.
Multiple myeloma
Q759. Glivec is an agent mainly used in management of:
A.
Chronic lymphocytic leukemia
B
Chronic myeloid leukemia
C.
Hairy cell leukemia
D.
Myelofibrosis
E.
Multiple myeloma
Q760. Melphalan is a chemotherapeutic agent mainly used in the management of:
A.
Chronic lymphocytic leukemia
B.
Chronic myeloid leukemia
C.
Hairy cell leukemia
D.
Myelofibrosis
E
Multiple myeloma
Q761. Chlorambucil is a chemotherapeutic agent mainly used in the management of:
A
Chronic lymphocytic leukemia
B.
Chronic myeloid leukemia
C.
Hairy cell leukemia
D.
Myelofibrosis
E.
Multiple myeloma
Q762. α- interferon never had a role in the in the management of:
A
Chronic lymphocytic leukemia
B.
Chronic myeloid leukemia
C.
Hairy cell leukemia
D.
Hepatitis C virus infection
109
Professor Anwar Sheikha, MD, FRCP, FRCPath., FACP
One Thousand Hematology MCQ
E.
Melanoma
Q763. Fludarabine is a drug mainly used in the management of:
A
Chronic lymphocytic leukemia
B.
Chronic myeloid leukemia
C.
Hairy cell leukemia
D.
Myelofibrosis
E.
Multiple myeloma
Q764. Anagrelide is a drug mainly used in management of:
A.
Chronic lymphocytic leukemia
B.
Chronic myeloid leukemia
C.
Hairy cell leukemia
D
Essential thrombocythemia
E.
Multiple myeloma
Q765. Glivec “Imatinib Mesylate” is a new drug mainly used in management of:
A.
Chronic lymphocytic leukemia
B
Chronic myeloid leukemia
C.
Hairy cell leukemia
D.
Myelofibrosis
E.
Multiple myeloma
Q766. Drugs could cause all but one of the following anemias:
A.
Bone marrow suppression like with cisplatinum
B.
Hemolytic anemia like with Methyl Dopa
C.
Intravascular hemolysis in Glucose 6 Phosphate Dehydrogenase
Deficient patient receiving dapsone
D.
Aplastic anemia as in a typhoid fever patient receiving chloramphenicol
E
Bone marrow suppression like treatment with vincristine
Q767. Osteolytic lesions could be seen in all but one of the following diseases:
A.
Adult T-Cell Leukemia/Lymphoma
B.
Acute Leukemia
C
Waldenstrom’s macroglobulinemia
D.
Multiple myeloma
E.
Solitary Osseous Plasmacytoma “SOP”
Q768. One of the following groups of cluster designations supports the diagnosis of
multiple myeloma:
A.
CD41 & CD61
B.
CD3 & CD8
C.
CD30 & CD15
D.
CD13, CD33, CD15, MPO & CD117
E
CD38, CD79a, CD56/58 & CD138
Q769. One of the following groups of cluster designations supports the diagnosis of
acute megakaryoblastic Leukemia:
A
CD41 & CD61
B.
CD3 & CD8
C.
CD30 & CD15
D.
CD13, CD33, CD15, MPO & CD117
E.
CD38, CD79a, CD56/58 & CD138
110
Professor Anwar Sheikha, MD, FRCP, FRCPath., FACP
One Thousand Hematology MCQ
Q770. One of the following groups of cluster designations supports the diagnosis of
acute myeloid leukemia:
A.
CD41 & CD61
B.
CD3 & CD8
C.
CD30 & CD15
D
CD13, CD33, CD15, MPO & CD117
E.
CD38, CD79a, CD56/58 & CD138
Q771. One of the following groups of cluster designations supports the diagnosis of
classical Hodgkin lymphoma:
A.
CD41 & CD61
B.
CD3 & CD8
C
CD30 & CD15
D.
CD13, CD33, CD15, MPO & CD117
E.
CD38, CD79a, CD56/58 & CD138
Q772. One of the following groups of cluster designations supports the diagnosis of
Large Granular T-cell Leukemia:
A.
CD41 & CD61
B
CD3 & CD8
C.
CD30 & CD15
D.
CD13, CD33, CD15, MPO & CD117
E.
CD38, CD79a, CD56/58 & CD138
Q773. One of the following features relate to SMOLDERING myeloma:
A.
More than three osteolytic bone lesions
B
Marrow plasmacytosis of <30%
C.
Hypercalcemia
D.
Normal ESR
E.
Urgent need for curative treatment to avoid progression to myeloma
Q774. One of the following features relate to INDOLENT myeloma:
A
Up to three painless lytic bone lesions
B.
Anemia
C.
Infection
D.
Bleeding
E.
Urgent need for curative management to avoid disease progression
Q775. One of the following features relate to MGUS
“Monoclonal Gammopathy of Undetermined Significance”:
A
Marrow plasmacytosis of less than 10%
B.
Up to three lytic bone lesions
C.
Progression to myeloma in the majority of patients
D.
Presence of plasma cells in the blood
E.
IgG paraprotein of more than 35 g/L
111
Professor Anwar Sheikha, MD, FRCP, FRCPath., FACP
One Thousand Hematology MCQ
Q776. During treatment of acute lymphoblastic leukemia, dosage of only one of the
following medications is changed for severe neutropenia or thrombocytopenia:
A
Oral Methotrexate
B.
L-Asparaginase
C.
Vincristine
D.
Steroid
E.
Intrathecal Methotrexate or Ara-C
Q777. Pancreatitis is a well-known complication of one of the following medications
during the management of acute lymphoblastic leukemia in children:
A.
Oral Methotrexate
B
L-Asparaginase
C.
Vincristine
D.
Steroid
E.
Intrathecal Methotrexate or Ara-C
Q778. Neurotoxicity is an expected complication of one of the following medications
during the management of acute lymphoblastic leukemia in children:
A.
Oral Methotrexate
B.
L-Asparaginase
C
Vincristine
D.
Steroid
E.
Intrathecal Methotrexate or Ara-C
Q779. All but one of the followings denote a high risk feature in childhood
acute lymphoblastic leukemia:
A.
Age below one year
B.
Philadelphia chromosome
C.
Slow early response to induction
D
White cell count below 10,000/uL
E.
White cell count above 100,000/uL
Q780. During acute lymphoblastic leukemia management, vincristine neurotoxicity could
present itself as any but one of the following manifestations:
A.
Severe jaw pain
B.
Foot drop
C.
Ileus
D.
Constipation
E
Diarrhea
Q781. Retinoic Acid Syndrome complicating ATRA treatment in Acute Promyelocytic
Leukemia could present itself as any but one of the following manifestations:
A.
It usually happens one to two weeks after treatment
B.
Unexplained fever
C
Usually associated with development of severe neutropenia
D.
Respiratory distress
E.
Renal failure
112
Professor Anwar Sheikha, MD, FRCP, FRCPath., FACP
One Thousand Hematology MCQ
Q782. All but one of the following complications could complicate CLL
“Chronic Lymphocytic Leukemia”:
A
Aplastic anemia
B.
Pure red cell aplasia
C.
Autoimmune hemolytic anemia
D.
Immune thrombocytopenic purpura
E.
Hypogammaglobulinemia
Q783. Patients with chronic lymphocytic leukemia could develop any but one of these:
A.
Varicella zoster
B
Mycosis fungoides
C.
Richter transformation
D.
Prolymphocytic leukemia
E.
Pneumocystis infection
Q784. All but one of the following agents are effective in the management of CLL
“Chronic Lymphocytic Leukemia”:
A
Glivec “Gleevec”
B.
Fludarabine
C.
Rituximab “Anti-CD20”
D.
Chlorambucil
E.
COP Combination Chemotherapy “Chlorambucil, Oncovin & Prednisolone”
Q785. All but one of the following markers are positive in hairy cell leukemia:
A.
CD103
B.
CD22
C.
CD11c
D
CD136
E.
“TRAP” Tartarate Resistant Acid Phosphatase
Q786-790
Match the following lymphoproliferative disorders with the correct
immunophenotype:
A.
Follicular Lymphoma
B.
CLL “Chronic Lymphocytic Leukemia”
C.
Prolymphocytic leukemia
D.
Hairy Cell leukemia
E.
Mantle Cell Lymphoma
?
DR+
CD19+
CD20+
CD5+
CD22-
CD23+
CD10-
Weak SIg
DR+
CD19+
CD20+
CD5-
CD22+
CD23-
CD10-
DR+
CD19+
CD20+
CD5+
CD22+
CD23-
CD10-
Bright
SIg
Mod. SIg
DR+
CD19+
CD20+
CD5-
CD22+
CD23-
CD10+
DR+
CD19+
CD20+
CD5-
CD22+
CD23-
CD10-
786
787
788
Cyclin
D1
789
790
CD11c
+
Bright
SIg
Bright
SIg
BCEAD
113
Professor Anwar Sheikha, MD, FRCP, FRCPath., FACP
One Thousand Hematology MCQ
Q791. In the W.H.O. classification of myelodysplastic syndromes,
all but one of the followings are features of Refractory Anemia:
A.
Anemia
B
Neutropenia
C.
Marrow dyserythropoiesis
D.
Less than 5% marrow Blasts
E.
Less than 15% marrow ring sideroblasts
Q792. In the W.H.O. classification of myelodysplastic syndromes, all but one of the
followings are features of Refractory Cytopenia with Multilineage Dysplasia::
A.
Bicytopenia
B.
Pancytopenia
C
Auer Rods
D.
Less than 5% marrow blasts
E.
Less than 15% marrow ring sideroblasts
Q793. In the W.H.O. classification of myelodysplastic syndromes, only one of the
following features is true in RAEB-2 “Refractory Anemia with Excess Blasts-2:
A.
Monocytosis of more a thousand per uL
B
Auer Rods
C.
Less than 5% peripheral blood blasts
D.
Less than 10% marrow blasts
E.
More than 15% marrow ring sideroblasts
Q794. In the W.H.O. classification of myelodysplastic syndromes, MDS-U
“Myelodysplastic Syndrome-Unclassifiable” could present with all but one of the
following features:
A.
Anemia
B.
Neutropenia
C.
Thrombocytopenia
D
Monocytosis
E.
Marrow megakaryocyte dysplasia
Q795. All but one of the following features are true about pruritus in polycythemia vera:
A.
Usually made worse by bathing
B.
Could be related to basophilia
C
Always respond to antihistamines
D.
Could respond to treatment with hydroxyurea
E.
Cimetidine could be helpful
Q796. All but one of the following features could be seen in polycythemia vera:
A
Low LAP “Leukocyte Alkaline Phosphatase”
B.
Hypercellular bone marrow
C.
Thrombohemorrhagic complications
D.
Budd-Chiari Syndrome
E.
Pruritus
114
Professor Anwar Sheikha, MD, FRCP, FRCPath., FACP
One Thousand Hematology MCQ
Q797. All but one of the following iron status features are true in polycythemia vera:
A.
Serum iron is usually low
B.
TIBC “Total Iron Binding Capacity” is usually high
C.
Serum ferritin is usually low
D
Siderotic granules are abundant in the red cells
E.
Marrow iron stores are usually depleted
Q798. Bone marrow examination is essential in diagnosing:
A.
Polycythemia vera
B.
Autoimmune hemolytic anemia
C.
Chronic myeloid leukemia
D.
Chronic lymphocytic leukemia
E.
Megaloblastic anemia
Q799. All but one of the followings are usual marrow features of polycythemia vera:
A.
Erythroid hyperplasia
B.
Megakaryocyte hyperplasia
C.
Reticulin fibrosis
D.
Myeloid hyperplasia
E
Marrow iron overload
Q800. Increased marrow megakaryocytes is not a feature of one of the followings:
A.
5q- syndrome
B.
ITP “Immune Thrombocytopenic Purpura”
C
Aplastic anemia
D.
Myelofibrosis
E.
Essential thrombocythemia
Q801. All but one of the following parameters are commonly raised in IgG
multiple myeloma:
A.
ESR “Erythrocyte Sedimentation Rate”
B.
LDH “Lactate Dehydrogenase”
C
Serum Alkaline Phosphatase
D.
β2- microglobulin
E.
Serum monoclonal IgG
Q802. The following skeletal radiological abnormalities could be seen in
multiple myeloma:
A.
Generalized osteopenia
B.
Punched-out osteolytic lesions
C.
Pathological fractures
D
All of the above
E.
None of the above
Q803. In the management of multiple myeloma, only one of the following medications
has myelosuppressive effect:
A
Melphalan
B.
Thalidomide
C.
Dexamethasone
D.
Vincristine
115
Professor Anwar Sheikha, MD, FRCP, FRCPath., FACP
One Thousand Hematology MCQ
E.
Prednisolone
Q804. All but one of the following lines are followed in multiple myeloma management:
A.
MP
“Melphalan & Prednisolone”
B.
Thalidomide or Lenalidomide with or without other chemotherapies
C.
VAD “Vincristine, Adriamycin & Dexamethasone”
D
Arsenic trioxide
E.
M2 Protocol “Melphalan, Vincristine, Carmustine & Cyclophosphamide”
Q805. All but one of the following adjuvant therapies could be needed in the
management of multiple myeloma:
A.
Bisphosphonates “Pamidronate or Zoledronate” to prevent bone
resorption
B.
Erythropoietin rather than blood transfusion to correct anemia
C.
Local radiation to alleviate lytic bone pains
D.
Pneumovax to prevent pneumococcal infection
E
Heparinization to prevent DIC “Disseminated Intravascular Coagulation”
Q806. Early management is recommended for the following paraproteinemias:
A.
MGUS “Monoclonal Gammopathy of Undetermined Significance”
B.
Smoldering myeloma
C.
Stage I Multiple Myeloma
D.
All of the above
E
None of the above
Q807. Currently, the most curable type of acute myeloid Leukemia is:
A.
M0
“Minimally Differentiated AML”
B.
M1
“AML without Differentiation”
C.
M2
“AML with Differentiation”
D
M3
“Acute Promyelocytic Leukemia”
E.
M6
“Erythroleukemia”
Q808. All but one of the treatment modalities or drugs are effective in the management
of acute promyelocytic leukemia:
A.
Daunorubicin
B.
ATRA “All Transretinoic Acid”
C
Glivec “Imatinib Mesylate”
D.
Arsenic Trioxide
E.
Stem Cell Transplantation
Q809. Platelets are not large in one of the following diseases:
A
Iron deficiency anemia
B.
Essential Thrombocythemia
C.
ITP “Immune Thrombocytopenic Purpura”
D.
PRV “Polycythemia Rubra Vera”
E.
Bernard-Soulier Syndrome
Q810. Dry tap on bone marrow aspiration is an expected feature of all but one of the
following diseases:
A.
Faulty technique
B.
Myelofibrosis
C
Pure red cell aplasia
D.
Hairy cell leukemia
116
Professor Anwar Sheikha, MD, FRCP, FRCPath., FACP
E.
One Thousand Hematology MCQ
Acute leukemia
Q811. In the Hemoglobin Oxygen-Dissociation curve, Oxygen affinity is decreased
and the curve will be right-shifted in all but one of the following conditions:
A.
Increased concentration of CO2
B.
Increased H+
C.
Increased 2,3 DPG
D
Increased HbF
E.
Increased HbS
Q812. Pelger-Hüet anomaly refers to:
A.
Nuclear fragments left in the developing erythroid cells
B.
Neutrophil cytoplasmic light blue bodies of endoplasmic reticulum
C.
Small diffuse red cell inclusions, composed of RNA
D.
Golf ball appearing red cells seen in supravitally stained smears
E
Bilobed mature neutrophils
Q813. Döhle bodies are:
A.
Nuclear fragments left in the developing erythroid cells
B
Neutrophil cytoplasmic light blue bodies of endoplasmic reticulum
C.
Small diffuse red cell inclusions, composed of RNA
D.
Golf ball appearing red cells seen in supravitally stained smears
E.
Bilobed mature neutrophils
Q814. Basophilic stippling refers to:
A.
Nuclear fragments left in the developing erythroid cells
B.
Neutrophil cytoplasmic light blue bodies of endoplasmic reticulum
C
Small diffuse red cell inclusions, composed of RNA
D.
Golf ball appearing red cells seen in supravitally stained smears
E.
Bilobed mature neutrophils
Q815. Red cells in Hemoglobin H are:
A.
Nuclear fragments left in the developing erythroid cells
B.
Neutrophil cytoplasmic light blue bodies of endoplasmic reticulum
C.
Small diffuse red cell inclusions, composed of RNA
D
Golf ball appearing red cells seen in supravitally stained smears
E.
Bilobed mature neutrophils
Q816. Howell-Jolly bodies are:
A
Nuclear fragments left in the developing erythroid cells
B.
Neutrophil cytoplasmic light blue bodies of endoplasmic reticulum
C.
Small diffuse red cell inclusions, composed of RNA
D.
Golf ball appearing red cells seen in supravitally stained smears
E.
Bilobed mature neutrophils
Q817. CD5 is positive in:
A.
B-CLL
B.
T-NHL
C.
Mantle Cell Lymphoma
D
All of the above
E.
None of the above
117
Professor Anwar Sheikha, MD, FRCP, FRCPath., FACP
One Thousand Hematology MCQ
Q818. CD23 is positive in:
A
B-CLL
“Chronic Lymphocytic Leukemia”
B.
B-PLL
“Prolymphocytic Leukemia”
C.
Mantle Cell Lymphoma
D.
Follicular Lymphoma
E.
HCL “Hairy Cell Leukemia”
Q819. SmIg “Surface Membrane Immunoglobulin” is brightly positive in all
but one of the following blood diseases:
A
B-CLL
“Chronic Lymphocytic Leukemia”
B.
B-PLL
“Prolymphocytic Leukemia”
C.
Mantle Cell Lymphoma
D.
Follicular Lymphoma
E.
HCL “Hairy Cell Leukemia”
Q820. CD11c is positive in:
A.
B-CLL
“Chronic Lymphocytic Leukemia”
B.
B-PLL
“Prolymphocytic Leukemia”
C.
Mantle Cell Lymphoma
D.
Follicular Lymphoma
E
HCL “Hairy Cell Leukemia”
Q821. Bone marrow examination is an absolute requirement for the diagnosis of :
A.
CLL “Chronic Lymphocytic Leukemia”
B.
CML “Chronic Myeloid Leukemia”
C.
PRV “Polycythemia Rubra Vera”
D
AA
“Aplastic Anemia”
E.
AIHA “Autoimmune Hemolytic Anemia”
Q822. Bone marrow examination is not an absolute requirement for the diagnosis of
one of the following diseases:
A.
RARS “Refractory Anemia with Ring Sideroblasts”
B.
PRCA “Pure Red Cell Aplasia”
C.
RAEB “Refractory Anemia with Excess Blasts”
D
Lead poisoning
E.
Aplastic crisis of sickle cell disease
Q823. Bone marrow examination is essential in diagnosing all but one of the
followings diseases:
A.
Megaloblastic anemia
B.
Gaucher’s disease
C.
Niemann-Pick disease
D
Hereditary spherocytosis
E.
Myelodysplastic syndromes
Q824. Bone marrow examination is not essential for the diagnosis of :
A.
Multiple Myeloma
B.
Myelofibrosis
C
Secondary Polycythemia
D.
Staging of Non-Hodgkin Lymphoma
E.
Visceral Leishmaniasis
118
Professor Anwar Sheikha, MD, FRCP, FRCPath., FACP
One Thousand Hematology MCQ
Q825. Bone marrow is helpful in establishing the diagnosis of all but one of the
following conditions:
A
Thalassemia major
B.
ITP “Immune Thrombocytopenic Purpura”
C.
HCL “Hairy Cell Leukemia”
D.
Hypersplenism
E.
Waldenström’s macroglobulinemia
Q826. Bone marrow is informative and usually indicated for the diagnosis
of all but one of the following conditions:
A.
5q- Syndrome
B.
Essential thrombocythemia
C.
Leukoerythroblastic anemia
D.
HHH “Histiocytic Hyperplasia with Hemophagocytosis”
E
DIC “Disseminated Intravascular Coagulopathy”
Q827. One of the following diseases is not known to be induced by radiation, chemicals
or alkylating agents:
A
CLL “Chronic Lymphocytic Leukemia”
B.
AML “Acute Myeloid Leukemia”
C.
MDS “Myelodysplastic Syndromes”
D.
CML “Chronic Myeloid Leukemia”
E.
Aplastic Anemia
Q828. All but one of the following complications is expected in
Chronic Lymphocytic Leukemia:
A.
ITP “Immune Thrombocytopenic Purpura”
B.
AIHA “Autoimmune Hemolytic Anemia”
C
Priapism
D.
Varicella Zoster
E.
Hypogammaglobulinemia
Q829. One of the followings is a rare but well-known complication of
Chronic Myeloid Leukemia:
A.
ITP “Immune Thrombocytopenic Purpura”
B.
AIHA “Autoimmune Hemolytic Anemia”
C
Priapism
D.
Varicella Zoster
E.
Hypogammaglobulinemia
Q830. All but one of the following features are more in favor of the diagnosis of
B- Prolymphocytic Leukemia rather than B- Chronic Lymphocytic Leukemia:
A.
Extreme lymphocytosis
B.
Prominent splenomegaly
C.
Aggressive behavior
D
Strong female predominance
E.
Refractory to treatment
119
Professor Anwar Sheikha, MD, FRCP, FRCPath., FACP
One Thousand Hematology MCQ
Q831. All but one of the following features are more in favor of the diagnosis of
B- Prolymphocytic Leukemia rather than B- Chronic Lymphocytic Leukemia:
A
Remarkable lymphadenopathy
B.
Positive CD22
C.
High density SmIg
D.
Strong male predominance
E.
Presence of lymphocytes with prominent vesicular central nucleoli
Q832. All but one of the following features are more in favor of the diagnosis of
B- Chronic Lymphocytic Leukemia rather than B- Prolymphocytic Leukemia:
A.
Morphologically more mature looking lymphocytes
B
Immunologically more mature lymphocytes
C.
Positive CD5
D.
Positive CD23
E.
Remarkable lymphadenopathy
Q833. The following anticoagulants can be used in collecting blood samples:
A.
EDTA (Ethylene diamine tetra acetic acid)
B.
Heparin
C.
Citrate
D
All of the above
E.
None of the above
Q834. Management of Chronic Myeloid Leukemia could involve all
but one of the followings:
A
Steroid
B.
Leukopheresis
C.
Imatinib mesylate “Glivec”
D.
Interferon
E.
Hydroxyurea
Q835. All but one of the followings could be used in managing
Chronic Lymphocytic Leukemia:
A.
Steroid
B.
Chlorambucil
C
Imatinib mesylate “Glivec”
D.
Fludarabine
E.
Immunoglobulin
Q836. Hairy cell leukemia could be initially treated by any of the following measures
EXCEPT:
A
Bone marrow or peripheral stem cell transplantation
B.
Cladribine “2-Chlorodeoxyadenosine”
C.
Pentostatin “2-Deoxycoforomycin”
D.
Splenectomy
E.
Interferon
120
Professor Anwar Sheikha, MD, FRCP, FRCPath., FACP
One Thousand Hematology MCQ
Q837. One of the followings is not used in the management of
Chronic Myeloid Leukemia:
A.
Busulphan
B.
Hydroxyurea
C.
Bone marrow or peripheral stem cell transplantation
D
Fludarabine
E.
Imatinib mesylate “Glivec”
Q838. All of the followings could be used in managing Multiple Myeloma EXCEPT:
A
Imatinib mesylate “Glivec”
B.
Plasmapheresis
C.
Zoledronate
D.
Lenalidomide “Revlimid”
E.
Melphalan
Q839. Wait and watch policy could be adopted in managing all but one of the following
hematological malignancies:
A.
Chronic Lymphocytic Leukemia
B.
Multiple myeloma
C.
Small Lymphocytic Lymphoma
D
Burkitt lymphoma
E.
Follicular lymphoma
Q840. Splenectomy is not advisable in one of the following diseases:
A.
Hereditary spherocytosis
B.
Hereditary elliptocytosis
C
Hereditary stomatocytosis
D.
Hairy cell leukemia
E.
Immune thrombocytopenic purpura
Q841. One of the following diseases is not common in the black population:
A
Hereditary spherocytosis
B.
Multiple myeloma
C.
Sickle cell disease
D.
HIV infection
E.
Endemic Burkitt lymphoma
Q842. Gemtuzumab Ozogamicin “Myelotarg” is a novel therapeutic agent effective in
the management of relapsed and refractory:
A.
Chronic Lymphocytic Leukemia
B.
Multiple Myeloma
C
Acute Myeloid Leukemia
D.
Thalassemia iron overload
E.
Paroxysmal nocturnal hemoglobinuria
Q843. Alemtuzumab “Campath” is a novel therapeutic agent effective in the
management of relapsed and refractory:
A
Chronic Lymphocytic Leukemia
B.
Multiple Myeloma
C.
Acute Myeloid Leukemia
D.
Thalassemia iron overload
121
Professor Anwar Sheikha, MD, FRCP, FRCPath., FACP
One Thousand Hematology MCQ
E.
Paroxysmal nocturnal hemoglobinuria
Q844. Bortezomib “Velcade” is a novel therapeutic agent with proteosome inhibiting
activity effective in the management of relapsed and refractory:
A.
Chronic Lymphocytic Leukemia
B
Multiple Myeloma
C.
Acute Myeloid Leukemia
D.
Thalassemia iron overload
E.
Paroxysmal nocturnal hemoglobinuria
Q845. Deferasirox “Exjade” is a new orally effective drug with promising effect in the
management of patients with:
A.
Glucose 6 Phosphate Dehydrogenase deficiency
B.
Polycythemia Rubra Vera
C.
Secondary polycythemia
D
Transfusion dependent thalassemia
E.
Paroxysmal nocturnal hemoglobinuria
Q846. Eculizumab is a novel therapeutic agent with unique effect in patients with:
A.
Chronic Lymphocytic Leukemia
B.
Multiple Myeloma
C.
Acute Myeloid Leukemia
D.
Thalassemia iron overload
E
Paroxysmal nocturnal hemoglobinuria
Q847. All but one of the following novel agents are monoclonal antibosied used in the
management of different types of neoplastic disorders:
A
Bortezomib “Velcade”
B.
Rituximab “MabThera”
C.
Alemtuzumab “Campath”
D.
Trastuzumab “Herceptin”
E.
Gemtuzumab “Myelotarg”
Q848. One of the following hemoglobin electrophoretic patterns in a 10 year old Afghani
patient matches with the diagnosis of HbE trait.
HbA%
HbF% HbA2% HbS%
HbE%
HbC%
HbH%
00
98
2
00
00
00
00
A
65
00
00
35
00
00
00
B
00
00
00
100
00
00
00
C
74
00
00
00
26
00
00
D
58
00
00
00
42
00
00
E.
Q849. One of the following hemoglobin electrophoretic patterns in a 10 year old
Bangladeshi patient matches with the diagnosis of HbE trait with concomitant αthalassemia trait.
HbA%
HbF% HbA2% HbS%
HbE%
HbC%
HbH%
00
98
2
00
00
00
00
A
65
00
00
35
00
00
00
B
00
00
00
100
00
00
00
C
74
00
00
00
26
00
00
D.
122
Professor Anwar Sheikha, MD, FRCP, FRCPath., FACP
E
58
00
00
One Thousand Hematology MCQ
00
42
00
00
Q850. One of the following hemoglobin electrophoretic patterns in a newly diagnosed
severely anemic Kurdish patient with hemoglobinopathy matches with the diagnosis of
β- thalassemia major.
HbA%
HbF%
HbA2%
HbS%
HbE%
HbC%
HbH%
00
98
2
00
00
00
00
A.
65
00
00
35
00
00
00
B
00
00
00
100
00
00
00
C
74
00
00
00
26
00
00
D
58
00
00
00
42
00
00
E
Q851. One of the following hemoglobin electrophoretic patterns in a 10 year old Black
African patient admitted with severe vaso-occlusive painful crisis matches with the
diagnosis of sickle cell disease.
HbA%
HbF% HbA2% HbS%
HbE%
HbC%
HbH%
00
98
2
00
00
00
00
A
65
00
00
35
00
00
00
B
00
00
00
100
00
00
00
C.
74
00
00
00
26
00
00
D
58
00
00
00
42
00
00
E
Q852. One of the following hemoglobin electrophoretic patterns in a Saudi patient from
the Eastern Provinces of Saudi Arabia matches with inheritance of a sickle cell gene
and concomitant α–chain defect.
HbA%
HbF%
HbA2%
HbS%
HbE%
HbC%
HbH%
00
98
2
00
00
00
00
A
65
00
00
35
00
00
00
B.
00
00
00
100
00
00
00
C
74
00
00
00
26
00
00
D
58
00
00
00
42
00
00
E
Q853. In the new W.H.O. classification of hematological malignancies, cytogenetic
abnormalities have a strong prognostic influence. Match one of the following
karyotypic changes with Acute Myeloid Leukemia with maturation.
A.
t(8;14)
B.
t(1;22)
C.
t(15;17)
D.
inv(16)
E
t(8;21)
Q854. In the new W.H.O. classification of hematological malignancies, cytogenetic
abnormalities have a strong prognostic influence. Match one of the following
karyotypic changes with hypogranular Acute Promyelocytic Leukemia.
A.
t(8;14)
B.
t(1;22)
C
t(15;17)
D.
inv (16)
123
Professor Anwar Sheikha, MD, FRCP, FRCPath., FACP
One Thousand Hematology MCQ
E.
t(8;21)
Q855. In the new W.H.O. classification of hematological malignancies, cytogenetic
abnormalities have a strong prognostic influence. Match one of the following
karyotypic changes with Acute Myelomonoblastic Leukemia with Eosinophilia.
A.
t(8;14)
B.
t(1;22)
C.
t(15;17)
D
inv(16)
E.
t(8;21)
Q856. In the new W.H.O. classification of hematological malignancies, cytogenetic
abnormalities have a strong prognostic influence. Match one of the following
karyotypic changes with Burkitt lymphoma.
A
t(8;14)
B.
t(1;22)
C.
t(15;17)
D.
inv16
E.
t(8;21)
Q857. In the new W.H.O. classification of hematological malignancies, cytogenetic
abnormalities have a strong prognostic influence. Match one of the following
karyotypic changes with Acute Megakaryoblastic Leukemia of infancy.
A.
t(8;14)
B
t(1;22)
C.
t(15;17)
D.
inv16
E.
t(8;21)
Q858. In the new W.H.O. classification of hematological malignancies, cytogenetic
abnormalities have a strong prognostic influence. Which one of the following
karyotypic changes has a poor prognosis?
A.
5q- syndrome
B
t(1;22)
C.
t(15;17)
D.
inv(16)
E.
t(8;21)
Q859. In multiply transfused patients, filters could reduce the incidence and intensity of:
A.
Immediate hemolytic transfusion reaction
B.
Delayed hemolytic transfusion reaction
C
Febrile Non-Hemolytic Transfusion Reaction
D.
Red cell alloimmunization
E.
Anaphylactic reactions
Q860. One of these blood products is best be stored and stirred at room temperature
before transfusion:
A.
Packed red blood cells
B.
Granulocytes
C
Platelet concentrate
D.
Fresh Frozen Plasma
E.
Cryoprecipitate
124
Professor Anwar Sheikha, MD, FRCP, FRCPath., FACP
One Thousand Hematology MCQ
Q861. In a lady with immune thrombocytopenic purpura, steroid should be started:
A.
Once the platelet count drops below150K/uL
B.
When the platelet count drops below100K/uL even without bleeding
C.
Anytime platelet count drops below 50/uL
D
When the patient bleeds especially if platelet is <20K/uL
E.
When platelet count <10K/uL with blood smear showing platelet aggregate
Q862. Budd-Chiari syndrome could be a complication of:
A.
PNH “Paroxysmal Nocturnal Hemoglobinuria”
B.
E.T. “Essential Thrombocythemia”
C.
PRV “Polycythemia Rubra Vera”
D
All of the above
E.
None of the above
Q863. Decision regarding treatment of patients with immune thrombocytopenic purpura
should take all but one of these features into consideration:
A.
Platelet count
B.
Presence or absence of bleeding
C.
Life style of the patient
D
Degree of marrow megakaryocyte increase
E.
Plateletcrit
Q864. One of these inherited hematological conditions is not sex-linked:
A
von Willebrand Disease
B.
Hemophilia A
C.
Christmas disease
D.
Sideroblastic anemia
E.
Wiskott-Aldrich syndrome
Q865. A reliable clue to the possibility of immediate hemolytic transfusion reaction
following a mismatched ABO incompatible transfusion is:
A.
Jaundice
B.
Fever
C
Red urine
D.
Hypertension
E.
Splenomegaly
Q866. Total hip replacement is a highly thrombogenic event. Without proper
prophylactic anticoagulation, the incidence of Deep Vein Thrombosis could be:
A.
100%
B
50%
C.
10%
D.
5%
E.
None
Q867. An acute severe anemia in a 10 year old child with sickle cells disease and
reticulocytopenia is most probably due to:
A.
Acute splenic sequestration crisis
B.
Acute painful vaso-occlusive crisis
C.
Hemolytic crisis
D
Aplastic crisis
125
Professor Anwar Sheikha, MD, FRCP, FRCPath., FACP
E.
One Thousand Hematology MCQ
Megaloblastic crisis
Q868. An acute severe anemia associated with jaundice in a 10 year old child with
sickle cells disease and reticulocytosis is most probably due to:
A.
Acute splenic sequestration crisis
B.
Acute painful vaso-occlusive crisis
C
Hemolytic crisis
D.
Aplastic crisis
E.
Megaloblastic crisis
Q869. An acute severe anemia in a one year old child with sickle cells disease
associated with dramatic splenic enlargement is most probably due to:
A
Acute splenic sequestration crisis
B.
Acute painful vaso-occlusive crisis
C.
Hemolytic crisis
D.
Aplastic crisis
E.
Megaloblastic crisis
Q870. One of the following features does not relate to the aplastic crisis associated with
sickle cell disease:
A.
Acute severe anemia
B.
Recent history of Parvovirus B19 infection
C
Pancytopenia
D.
Marrow pure red cell aplasia
E.
Very low reticulocyte count
Q871. An acute hemolytic anemia associated with progressive jaundice in a female
black neonate could be due to:
A.
G6PDH deficiency
B.
β- thalassemia major
C.
Sickle cell disease
D.
All of the above
E
None of the above
Q872. One of the following facts is not true about M-GUS “Monoclonal Gammopathy of
Undetermined Significance”
A.
Almost a quarter of patients will eventually develop myeloma
B.
Could be seen in 1% of people above 50 years of age
C.
Could be seen in 3% of people above 70 years of age.
D
All patients should be treated to prevent transformation to myeloma
E.
Approximately two-thirds of patients never develop a malignant disorder
Q873. All but one of the following criteria is not part of Antiphospholipid Syndrome:
A
Three abortions before 10 weeks of gestation
B.
One or more abortions after 10 weeks with normal fetal features
C.
Any child loss prior to 34th week associated with pre-eclampsia
D.
Presence of lupus anticoagulant
E.
Significantly positive anticardiolipin antibodies
126
Professor Anwar Sheikha, MD, FRCP, FRCPath., FACP
One Thousand Hematology MCQ
Q874. The normal absolute reticulocyte count is:
A.
200 to 2,000/uL
B.
4,000 to 10,000/uL
C
10,000 to 100,000/uL
D.
100,000 to 150,000/uL
E.
150,000 to 400,000/uL
Q875. Directed therapy with monoclonal antibodies effective against malignant cells
positive for CD33 could be specifically achieved by one of the following agents:
A
Gemtuzumab ozagamicin “Myelotarg”
B.
Eculizumab
C.
Alemtuzumab
“Campath”
D.
Rituximab
“MabThera”
E.
Trastuzumab “Herceptin”
Q876. Directed therapy with monoclonal antibodies effective against cells positive for
CD59 could be specifically achieved by one of the following agents:
A.
Gemtuzumab ozagamicin “Myelotarg”
B
Eculuzumab
C.
Alemtuzumab
“Campath”
D.
Rituximab
“MabThera”
E.
Trastuzumab “Herceptin”
Q877. Directed therapy with monoclonal antibodies effective against malignant cells
positive for HER2 family of antigens could be achieved by one of the followings:
A.
Gemtuzumab ozagamicin “Myelotarg”
B.
Eculuzumab
C.
Alemtuzumab
“Campath”
D.
Rituximab
“MabThera”
E
Trastuzumab “Herceptin”
Q879. Directed therapy with monoclonal antibodies effective against malignant cells
positive for CD20 could be specifically achieved by one of the following agents:
A.
Gemtuzumab ozagamicin “Myelotarg”
B.
Eculuzumab
C.
Alemtuzumab
“Campath”
D
Rituximab
“MabThera”
E.
Trastuzumab “Herceptin”
Q880. Directed therapy with monoclonal antibodies effective against malignant cells
positive for CD52 could be specifically achieved by one of the following agents:
A.
Gemtuzumab ozagamicin “Myelotarg”
B.
Eculuzumab
C
Alemtuzumab
“Campath”
D.
Rituximab
“MabThera”
E.
Trastuzumab “Herceptin”
127
Professor Anwar Sheikha, MD, FRCP, FRCPath., FACP
One Thousand Hematology MCQ
Q881. In a normal peripheral blood film, the proportion of small to large lymphocytes is:
A
90:10
B.
80:20
C.
70:30
D.
60:40
E.
50:50
Q882. In blood, the proportion of T-Helper to T-Suppressor lymphocytes is:
A.
10:01
B.
05:01
C
02:01
D.
01:01
E.
01:02
Q883. The life-span of mature memory lymphocytes/ red blood cells/platelet/ t 1/2
neutrophils could be expressed one of the followings:
A
Years/ months/ days/ hours
B.
Months/ days/ hours/ hours
C.
Days/ days/ days/ days
D.
months/ months/ months/ days
E.
days/ days/ days/ days
Q884. In a normal blood sample with 5 million red cells/uL; platelet count of 250K/uL &
a white cell count of 5K/uL, the proportion of the cells in the blood smear will be:
A.
1000/ 05/ 01
B.
100/ 50/ 01
C.
10/ 05/ 01
D
1000/ 50/ 01
E.
1000/ 500/ 01
Q885. The proportion of T to B lymphocytes in the blood is:
A.
50:50
B.
20:80
C
80:20
D.
60:40
E.
40:60
Q886. One of the followings cannot be a function for T- lymphocytes:
A
Active killing of pyogenic bacteria
B.
Medication of cellular immunity against viruses, fungi & tuberculosis
C.
Participation in delayed hypersensitivity reactions & graft rejection
D.
Helping B- lymphocytes to produce antibodies
E.
Suppression of B- lymphocytes
Q887. All of the following procedures are correct about the therapeutic use of
unfractionated heparin in thromboembolic incidents EXCEPT:
A.
A bolus of 5,000 units should be given initially
B
It could be given intravenously, intramuscularly or subcutaneously
C.
The usual i.v. infusion dose is around 30,000 units over 24 hours
D.
The total daily dose could be given subcutaneously in two divided doses
E.
The degree of anticoagulation could be regulated by keeping APTT
128
Professor Anwar Sheikha, MD, FRCP, FRCPath., FACP
One Thousand Hematology MCQ
ratio between 1.5 and 2.5 normal value
Q888. One of the followings is correct about prophylactic use of unfractionated heparin:
A.
It should be monitored by APTT
B.
It could be given intravenously, intramuscularly or subcutaneously
C
The usual dose is 5,000 units subcutaneously twice daily
D.
It has its impact on thrombin rather than the early activated factors
E.
To avoid bleeding in surgical patients, it should be given postoperatively
Q889. All but one of the following facts relates to Low Molecular Weight Heparin:
A.
Because of its predictable action, it does not usually need monitoring
B.
It is given through the subcutaneous route
C.
It could be used prophylactically or therapeutically
D
It could be given intravenously
E.
Associated with less chance of Heparin Induced Thrombocytopenia
Q890. All but one of the followings are known complications of heparin therapy:
A.
Osteoporosis
B.
Bleeding from over-anticoagulation
C.
Thrombosis from Heparin Induced Thrombocytopenia “HIT”
D
Immune thrombocytopenic Purpura “ITP”
E.
Alopecia
Q891. All but one of the following facts relate to Warfarin:
A.
It is a rat poison
B
It could be given orally or parenterally
C.
It should overlap with heparin for many days
D.
It is best monitored by INR “International Normalized Ratio”
E.
It functions through paralyzing Factors II, VII, IX & X.
Q892. All but one of the following agents could be used in the management of von
Willebrand disease:
A
High purity Factor VIII concentrate
B.
Fresh Frozen Plasma
C.
DDAVP
D.
Tranexamic acid
E.
Very high purity VWF concentrate if available
Q893. Autologous hematopoietic stem cell transplantation could be the treatment of
choice in:
A.
Thalassemia major
B.
Sickle cell disease
C.
Aplastic anemia
D
Multiple myeloma
E.
Chronic myeloid leukemia
Q894. Allogeneic hematopoietic stem cell transplantation could be used as an initial
curative therapeutic measure in all but one of the following blood diseases:
A.
Thalassemia major
B.
Severe symptomatic sickle cell disease
C.
Aplastic anemia
D.
Multiple myeloma
129
Professor Anwar Sheikha, MD, FRCP, FRCPath., FACP
E
One Thousand Hematology MCQ
Chronic myeloid leukemia
Q895. Chronic Graft versus Host disease could be a debilitating complication of
hematopoietic stem cell transplantations. Features could include any of the
followings:
A.
Dry eyes
B.
Chronic liver disease
C
Diabetes mellitus
D.
Increased risk of infection
E.
Scleroderma like manifestations
Q896. Acute Graft versus Host disease could be a serious complication of
hematopoietic stem cell transplantations. Features could include any of the followings
A.
Acute liver damage
B.
Acute skin damage
C.
Acute intestinal damage
D.
Rising liver function tests
E
Xerostomia
Q897. All but one of the following therapeutic measures could benefit patients overanticoagulated with Warfarin:
A.
Vitamin K
B.
Fresh Frozen Plasma
C.
Simple discontinuation of the Warfarin
D
Protamine sulfate
E.
Physically cautious life style
Q898. One of these congenital defects is not associated with thrombophilia:
A.
Protein C deficiency
B.
Prothrombin G20210A mutation
C.
Factor V Leiden
D.
Factor XII deficiency
E
Factor XIII deficiency
Q899. The most curative management for chronic myeloid leukemia is:
A.
ABVD “Adriamycin, Bleomycin, Vinblastine and Dacarbazine”
B.
Fludarabine, Cyclophosphamide and Rituximab
C.
Imatinib mesylate
D
Hematopoietic peripheral stem cell transplantation
E.
R-CHOP “Rituximab, Cyclophosphamide, Adriamycin, Oncovin &
Prednisolone”
Q900. Graft versus Leukemia effect in hematopoietic stem cell transplantation:
A.
Is related to the B- lymphocyte content of the infused cells
B.
Could be enhanced by depleting T cells from the harvested sample
C.
It should be quite severe to have beneficial effect
D.
Could be suppressed by donor lymphocyte infusion
E
Its presence is associated with a higher cure rate
130
Professor Anwar Sheikha, MD, FRCP, FRCPath., FACP
One Thousand Hematology MCQ
Q901. In automated hematology cell analyzers, spurious high hemoglobin reading could
be due to all but one of the following situations:
A.
Hypercholesterolemia
B.
Paraproteinemia
C
Hyperglycemia
D.
Increased sample turbidity
E.
Incomplete red cell lysis
Q902. In automated hematology cell analyzers, hyperlipidemia could falsely increase
the value of:
A.
Red cell count
B.
White cell count
C
Hemoglobin
D.
MCV “Mean Cell Volume”
E.
MPV “Mean Platelet Volume”
Q903. One of the following features is the least likely complication of ABO incompatible
blood transfusion:
A
High fever with chills and rigors
B.
Loin pain
C.
Chest tightness
D.
Hypotension
E.
Hemoglobinemia
Q904. All but one of the following treatment modalities could be used in the
management of aplastic anemia:
A.
Hematopoietic stem cell transplantation
B.
Steroid
C.
Cyclosporine
D.
Antilymphocyte globulin
E.
Platelet and red cell support
Q905. One of the following options should be given priority in the management of an
attack of Cold Hemagglutinin Disease “CHAD”:
A.
Steroid
B.
Intravenous immunoglobulin
C.
Splenectomy
D.
Rituximab
E
Plasma exchange using a blood warmer
Q906. One of the following hemoglobin electrophoretic patterns belongs to a 40 year old
Black African patient admitted following a road traffic accident and scheduled for
surgery. He was diagnosed in the hematology laboratory as sickle cell trait.
HbA%
HbF% HbA2% HbS%
HbE%
HbC%
HbH%
80%
18%
2%
00
00
00
00
A
94%
1%
5%
00
00
00
00
B
58%
2%
00
40%
00
00
00
C.
00
00
2%
52%
00
46%
00
D
84
00
00
00
00
00
16
E
131
Professor Anwar Sheikha, MD, FRCP, FRCPath., FACP
One Thousand Hematology MCQ
Q907. One of the following hemoglobin electrophoretic patterns belongs to a 32 year old
male Kurdish patient who had immigrated to Sweden and came back to Kurdistan for an
arranged marriage. The test was performed as part of a premarital obligatory screening
program. The new regulations strongly advised against that marriage and the lady,
being a newly graduate medical intern and aware of having β- thalassemia minor
herself, appreciated the situation and terminated the engagement.
HbA%
HbF% HbA2% HbS%
HbE%
HbC%
HbH%
80%
18%
2%
00
00
00
00
A
94%
1%
5%
00
00
00
00
B.
58%
2%
00
40%
00
00
00
C
00
00
2%
52%
00
46%
00
D
84
00
00
00
00
00
16
E
Q908. One of the following hemoglobin electrophoretic patterns belongs to a 20 year old
Kurdish patients with moderate anemia and splenomegaly. He has never been
transfused. He was told to have mild form of thalassemia that did not need transfusion
at the time.
HbA%
HbF% HbA2% HbS%
HbE%
HbC%
HbH%
80%
18%
2%
00
00
00
00
A.
94%
1%
5%
00
00
00
00
B
58%
2%
00
40%
00
00
00
C
00
00
2%
52%
00
46%
00
D
84
00
00
00
00
00
16
E
Q909. One of the following hemoglobin electrophoretic patterns belongs to a visiting
professor from Ghana . He is anemic and knows that he has Hemoglobin SC disease.
HbA%
HbF% HbA2% HbS%
HbE%
HbC%
HbH%
80%
18%
2%
00
00
00
00
A
94%
1%
5%
00
00
00
00
B
58%
2%
00
40%
00
00
00
C
00
00
2%
52%
00
46%
00
D.
84
00
00
00
00
00
16
E
Q910. One of the following hemoglobin electrophoretic patterns belongs to an anemic
patient who was told to have Hemoglobin H disease. He has splenomegaly and blood
smear showed Golf ball like red cells.
HbA%
HbF% HbA2% HbS%
HbE%
HbC%
HbH%
80%
18%
2%
00
00
00
00
A
94%
1%
5%
00
00
00
00
B
58%
2%
00
40%
00
00
00
C
00
00
2%
52%
00
46%
00
D
84
00
00
00
00
00
16
E.
132
Professor Anwar Sheikha, MD, FRCP, FRCPath., FACP
One Thousand Hematology MCQ
Q911. One of the following features is a characteristic complication of blood transfusion
due to Febrile Non Hemolytic Transfusion Reaction:
A
High fever with chills and rigors
B.
Loin pain
C.
Chest tightness
D.
Hypotension
E.
Hemoglobinemia
Q912. All but one of the following options could be considered in the management of an
acute attack of Warm Autoimmune Hemolytic Anemia “AIHA”:
A.
Steroid
B
Iron supplementation
C.
Splenectomy
D.
Rituximab
E.
Transfusion of the least incompatible blood
Q913. The most practical immediate management of a 2 month old male child with postcircumcision bleeding is:
A.
Factor VIII concentrate
B.
Factor IX concentrate
C.
Factor rVIIa “Novo7”
D.
Cryoprecipitate
E
Fresh Frozen Plasma
Q914. A practical hemostatic support for a patient with hemorrhagic intracranial bleed
could include:
A.
Factor VIII concentrate
B.
Factor IX concentrate
C
Factor rVIIa “Novo7”
D.
Cryoprecipitate
E.
Fresh Frozen Plasma
Q915. The initial management of a symptomatic patient with multiple myeloma who is
likely to be considered for a later date autotransplantation should not include:
A.
Thalidomide
B.
Lenalidomide
C
Melphalan
D.
Dexamethasone
E.
Vincrisitne
Q916. The ideal management of a patient with solitary plasmacytoma of a vertebra is:
A.
Wait, watch and worry!
B.
Melphalan and Prednisolone
C.
Thalidomide
D
Radiation
E.
Zoledronic acid
133
Professor Anwar Sheikha, MD, FRCP, FRCPath., FACP
One Thousand Hematology MCQ
Q917. One of the followings is not part of the POEMS syndrome:
A.
Polyneuropathy
B.
Organomegaly
C.
Endocrinopathy
D.
Myeloma
E
Sepsis
Q918. One of these paraproteinemais is associated with the shortest survival:
A
Multiple myeloma
B.
Indolent myeloma
C.
Waldenström macroglobulinemia
D.
SOP “Solitary Plasmacytoma of Bone”
E.
M-GUS “Monoclonal Gammopathy of Undetermined Significance”
Q919. To diagnose multiple myeloma, a major and a minor criteria is required. All but
one of the followings are regarded as major criteria:
A.
IgG Paraproteinemia of more than 35 gm/L
B.
IgA Paraprotein of more than 20 gm/L
C
IgM Paraproteinemia of more than 5 gm/L
D.
Marrow plasma cell of more than 30%
E.
Tissue plasmacytoma
Q920. All but one of the following features are regarded as favorable prognostic factors
in patients with multiple myeloma:
A.
β2- microglobulin of <2.5 mg/L
B.
C- reactive protein of <4.0 mg/dL
C
Plasma cell labeling index of >1%
D.
Long duration of initial plateau phase
E.
Chemotherapy sensitive disease
Q921. The least likely option in the management of Immune Thrombocytopenic Purpura
“ITP” is:
A
Platelet transfusion
B.
Intravenous immunoglobulin
C.
Splenectomy
D.
Rituximab
E.
Steroid
Q922. Spontaneous abortion is a known complication of one of these thrombophilias:
A.
Factor V Leiden heterozygous inheritance
B.
Protein S deficiency
C.
Protein C deficiency
D
Antiphospholipid syndrome
E.
Homocystinuria
Q923. One of these products is a novel hemostatic agent that can be used with benefit
in the management of patients with intracranial bleed.
A.
Platelet concentrate
B.
Factor VIII, XIII, vWF & Fibrinogen
C
Factor rVIIa “Novo7”
D.
Factor VIII concentrate
134
Professor Anwar Sheikha, MD, FRCP, FRCPath., FACP
E.
One Thousand Hematology MCQ
Most clotting factors including F V & XI that lack specific replacement
Q924. One of these blood products is essential in the management of leukemia patients
subjected to intensive induction regimens:
A
Platelet concentrate
B.
Factor VIII, XIII, vWF & Fibrinogen
C.
Factor rVIIa “Novo7”
D.
Factor VIII concentrate
E.
Most clotting factors including F V & XI that lack specific replacement
Q925. Fresh Frozen Plasma “FFP” is a practical choice to treat bleeding patients when
the diagnosis is not established yet because it contains:
A.
Platelet concentrate
B.
Factor VIII, XIII, vWF & Fibrinogen
C.
Factor rVIIa “Novo7”
D.
Factor VIII concentrate
E
Most clotting factors including FV & XI that lack specific replacement
Q926. Cryoprecipitate is a rich source of:
A.
Platelet concentrate
B
Factor VIII, XIII, vWF & Fibrinogen
C.
Factor rVIIa “Novo7”
D.
Factor VIII concentrate
E.
Most clotting factors including F V & XI that lack specific replacement
Q927. One of these agents is a good source of a novel hemostatic agent that can be
used with benefit in the management of intracranial bleed.
A.
Platelet concentrate
B.
Factor VIII, XIII, vWF & Fibrinogen
C
Factor rVIIa “Novo7”
D.
Factor VIII concentrate
E.
Most clotting factors including F V & XI that lack specific replacement
Q928. FEIBA “Factor VIII inhibitor Bypassing Activity” and Factor rVIIa “Novo7” are
effective in the management of:
A.
Hemophiliacs with massive bleeding
B.
Hemophiliacs with cerebral bleeding
C
Hemophiliacs with Factor VIII inhibitors
D.
von Willebrand patients needing dental clearance
E.
A factor XII deficient patient with cerebrovascular accident
Q929. All but one of the followings are requirements to diagnose smouldering myeloma:
A.
Marrow plasmacytosis <30%
B.
IgG paraprotein of <70 gm/L
C.
IgA paraprotein of <50 gm/L
D.
No or < 3 lytic bone lesions
E
Urine light chain >4 gm/24 hours
135
Professor Anwar Sheikha, MD, FRCP, FRCPath., FACP
One Thousand Hematology MCQ
Q930. All but one of the following criteria are required to make the diagnosis of M-GUS
“Monoclonal Gammopathy of Undetermined Significance”
A.
IgG of <35 gm/L
B.
IgA of <20 gm/L
C.
Urine light chain of < 1 gm/24 hours
D
Less than three lytic lesions
E.
Marrow plasmacytosis of <10%
Q931. The frequency of t(9;22) or what is known as Philadelphia chromosome in Adult
Acute Lymphoblastic Leukemia is:
A.
02-04%
B.
05-10%
C.
10-20%
D
20-25%
E.
25-50%
Q932. Warfarin affects all but one of the clotting factors:
A.
Factor II
B.
Factor VII
C.
Factor X
D.
Protein C
E
Factor V
Q933. The frequency of t(9;22) or what is known as Philadelphia chromosome in Acute
Lymphoblastic Leukemia in children is:
A
02-04%
B.
05-10%
C.
10-20%
D.
20-25%
E.
25-50%
Q934. A reasonable initial treatment for a 20 year old patient with aplastic anemia is:
A.
High dose cyclophosphamide
B
Hematopoietic peripheral stem cell transplantation
C.
Antithymocyte Globulin
D.
Antithymocyte Globulin and Cyclosporine
E.
Androgens
Q935. A reasonable initial treatment for a 60 year old patient with aplastic anemia is:
A.
High dose cyclophosphamide
B.
Hematopoietic peripheral stem cell transplantation
C.
Antithymocyte Globulin
D
Antithymocyte Globulin and Cyclosporine
E.
Androgens
Q936. Blood products support to aplastic anemia patients should be limited in order to:
A.
Prevent sensitization to transplant antigens
B.
Prevent sensitization to blood group antigens
C.
Prevent seroconversion in CMV-negative patients
D
All of the above
E.
None of the above
136
Professor Anwar Sheikha, MD, FRCP, FRCPath., FACP
One Thousand Hematology MCQ
Q937. All but one of the followings is true about blood transfusion support
to a freshly diagnosed aplastic anemia patient:
A.
Limited to necessity especially if patient is scheduled for transplantation
B.
Never be from family members, especially if transplant is sought
C
Should always be CMV-negative blood products
D.
Preferably be irradiated leukocyte-depleted blood products
E.
Febrile transfusion reaction is not a possibility because of pancytopenia
Q938. Rai and Binet staging systems are used to stage patients with chronic
lymphocytic leukemia. A patient presenting with a lymphocyte count of
100,000/uL; splenomegaly, cervical and axillary lymphadenopathy with a platelet
count of 50K/uL from extensive marrow infiltration by lymphocytes is staged as:
A.
IA “Rai Stage I and Binet stage A”
B.
IB “Rai Stage I and Binet Stage B”
C.
IIB “Rai Stage II and Binet Stage B”
D
IVC “Rai Stage III and Binet Stage C”
E.
0A “Rai Stage O and Binet Stage A”
Q939. The best pre-conditioning regimen for stem cell transplantation in multiply
transfused and sensitized patients with aplastic anemia is:
A.
Cyclophosphamide alone
B.
Busulphan and Cyclophosphamide
C.
Cyclophosphamide and Limited lymphoid irradiation
D
Cyclophosphamide and ATG
E.
Total Nodal irradiation
Q940. The best pre-conditioning regimen for stem cell transplantation in a fresh patient
with aplastic anemia is:
A
Cyclophosphamide alone
B.
Busulphan and Cyclophosphamide
C.
Cyclophosphamide and Limited lymphoid irradiation
D.
Cyclophosphamide and ATG
E.
Total Nodal irradiation
Q941. To diagnose multiple myeloma, a major and a minor criteria or three minor
criteria are required. All but one of the followings is regarded as minor criteria:
A.
Osteolytic bone lesions
B.
Marrow plasmacytosis 10-30%
C
Low normal immunoglobulins
D.
IgG paraprotein of less than 35gm/L or IgA less than 20 gm/L
E.
Plasmacytoma
Q942. A pregnant lady presenting with a platelet count of 80K/uL in the third trimester
of pregnancy could have any but one of the followings:
A.
Spurious thrombocytopenia
B.
Gestational thrombocytopenia
C.
Immune thrombocytopenia
D
Glanzmann’s disease
E.
HELLP syndrome
137
Professor Anwar Sheikha, MD, FRCP, FRCPath., FACP
One Thousand Hematology MCQ
Q943. Emergency management of a patient with immune thrombocytopenic purpura
presenting with a major bleed could include any but one of the followings:
A.
Platelet transfusion
B
Cyclophosphamide
C.
Methylprednisolone 1 gm/day for three days
D.
Intravenous immunoglobulin 1 gm/kg/day for two days
E.
Intravenous immunoglobulin immediately followed by platelet transfusion
Q944. Therapeutic modalities for von Willebrand disease could include any but one of
the following:
A.
DDAVP
B
Novo7
C.
Tranexamic acid
D.
Factor replacement “factor VIII/vWF Concentrate”
E
Topical agents
Q945. When managing a patient with aplastic anemia:
A.
Always give a chance for spontaneous recovery
B.
Liberally use Growth factors (Erythropoietin, G-CSF, GM-CSF)
C
Always start looking for a stem cell donor in transplantable patients
D.
Try androgens as it has been found to be effective in most patients
E.
Correct the anemia whenever the hemoglobin drops below 10 gm/dL
Q946. Megakaryocyte hyperplasia, clustering & nuclear hyperlobations are features of:
A.
5q- syndrome
B.
Myelodysplastic syndromes
C
Polycythemia rubra vera
D.
Immune thrombocytopenic purpura
E.
Aplastic anemia
Q947. In patients with polycythemia rubra vera one of the following management lines is
mandatory:
A
Venesections
B.
Aspirin
C.
Cytoreduction using hydroxyurea
D.
Allopurinol
E.
Iron supplementation when they become iron deficient
Q948. Patients with polycythemia rubra vera and thrombotic problems should avoid:
A.
smoking
B.
Hormone replacement therapy
C.
Contraceptive pills
D
All of the above
E.
None of the above
Q949. The cytoreductive agent of choice in a pregnant polycythemia vera patient is:
A
Interferon-α
B.
Hydroxyurea
C.
Anagrelide
D.
Busulphan
E.
Radioactive P32
138
Professor Anwar Sheikha, MD, FRCP, FRCPath., FACP
One Thousand Hematology MCQ
Q950. Ideal management of a hereditary spherocytosis patient who has frequent
symptomatic attacks of abdominal pain from gall stones is:
A.
Splenectomy
B.
Cholecystectomy
C
Combined splenectomy and cholecystectomy.
D.
Folic acid
E.
Pneumovax, Meningococcal & Hemophilus influenzae B vaccination
Q951. Rai and Binet staging systems are used to stage patients with chronic
lymphocytic leukemia. A patient presenting with a lymphocyte count of
100,000/uL, cervical and axillary lymphadenopathy is staged as:
A
IA “Rai Stage I and Binet stage A”
B.
IB “Rai Stage I and Binet Stage B”
C.
IIB “Rai Stage II and Binet Stage B”
D.
IIIC “Rai Stage III and Binet Stage C”
E.
0A “Rai Stage O and Binet Stage A”
Q952. Rai and Binet staging systems are used to stage patients with chronic
lymphocytic leukemia. A patient presenting with a lymphocyte count of
100,000/uL but no lymphadenopathy or organomegaly is staged as:
A.
IA “Rai Stage I and Binet stage A”
B.
IB “Rai Stage I and Binet Stage B”
C.
IIB “Rai Stage II and Binet Stage B”
D.
IIIC “Rai Stage III and Binet Stage C”
E
0A “Rai Stage O and Binet Stage A”
Q953. Rai and Binet staging systems are used to stage patients with chronic
lymphocytic leukemia. A patient presenting with a lymphocyte count of
100,000/uL, cervical, axillary and inguinal lymphadenopathy but no
organomegaly or cytopenia is staged as:
A.
IA “Rai Stage I and Binet stage A”
B
IB “Rai Stage I and Binet Stage B”
C.
IIB “Rai Stage II and Binet Stage B”
D.
IIIC “Rai Stage III and Binet Stage C”
E.
0A “Rai Stage O and Binet Stage A”
Q954. Rai and Binet staging systems are used to stage patients with chronic
lymphocytic leukemia. A patient presenting with a lymphocyte count of
100,000/uL, splenomegaly, cervical and axillary lymphadenopathy is staged as:
A.
IA “Rai Stage I and Binet stage A”
B.
IB “Rai Stage I and Binet Stage B”
C
IIB “Rai Stage II and Binet Stage B”
D.
IIIC “Rai Stage III and Binet Stage C”
E.
0A “Rai Stage O and Binet Stage A”
139
Professor Anwar Sheikha, MD, FRCP, FRCPath., FACP
One Thousand Hematology MCQ
Q955. Rai and Binet staging systems are used to stage patients with chronic
lymphocytic leukemia. A patient presenting with a lymphocyte count of
100,000/uL, splenomegaly, cervical and axillary lymphadenopathy with
hemoglobin of 80 gm/L and spherocytosis with positive Coomb’s test is staged
as:
A.
IA “Rai Stage I and Binet stage A”
B.
IB “Rai Stage I and Binet Stage B”
C
IIB “Rai Stage II and Binet Stage B”
D.
IIIC “Rai Stage III and Binet Stage C”
E.
0A “Rai Stage O and Binet Stage A”
Q956. Rai and Binet staging systems are used to stage patients with chronic
lymphocytic leukemia. A patient presenting with a lymphocyte count of
100,000/uL, splenomegaly, cervical and axillary lymphadenopathy with moderate
anemia from extensive marrow infiltration is staged as:
A.
IA “Rai Stage I and Binet stage A”
B.
IB “Rai Stage I and Binet Stage B”
C.
IIB “Rai Stage II and Binet Stage B”
D
IIIC “Rai Stage III and Binet Stage C”
E.
0A “Rai Stage O and Binet Stage A”
Q957. Mixed Lineage Leukemia “MLL” has all but one of the following features:
A.
Usually carries translocation of chromosome 11q23 gene
B
Typically CD10 positive
C.
Most commonly occurs in infancy
D.
Co-express both lymphoid and myeloid markers
E.
Usually associated with poor prognosis and early relapse
Q958. Treatment options for prolymphocytic leuekemia includes:
A.
Alemtuzumab
B.
Cladribine
C.
Fludarabine
D
Any of the above could be quite effective
E.
None of the above is effective
Q959. The current treatment of choice for a hairy cell leukemia patient presenting with
huge spleen and cytopenias is:
A.
Splenectomy
B.
Interferon-α
C.
Pentostatin
D
Cladribine
E.
Anti-tuberculosis medications
Q960. The initial treatment for a hairy cell leukemia patient presenting with pulmonary
tuberculosis, a huge spleen and cytopenias is:
A.
Splenectomy
B.
Interferon-α
C.
Pentostatin
D.
Cladribine
140
Professor Anwar Sheikha, MD, FRCP, FRCPath., FACP
E
One Thousand Hematology MCQ
Anti-tuberculosis medications
Q961. The ideal management for a 34 year old patient with diffuse large B- cell
lymphoma presenting with cervical lymphadenopathy and staged as CS IA is:
A.
Wait and watch policy
B
Local radiation therapy
C.
Rituximab
D.
R-CHOP
E.
Hematopoietic stem cell autotransplantation
Q962. Mucosa Associated Lymphoid Tissue “MALT” and other non-Hodgkin lymphomas
are well-known to be associated with bacterial infections. All but one of the
associations is true:
A
α- heavy chain disease and Helicobacter pylori infection
B.
Gastric MALT lymphoma and Helicobacter pylori infection
C.
Ocular adnexial MLALT lymphoma and Chlamydia psittaci infection
D.
Cutaneous MALT lymphoma and infection with Borrelia burgdorferi
E.
IPSID “Immunoproliferative Small Intestinal Disease” and
Campylobacter jejuni infection
Q963. The ideal management for a 74 year old patient presenting with relatively
asymptomatic stage IV indolent lymphoma is:
A
Wait and watch policy
B.
Local radiation therapy
C.
Rituximab
D.
R-CHOP
E.
Combined modality “Radiation and Chemotherapy” treatment
Q964. The ideal management for a 74 year old patient presenting with an indolent
lymphoma involving both cervical lymph nodes is:
A.
Wait and watch policy
B
Local radiation therapy
C.
Rituximab
D.
R-CHOP
E.
Combined modality “Radiation and Chemotherapy” treatment
Q965. One of the following approaches applies to the treatment of Essential
Thrombocythemia:
A.
Anagrelide is the initial current treatment of choice in all patients
B.
Aspirin should be avoided because of the inherent platelet dysfunction
C.
Surgery could be safe if patients do not show bleeding problems
D
In high-risk cases, hydroxyurea should be used to control platelet count
<600K/uL
E.
Because of the thromboembolic complications, every patient must be
treated once the diagnosis is made
Q966. All but one of the following criteria are essential in the diagnosis of Essential
Thrombocythemia:
A
Splenomegaly
B.
Presence of marrow iron stores or normal serum ferritin
C.
Absence of BCR-ABL fusion gene
D.
Sustained platelet count >600,000/uL
141
Professor Anwar Sheikha, MD, FRCP, FRCPath., FACP
E.
One Thousand Hematology MCQ
Absence of conditions associated with reactive thrombocytosis
Q967. All but one of the followings are true about Essential Thrombocythemia:
A.
It is the commonest myeloproliferative disorder in USA
B.
The median age is 60 years
C
As a myeloproliferative disorder, almost all cases have splenomegaly
D.
It is more common in female than male
E.
Thrombotic and hemorrhagic complications are well-known associations
Q968. The stages after Breaking Bad News to patients with cancer are usually:
A
Shock, denial, anger, bargaining, depression and finally acceptance
B.
Depression, denial, shock, anger, bargaining and finally acceptance
C.
Anger, bargaining, depression, shock, denial and finally acceptance
D.
Bargaining, shock, denial, anger, depression and finally acceptance
E.
Denial, bargaining, depression, anger, shock and finally acceptance
Q969. A patient who develops respiratory distress and hypoxemia a few hours after
platelet transfusion, due to antibody in the plasma against the recipient’s
leukocytes (Leukoagglutinins), is probably suffering from:
A
TRALI
“Transfusion-Related Acute Lung Injury”
B.
FNHTR
“Febrile Non Hemolytic Transfusion Reaction”
C.
HTR
“Hemolytic Transfusion Reaction
D.
Fluid Overload
E.
IgA deficiency
Q970. A multiply transfused patient who usually develops high fever after blood
transfusion, due to antibodies in the recipient’s plasma against the donor’s
leukocytes, is said to have:
A.
TRALI
“Transfusion-Related Acute Lung Injury”
B
FNHTR
“Febrile Non Hemolytic Transfusion Reaction”
C.
HTR
“Hemolytic Transfusion Reaction”
D.
Fluid Overload
E.
IgA deficiency
Q971. A frightening degree of anaphylaxis following blood transfusion could be due to:
A.
TRALI
“Transfusion-Related Acute Lung Injury”
B.
FNHTR
“Febrile Non Hemolytic Transfusion Reaction”
C.
HTR
“Hemolytic Transfusion Reaction”
D.
Fluid overload
E
IgA deficiency
Q972. A patient, who looks apprehensive after blood transfusion with tightness in the
chest, hypotension and loin pain, could have:
A.
TRALI
“Transfusion-Related Acute Lung Injury”
B.
FNHTR
“Febrile Non Hemolytic Transfusion Reaction”
C
HTR
“Hemolytic Transfusion Reaction”
D.
Fluid overload
E.
IgA deficiency
142
Professor Anwar Sheikha, MD, FRCP, FRCPath., FACP
One Thousand Hematology MCQ
Q973. A chronically anemic elderly patient, who becomes distressed after blood
transfusion, with shortness of breath and crackles at the base of the chest, is
probably suffering from:
A.
TRALI
“Transfusion-Related Acute Lung Injury”
B.
FNHTR
“Febrile Non Hemolytic Transfusion Reaction”
C.
HTR
“Hemolytic Transfusion Reaction”
D
Fluid overload
E.
IgA deficiency
Q974. The acclaimed Universal Hemostatic Agent Novo7 “rFVIIa” could benefit all but
one of the following bleeding crises:
A.
Intra-cerebral hemorrhage
B
Surgical bleeding from a major artery injury
C.
Factor VII deficiency bleeding
D.
Hemophilia A with inhibitor
E.
Glanzmann’s thrombasthenia
Q975. The most likely factor deficiency in a patient with a prolonged APTT of 100
seconds, who does not have any bleeding history even after major past surgery,
is:
A.
Factor VIII deficiency “Hemophilia A”
B.
Factor IX deficiency “Hemophilia B”
C.
von Willebrand Disease
D
Factor XII deficiency
E.
Factor VII deficiency
Q976. The ideal currently accepted management of a young adult patient with Hodgkin
lymphoma, who presents with a large bulky mediastinal mass, palpable
progressive painless cervical lymphadenopathy and splenomegaly, is :
A.
8 cycles of MOPP chemotherapy
B.
8 cycles of ABVD chemotherapy
C.
8 cycles of Hybrid MOPP/ABV chemotherapy
D.
Radiation therapy to all the involved sites
E
8 cycles of ABVD followed by mediastinal irradiation
Q977. To correct the anemia and replenish the stores of an 80 kg male patient with iron
deficiency anemia with a hemoglobin of 60 g/L, you need to infuse:
A.
0.5 gram of iron
B.
1.0 gram of iron
C
2.0 grams of iron
D.
5.0 grams of iron
E.
10.0 grams of iron
Q978. Management of a pregnant lady with habitual abortion and antiphospholipid
antibody syndrome could best be done using:
A.
Nightly Warfarin & Aspirin
B.
Steroid
C.
Anti-D
D.
Herudin
E
Daily Low Molecular Weight Heparin & Aspirin
143
Professor Anwar Sheikha, MD, FRCP, FRCPath., FACP
One Thousand Hematology MCQ
Q979. All but one of the followings could be considered for the management of a
pregnant polycythemia vera patient with a past history of deep vein thrombosis:
A
Hydroxyurea
B.
Low Molecular Weight Heparin
C.
Aspirin
D.
Venesections
E.
Interferon-α
Q980. Methemoglobinemia, following exposure to an offending agent, is best treated
with:
A.
Steroid
B.
Exchange transfusion
C
Methylene blue at the dose of 1 mg/kg i.v. over 5 minutes
D.
Immediate hyperbaric oxygen
E.
No treatment as it is a self-limiting condition
Q981. Cladribine “2-Chlorodeoxyadenosine” is an effective preferred initial
chemotherapeutic agent used in the management of:
A.
Diffuse Large B Cell Lymphoma
B
Hairy Cell Leukemia
C.
Primary CNS Lymphoma
D.
Hepatitis C infection
E.
Relapsed and refractory chronic lymphocytic leukemia
Q982. Alemtuzumab “Anti CD52” is used as the last option in the management of:
A.
Diffuse Large B Cell Lymphoma
B.
Hairy Cell Leukemia
C.
Primary CNS Lymphoma
D.
Hepatitis C infection
E
Relapsed and refractory chronic lymphocytic leukemia
Q983. Currently, Rituximab “Anti CD20” is an effective addition to the standard
chemotherapy regimen for:
A
Diffuse Large B Cell Lymphoma
B.
Hairy Cell Leukemia
C.
Primary CNS Lymphoma
D.
Hepatitis C infection
E.
Relapsed and refractory chronic lymphocytic leukemia
Q984. Interferon-α seems to be a widely used and effective drug in controlling:
A.
Diffuse Large B Cell Lymphoma
B.
Hairy Cell Leukemia as the first line of management
C.
Primary CNS Lymphoma
D
Hepatitis C infection
E.
Relapsed and refractory chronic lymphocytic leukemia
Q985. High dose methotrexate is an effective drug in the management of:
A.
Diffuse Large B Cell Lymphoma
B.
Hairy Cell Leukemia
C
Primary CNS Lymphoma
D.
Hepatitis C infection
E.
Relapsed and refractory chronic lymphocytic leukemia
144
Professor Anwar Sheikha, MD, FRCP, FRCPath., FACP
One Thousand Hematology MCQ
Q986. Zoledronic acid is a bisphosphonate that has been found to be effective in:
A.
Waldenström macroglobulinemia
B.
Acute Promyelocytic Leukemia
C.
HER2 over-expressed Breast Cancer
D.
PNH “Paroxysmal Nocturnal Hemoglobinuria”
E
Osteoporosis
Q987. Lenalidomide “Revlimid” is an immunomodulator that has been found to be
effective in:
A
Multiple Myeloma
B.
Acute Promyelocytic Leukemia
C.
HER2 over-expressed Breast Cancer
D.
PNH “Paroxysmal Nocturnal Hemoglobinuria”
E.
Osteoporosis
Q988. Eculizumab is a potentially effective drug in:
A.
Multiple Myeloma
B.
Acute Promyelocytic Leukemia
C.
HER2 over-expressed Breast Cancer
D
PNH “Paroxysmal Nocturnal Hemoglobinuria”
E.
Osteoporosis
Q989. Trastuzumab “Herceptin” is effective in:
A.
Multiple Myeloma
B.
Acute Promyelocytic Leukemia
C
HER2 over-expressed Breast Cancer
D.
PNH “Paroxysmal Nocturnal Hemoglobinuria”
E.
Osteoporosis
Q990. ATRA “All Transretinoic Acid” is currently a standard treatment for:
A.
Multiple Myeloma
B
Acute Promyelocytic Leukemia
C.
HER2 over-expressed Breast Cancer
D.
PNH “Paroxysmal Nocturnal Hemoglobinuria”
E.
Osteoporosis
Q991. The current preferred chemotherapy protocol for Diffuse Large
B- Cell Lymphoma “DLBL” is:
A.
ABVD “Adriamycin, Bleomycin, Vinblastine and Dacarbazine”
B.
Fludarabine, Cyclophosphamide and Rituximab
C.
Imatinib mesylate
D.
Daunorubicin & Ara-C
E
R-CHOP “Rituximab, Cyclophosphamide, Adriamycin, Oncovin &
Prednisolone”
Q992. The current preferred management for chronic myeloid leukemia is:
A.
ABVD “Adriamycin, Bleomycin, Vinblastine and Dacarbazine”
B.
Fludarabine, Cyclophosphamide and Rituximab
C
Imatinib mesylate
D.
Daunorubicin & Ara-C
E.
R-CHOP “Rituximab, Cyclophosphamide, Adriamycin, Oncovin &
Prednisolone”
145
Professor Anwar Sheikha, MD, FRCP, FRCPath., FACP
One Thousand Hematology MCQ
Q993. An effective management protocol for chronic lymphocytic leukemia is:
A.
ABVD “Adriamycin, Bleomycin, Vinblastine and Dacarbazine”
B.
FCR “Fludarabine, Cyclophosphamide and Rituximab” .
C.
Imatinib mesylate
D
Daunorubicin & Ara-C
E.
R-CHOP “Rituximab, Cyclophosphamide, Adriamycin, Oncovin &
Prednisolone”
Q994. The current standard management for Hodgkin lymphoma is:
A
ABVD “Adriamycin, Bleomycin, Vinblastine and Dacarbazine”
B.
FCR “Fludarabine, Cyclophosphamide and Rituximab”
C.
Imatinib mesylate
D.
Daunorubicin & Ara-C
E.
R-CHOP “Rituximab, Cyclophosphamide, Adriamycin, Oncovin &
Prednisolone”
Q995. The standard management for induction therapy in acute myeloid leukemia is:
A.
ABVD “Adriamycin, Bleomycin, Vinblastine and Dacarbazine”
B.
FCR “Fludarabine, Cyclophosphamide and Rituximab”
C.
Imatinib mesylate “Glivec”
D
Daunorubicin & Ara-C
E.
R-CHOP “Rituximab, Cyclophosphamide, Adriamycin, Oncovin &
Prednisolone”
Q996. Iron overload could be managed by all but one of the followings:
A.
Deferasirox “Exjade”
B.
Deferiprone “L1”
C.
Deferoxamine “Desferal”
D
Vitamin C
E.
Venesections
Q997. One of the followings is not true about Hepcidin:
A.
It controls intestinal iron absorption and macrophage iron recycling
B
It is down-regulated in iron overload
C.
It is down-regulated in thalassemia major
D.
It is down-regulated in iron deficiency
E.
Could be of future potential use as a drug in thalassemia major
Q998. The most appropriate management for an asymptomatic middle-aged woman
with 5q- syndrome (Myelodysplastic Syndrome) presenting with hemoglobin of
105 gm/L, platelet count of 600K/uL and mild macrocytosis is:
A.
ATRA “All Transretinoic Acid”
B.
Non-myeloablative hematopoietic stem cell transplantation
C.
Anagrelide to control her high platelet count
D
Regular outpatient follow-up without treatment
E.
Induction chemotherapy with Daunorubicin and Ara-C
146
Professor Anwar Sheikha, MD, FRCP, FRCPath., FACP
One Thousand Hematology MCQ
Q999. Granulocyte Colony Stimulating Factor (G-CSF) has been found to have one of
the following effects in acute myeloid leukemia patients receiving induction
chemotherapy with Daunorubicin and Ara-C chemotherapy:
A
Reduces the duration of neutropenia
B.
Improves complete remission rate
C.
Improves overall survival
D.
Reduces mortality rate
E.
Increases the risk of relapse
Q1000. The widely accepted International Prognostic Index (IPI) scoring system for
predicting prognosis in Diffuse Large B- cell Lymphoma utilizes all but one of the
following features:
A.
Old Age (> 60 years)
B.
Advanced stage (stage III & IV)
C
B- symptoms
D.
High LDH
E.
Poor performance status
Good Luck.
147
Professor Anwar Sheikha, MD, FRCP, FRCPath., FACP
One Thousand Hematology MCQ
Q000. t(9;22) is a characteristic genetic abnormality in one of the following
hematological malignancies:
A.
Burkitt Lymphoma
B.
Mantle cell lymphoma
C.
Follicular lymphoma
D
Chronic myeloid leukemia
E.
Acute promyelocytic leukemia
Q000. t(11:18) is a characteristic genetic abnormality in one of the following
hematological malignancies:
A
Burkitt Lymphoma
B.
Mantle cell lymphoma
C.
Follicular lymphoma
D.
Chronic myeloid leukemia
E.
Acute promyelocytic leukemia
Q000. t(11;14) is a characteristic genetic abnormality in one of the following
hematological malignancies:
A.
Burkitt Lymphoma
B
Mantle cell lymphoma
C.
Follicular lymphoma
D.
Chronic myeloid leukemia
E.
Acute promyelocytic leukemia
Q000. t(15;17) is a characteristic genetic abnormality in one of the following
hematological malignancies:
A.
Burkitt Lymphoma
B.
Mantle cell lymphoma
C.
Follicular lymphoma
D.
Chronic myeloid leukemia
E
Acute promyelocytic leukemia
Q000. Thrombocytopenia is least likely in an ICU patient to be due to:
A.
Drug exposure
B.
HELLP syndrome in a hypertensive pregnant patient
C.
HIT in a patient with pulmonary embolism
D
Myelodysplastic syndrome “5q- syndrome”
E.
Hemolytic uremic syndrome in a child with schistocytosis and renal
impairment
Q000. One of the following therapeutic agents is not effective in splenectomized
immune thrombocytopenic purpura:
A.
Prednisolone
B.
Dexamethasone
C.
Intravenous immunoglobulin
D
Anti-D
E.
Rituximab
148
Professor Anwar Sheikha, MD, FRCP, FRCPath., FACP
One Thousand Hematology MCQ
Q000. One of the following infectious agents is not known to contribute to the
development of lymphomas:
A
CMV “Cytomegalovirus”
B.
EBV “Epstein-Barr virus”
C.
HTLV-1 “Human T- Leukemia Virus type-1”
D.
Helicobacter pylori
E.
Mixed bacterial infections
Q000. One of the following infectious agents has been shown to contribute to the
development of IPSID “Immunoproliferative Small Intestinal Disease”:
A.
CMV “Cytomegalovirus”
B.
EBV “Epstein-Barr virus”
C.
HTLV-1 “Human T- Leukemia Virus type-1”
D.
Helicobacter pylori
E
Mixed bacterial infections
Q000. One of the following infectious agents has been shown to cause Adult T-cell
Leukemia/ Lymphoma:
A.
CMV “Cytomegalovirus”
B.
EBV “Epstein-Barr virus”
C
HTLV-1 “Human T- Leukemia Virus type-1”
D.
Helicobacter pylori
E.
Mixed bacterial infections
Q000. A dimorphic red cell picture could be seen in all of the following EXCEPT:
(A). Sideroblastic anemia
(B). A transfused iron deficient anemia patient
(C). Untreated iron deficiency anemia.
(D). A normal patient with a two month history of continuously bleeding piles
(E). A thalassemia patient who is recently transfused
Q000.
A.
B.
C.
D.
E.
Q000.
A.
B.
C.
D.
E.
149
Professor Anwar Sheikha, MD, FRCP, FRCPath., FACP
One Thousand Hematology MCQ
Q000.
A.
B.
C.
D.
E.
Q000.
A.
B.
C.
D.
E.
Q000.
A.
B.
C.
D.
E.
Q000.
A.
B.
C.
D.
E.
Q000.
A.
B.
C.
D.
E.
Q000.
A.
B.
C.
D.
E.
Q000.
A.
B.
C.
D.
E.
150
Professor Anwar Sheikha, MD, FRCP, FRCPath., FACP
One Thousand Hematology MCQ
Q000.
A.
B.
C.
D.
E.
Q000.
A.
B.
C.
D.
E.
Q000.
A.
B.
C.
D.
E.
151
Professor Anwar Sheikha, MD, FRCP, FRCPath., FACP
One Thousand Hematology MCQ
152
Professor Anwar Sheikha, MD, FRCP, FRCPath., FACP
One Thousand Hematology MCQ
College of Medical Sciences Examination
Dr. Anwar Sheikha
Fall 2002
Select the most appropriate answer:
Q1.
The special stain “Sudan Black” is particularly helpful in staining
the following leukemia cells:
A.
Myeloblasts.
B.
Lymphoblasts
C.
Monoblasts
D.
Megakaryoblasts
E.
T- Lymphoblasts
Q2.
The special stain “Periodic Acid Schiff = PAS” is particularly helpful in
staining the following leukemia cells:
A.
Myeloblasts
B.
Lymphoblasts.
C.
Monoblasts
D.
Megakaryoblasts
E.
T- Lymphoblasts
Q3.
The special stain “Non-Specific Butyrate Esterase” is particularly helpful
in staining the following leukemia cells:
A.
Myeloblasts
B.
Lymphoblasts
C.
Monoblasts.
D.
Megakaryoblasts
E.
T- Lymphoblasts
Q4.
The special stain “Acid Phosphatase” is particularly helpful in staining the
following leukemia cells:
A.
Myeloblasts
B.
Lymphoblasts
C.
Monoblasts
D.
Megakaryoblasts
E.
T- Lymphoblasts.
Q5.
The special stain “Specific Chloroacetate Esterase” is particularly helpful
in staining the following leukemia cells:
A.
Myeloblasts.
B.
Lymphoblasts
C.
Monoblasts
153
Professor Anwar Sheikha, MD, FRCP, FRCPath., FACP
D.
E.
One Thousand Hematology MCQ
Megakaryoblasts
T- Lymphoblasts
Q6.
The special stain “Myeloperoxidase” is particularly helpful in staining the
following leukemia cells:
A.
Myeloblasts.
B.
Lymphoblasts
C.
Monoblasts
D.
Megakaryoblasts
E.
T- Lymphoblasts
Q7.
Leukemia of the mature and maturing granulocytes is called:
A.
“CML”
Chronic Myeloid Leukemia.
B.
“CLL”
Chronic Lymphocytic Leukemia
C.
“ALL”
Acute Lymphoblastic Leukemia
D.
“AML”
Acute Myeloid Leukemia
E.
“M3 AML” Acute Hypergranular Promyelocytic Leukemia
Q8.
Leukemia of the morphologically mature lymphocytes is called:
A.
“CML”
Chronic Myeloid Leukemia
B.
“CLL”
Chronic Lymphocytic Leukemia.
C.
“ALL”
Acute Lymphoblastic Leukemia
D.
“AML”
Acute Myeloid Leukemia
E.
“M3 AML” Acute Hypergranular Promyelocytic Leukemia
Q9.
Leukemia of the Lymphoblasts is called:
A.
“CML”
Chronic Myeloid Leukemia
B.
“CLL”
Chronic Lymphocytic Leukemia
C.
“ALL”
Acute Lymphoblastic Leukemia.
D.
“AML”
Acute Myeloid Leukemia
E.
“M3 AML” Acute Hypergranular Promyelocytic Leukemia
Q10. Leukemia of the Myeloblasts is called:
A.
“CML”
Chronic Myeloid Leukemia
B.
“CLL”
Chronic Lymphocytic Leukemia
C.
“ALL”
Acute Lymphoblastic Leukemia
D.
“AML”
Acute Myeloid Leukemia.
E.
“M3 AML” Acute Hypergranular Promyelocytic Leukemia
Q11. Leukemia of Promyelocytes is called:
A.
“CML”
Chronic Myeloid Leukemia
B.
“CLL”
Chronic Lymphocytic Leukemia
C.
“ALL”
Acute Lymphoblastic Leukemia
D.
“AML”
Acute Myeloid Leukemia
154
Professor Anwar Sheikha, MD, FRCP, FRCPath., FACP
E.
One Thousand Hematology MCQ
“M3 AML” Acute Hypergranular Promyelocytic Leukemia.
Q12. In the FAB Classification of Acute Myeloid Leukemias,
M1 is the leukemia of the:
A.
Myeloblasts.
B.
Promyelocytes
C.
Monoblasts
D.
Megakaryoblasts
E.
Lymphoblasts
Q13. In the FAB Classification of Acute Myeloid Leukemias,
M3 is the leukemia of the:
A.
Myeloblasts
B.
Promyelocytes.
C.
Monoblasts
D.
Megakaryoblasts
E.
Lymphoblasts
Q14. In the FAB Classification of Acute Myeloid Leukemias,
M5 is the leukemia of the:
A.
Myeloblasts
B.
Promyelocytes
C.
Monoblasts.
D.
Megakaryoblasts
E.
Lymphoblasts
Q15. In the FAB Classification of Acute Myeloid Leukemias,
M7 is the leukemia of the:
A.
Myeloblasts
B.
Promyelocytes
C.
Monoblasts
D.
Megakaryoblasts.
E.
Lymphoblasts
Q16. In the FAB Classification of Acute Myeloid Leukemias,
M2 is the leukemia of the:
A.
Myeloblasts & Promyelocytes.
B.
Myeloblasts & Monoblasts
C.
Myeloblasts & Erythroblasts
D.
Myeloblasts & Lymphoblasts
E.
Myeloblasts & Megakaryoblasts
155
Professor Anwar Sheikha, MD, FRCP, FRCPath., FACP
One Thousand Hematology MCQ
Q17. In the FAB Classification of Acute Myeloid Leukemias,
M4 is the leukemia of the:
A.
Myeloblasts & Promyelocytes
B.
Myeloblasts & Monoblasts.
C.
Myeloblasts & Erythroblasts
D.
Myeloblasts & Lymphoblasts
E.
Myeloblasts & Megakaryoblasts
Q18. In the FAB Classification of Acute Myeloid Leukemias,
M6 is the leukemia of the:
A.
Myeloblasts & Promyelocytes
B.
Myeloblasts & Monoblasts
C.
Myeloblasts & Erythroblasts.
D.
Myeloblasts & Lymphoblasts
E.
Myeloblasts & Megakaryoblasts
Q19. The blast cells in L1 Acute Lymphoblastic Leukemia are:
A.
Small and monomorphic.
B.
large and heterogeneous
C.
Large & homogenous with basophilic vacuolated cytoplasm
D.
Convoluted
E.
Heavily granulated with multiple Auer rods
Q20. The blast cells in L2 Acute Lymphoblastic Leukemia are:
B.
Small and monomorphic
B.
large and heterogeneous.
C.
Large & homogenous with basophilic vacuolated cytoplasm
D.
Convoluted
E.
Heavily granulated with multiple Auer rods
Q21. The blast cells in L3 Acute Lymphoblastic Leukemia are:
C.
Small and monomorphic
B.
large and heterogeneous
C.
Large & homogenous with basophilic vacuolated cytoplasm .
D.
Convoluted
E.
Heavily granulated with multiple Auer rods
Q22. The blast cells in T- Acute Lymphoblastic Leukemia are usually:
D.
Small and monomorphic
B.
small with cleaved nuclei
C.
Large & homogenous with basophilic vacuolated cytoplasm
D.
Convoluted.
156
Professor Anwar Sheikha, MD, FRCP, FRCPath., FACP
E.
One Thousand Hematology MCQ
Heavily granulated with multiple Auer rods
Select the most appropriate answer for the following red cell morphological
abnormalities or the disease associated with their presence:
Q1.
Presence of cells with this morphology in a child with severe pain suggests:
A.
Anisocytosis
B.
Poikilocytosis
C.
Thalassemia
D.
Sickle Cell anemia
E.
Hereditary spherocytosis
Q1.
Presence of cells with this morphology in a child with severe hypochromic
microcytic anemia, high HbF and huge spleen suggests:
A.
Anisocytosis
B.
Poikilocytosis
C.
Thalassemia
D.
Sickle Cell anemia
E.
Hereditary spherocytosis
Q3.
Variation in red cell morphology as seen here is called:
A.
Anisocytosis
B.
Poikilocytosis
C.
Thalassemia
D.
Sickle Cell anemia
E.
Hereditary spherocytosis
Q4.
Variation in red cell morphology as seen here is called:
A.
Anisocytosis
B.
Poikilocytosis
C.
Thalassemia
D.
Sickle Cell anemia
E.
Hereditary spherocytosis
Q5.
Presence of small red cells with this morphology in a patients with
hereditary hemolytic anemia is called:
A.
Anisocytosis
B.
Poikilocytosis
C.
Thalassemia
157
Professor Anwar Sheikha, MD, FRCP, FRCPath., FACP
D.
E.
Q14.
One Thousand Hematology MCQ
Sickle Cell anemia
Hereditary spherocytosis
The following anticoagulants cannot be used in collecting blood samples:
A.
EDTA (Ethylene diamine tetra acetic acid)
B.
Heparin
C.
Citrate
D.
All of the above.
E.
None of the above
MATCH EACH OF THE FOLLOWING DISEASES WITH ITS RELATED
MORPHOLOGICAL HEMATOLOGICAL CHANGES.
EACH CHOICE COULD BE USED ONCE ONLY.
Q51.
Q52.
Q53.
Q54.
Q55.
Q56.
Q57.
Q58.
Q59.
Q60.
MYELOFIBROSIS
HODGKIN’S DISEASE
MULTIPLE MYELOMA
ECZEMA
ANGIOCENTRIC NHL
WALDENSTROM’S
HIGH GRADE LYMPHOMA
ITP
KALA AZAR
ESSENTIAL THROMBOCYTHEMIA
(
(
(
(
(
(
(
(
(
(
)
)
)
)
)
)
)
)
)
)
A.
B.
C.
D.
E.
F.
G.
H.
I.
J.
INCREASED PLASMA CELLS
NASAL LYMPHOMA OF NK CELLS
↑ LYMPHOPLASMACYTOID CELLS
EOSINOPHILIA
LACUNAR REED STERNBERG CELLS
PROLIFERATION OF LYMPHOBLASTS
TEAR DROP RED CELLS
VERY HIGH PLATELET COUNT
LOW PLATELET COUNT
MARROW LEISHMAN DONOVAN BODIES
MATCH EACH BLEEDING DISORDER WITH ITS RELATED FEATURE.
Q61. VON WILLEBRAND DISEASE
Q62. HEMOPHILIA A
Q63. ITP
Q64. FACTOR XII DEFICIENCY
Q65. CHRISTMAS DISEASE
(
(
(
(
(
)
)
)
)
)
K.
L.
M.
N.
O.
LOW FACTOR VIII:c
LOW PLATELET COUNT
FACTOR IX DEFICIENCY
THROMBOSIS
PROLONGED APTT &
BLEEDING TIME
158
Professor Anwar Sheikha, MD, FRCP, FRCPath., FACP
One Thousand Hematology MCQ
Match the following diseases with the most appropriate features:
Each feature should be used once only; Put the right letter in the ANSWER BOX.
No
101
102
103
104
105
106
107
108
109
110
111
112
113
114
115
116
117
118
ANSWER
DISEASE
Hodgkin’s Disease
Myelofibrosis
Multiple Myeloma
Polycythemia
Rubra Vera
Nodular sclerosis HD
Burkitt’s Lymphoma
Essential
Thrombocythemia
Amyloidosis
Tropical
splenomegaly syndrome
T.T.P.
Glanzmann’s Disease
ITP (Immune Thrombocytopenic Purpura)
Mycosis Fungoides
von Willebrand Disease
Waldenstrom’s macroglobulinemia
Hemophilia A
AML (M3)
Centrocytic lymphoma
FEATURES
Prolonged APTT and bleeding time
Osteolytic lesions
IgM Gammopathy
Bleeding with low platelets &
marrow megakaryocytosis
Reed Sternberg cells
Follicular lymphoma
High Hb, Hct. & Red cell count.
Splenomegaly
Factor VIII:c Deficiency
Thrombosis, thrombocytopenia,
CNS symptoms, fever & MAHA.
Lacunar cells
Platelet GP IIb & IIIa deficiency
Tear drop red cells &
extramedullary hemopoiesis
Marked Malarial splenomegaly
plus high polyclonal IgM
Cutaneous T Cell Lymphoma
Thrombocytosis with splenic
atrophy
Macroglossia
Associated with D.I.C.
EBV & starry sky histology
A
B
C
D
E
F
G
H
I
J
K
L
M
N
O
P
Q
R
SELECT THE MOST APPROPRIATE ANSWER.
159
Professor Anwar Sheikha, MD, FRCP, FRCPath., FACP
One Thousand Hematology MCQ
Abbreviations
in
Hematology
#
1
2
3
4
5
6
7
8
9
10
11
12
13
14
15
Abbreviation
ALL
AML
CML
CLL
MPD
MDS
MM
TTP
HUS
MAHA
SLE
ITP
AIHA
ESR
HEMPAS
16
AGT
17 Ph’ Chromosome
18
LAP Score
19
G6P DH
20
2,3 DPG
Full Name of the Disease
Acute Lymphoblastic Leukemia
Acute Myeloid Leukemia
Chronic Myeloid Leukemia
Chronic Lymphocytic Leukemia
Myeloproliferative Disorders
Myelodysplastic Syndromes
Multiple Myeloma
Thrombotic Thrombocytopenic Purpura
Hemolytic Uremic Syndrome
Micro-angiopathic Hemolytic Anemia
Systemic Lupus Erythematosus
Immune Thrombocytopenic Purpura
Autoimmune Hemolytic Anemia
Erythrocyte Sedimentation rate
Hereditary Erythroid Multinuclearity
with Positive Acidified Serum test
Anti-Human Globulin “Coomb’s” Test
Philadelphia Chromosome
Leukocyte Alkaline Phosphatase
Glucose 6 Phosphate Dehydrogenase
2,3 Diphosphoglycerate
160
Professor Anwar Sheikha, MD, FRCP, FRCPath., FACP
21 P.T
22 A.P.T.T.
23 T.T.
24
RA
25
RARS
26
RAEB
One Thousand Hematology MCQ
Prothrombin Time
Activated Partial Thromboplastin Time
Thrombin Time
Refractory Anemia
Refractory Anemia with Ring Sideroblasts
Refractory Anemia with Excess Blasts
Good Luck.
Match the following (lettered) hematological features with the type of anemia you
think it is compatible. Each choice is used once only.
(A). A fifteen year old patient admitted with fever, anemia and bleeding. There is history
of Chloramphenicol medication. Bone marrow was found to be empty.
(B). A fifty year old male patient with uremia and a hemoglobin of 9 gm/dL.
MCV, MCH and MCHC are all within normal limits.
(C). A ten year old child with a history of frequent blood transfusion. His spleen is very
big and Hemoglobin F very high. Blood picture showed hypochromic microcytic anemia
with many target cells.
(D). A Saudi out-patient with a hemoglobin of 10 gm/dL and moderately palpable
spleen. The blood smear showed red cells to be oval in shape. MCV and MCH are low
but MCHC is high. No treatment given.
(E). A multiparous lady from Tihama with ten children and poor nutritional history.
Her hemoglobin is 5 gm/dL and MCV 112 fL. She has never taken iron or Folic Acid
during her pregnancies.
(F). A poor lady from Tihama with ten children and severe anemia. Blood smear showed
hypochromic microcytic red cells. Serum Ferritin was very low.
(G). A normal looking male out-patient with a hemoglobin of 11 gm/dL and many target
cells in the blood smear. The Hemoglobin A2 was 8%. Serum ferritin was normal.
(H). A child with severe anemia and mild jaundice. Coomb’s test was positive. He
responded to Steroid.
(I). A black Saudi child with history of repeated hospital admissions for painful crises.
Her hemoglobin was 7 gm/dL and bilirubin was high. Spleen was atrophic. Hemoglobin
electrophoresis was abnormal with no Hemoglobin A, A2 or F.
(J). An Egyptian child admitted for severe hemolytic anemia after eating “Fool”. Blood
smear showed many blister cells. Fool in Arabic means bean.
(141).
(142).
(143).
(144).
(145).
Megaloblastic Anemia
Iron Deficiency Anemia
Thalassemia Minor
Sickle Cell Disease
G6PDH Deficiency
……. (141)
……..(142)
……..(143)
……..(144)
……..(145)
161
Professor Anwar Sheikha, MD, FRCP, FRCPath., FACP
(146).
(147).
(148).
(149).
(150).
One Thousand Hematology MCQ
……..(146)
……..(147)
……..(148)
……..(149)
……. (150)
Autoimmune Hemolytic Anemia
Hereditary Elliptocytosis
Thalassemia Major
Anemia of Chronic Disorder
Aplastic Anemia
Select the most appropriate answer:
MATCH THE FOLLOWING HEMATOLOGICAL CONDITIONS WITH THEIR
APPROPRIATE CORRECT ASSOCIATION, DESCRIPTION OR DEFINITIONS; ONLY
ONE OF THE ASSOCIATIONS IS CORRECT:
Q 34. LEUKEMIAS AND AGE INCIDENCE :
Acute Lymphoblastic Leukemia
Acute Myeloblastic Leukemia
Chronic Myeloid Leukemia
Chronic Lymphoid Leukemia
Hairy Cell Leukemia
.......
.......
.......
.......
.......
(A).
(B).
(C).
(D).
(E)
Elderly
Childhood
Middle Age
Young Adult & Elderly
Middle Aged Male
Q 35. ANEMIAS AND RELATED DIAGNOSTIC HEMOGLOBIN TYPE :
β- thalassemia major
.......
(A). Hemoglobin A2
α- thalassemia intermedia
.......
(B). Hemoglobin F
Sickle cell disease
.......
(C). Hemoglobin H
β- thalassemia minor
.......
(D). Hemoglobin S
Uncontrolled diabetes
.......
(E)
Glycated Hemoglobin
Q 36. LYMPHOMAS AND DIAGNOSTIC MALIGNANT CELLS:
Hodgkin's Disease
.......
(A). Small lymphocytes
Small lymphocytic lymphoma
.......
(B). T-cells
Angiocentric lymphoma
.......
(C)
NK cells
Waldenstrom’s Lymphoma
.......
(D). Lymphoplasmacytoid cells
Mycosis fungoides
.......
(E). Reed Sternberg cells
Q 37. TYPE OF ANEMIA AND MEAN CORPUSCULAR VOLUME (MCV) :
Iron deficiency anemia
....... (A). 120 fL
Thalassemia major
....... (B).
55 fL + (MCHC 38%)
Megaloblastic anemia
....... (C).
55 fL + (Ferritin++)
Hereditary spherocytosis
....... (D)
55 fL
Chronic renal failure
....... (E). 106 fL
Q 38. TYPE OF BLEEDING AND DEFECTIVE HEMOSTATIC FACTORS:
von Willebrand disease
....... (A). Factor VIII:c
Hemophilia A
....... (B). Factor IX
Christmas disease
....... (C). II,VII,IX & X
Hemorrhagic disease of newborn ....... (D). von Willebrand Factor
DIC
....... (E). Most Factors
Q 39. COMPLICATIONS OF BLOOD TRANSFUSION AND ETIOLOGY :
Febrile transfusion reaction
....... (A). RBC lysis
Anaphylaxis
....... (B). Infected blood
Hemolytic transfusion reaction
....... (C). Donor WBC destruction
162
Professor Anwar Sheikha, MD, FRCP, FRCPath., FACP
Hypocoagulability & bleeding
TRALI
.......
.......
One Thousand Hematology MCQ
(D).
(E)
Massive transfusion
Recipient WBC destruction
Q 40. WHITE CELL NUMBER ABNORMALITY & RECOGNIZED ETIOLOGY :
Eosinophilia
....... (A). Typhoid fever
Neutropenia
....... (B). CMML
Atypical lymphocytosis
....... (C). Asthma
Monocytosis
....... (D). Infectious mononucleosis
Leukemoid reaction
....... (E)
Severe Infection
Q. FILL OUT THE FOLLOWING NORMAL HEMATOLOGICAL VALUES
All the blanks and the ones signed with the question mark (?)
MEN
------
WOMEN
------------
INFANT(Cord)
-------------------
CHILD (3 months)
-----------------------
Red cell Count
(+/- 1.0 x 1012/L)
Hemoglobin
(gm/dL)
Hematocrit (PCV)
MCV (fL)
---------------------------------------------------------------------------------------------------------------?
0.45 x 1015 (fL)
450
Mean Cell Volume (MCV) = ----------= -------------------- = ------- = 90 fL
?
5.0 x 1012
5
Adult = ?
-----------------------------------------------------------------------------------------------------------------?
15 gm/dL
150 gm/L
150 x 1012 (pcg)/L
Mean Cell Hemoglobin = ---------- = --------------- = ------------- == ----------------------= 30
(MCH)
?
5.0 x 1012/L
5.0 x 1012
5 x 1012 /L
pcg
Adult = ?
---------------------------------------------------------------------------------------------------------------Hb
150 gm/L
Mean Cell Hemoglobin Concentration (MCHC) = -------- = ------------ = 333 g/L = 33 g/dL
Adult and children = ?
PCV
0.45 L/L
-----------------------------------------------------------------------------------------------------------------Reticulocyte =
? - ? % = 10 000 - 100 000 / uL (Adults and children)
2.0 - 6.0% Mean = 150 000 / uL (Infant = cord blood)
----------------------------------------------------------------------------------------------------------------BLOOD VOLUME : Red Cell Volume
MEN ...………….
?
ml/Kg
WOMEN .………
?
ml/Kg
Plasma Volume ...........…………………
?
ml/Kg
Total Blood Volume ------------------------ ?
ml/Kg
___________________________________________________________________________
LEUKOCYTE COUNT: 4.0-11.0x109/L
Red cell life span = ?
days
163
Professor Anwar Sheikha, MD, FRCP, FRCPath., FACP
One Thousand Hematology MCQ
= 4 000 - 11 000/uL
Neutrophils = ?
/uL
Lymphocyte = ?
/uL
Monocyte = 200 - 800/uL
Eosinophil =
40 - 440/uL
Basophil =
1 - 100/uL
Platelet count = ?
/uL
ESR = < ?
mm/hr
Platelet life span = ?
days
Neutrophil half life = 7 hours
Serum Iron = 13-32 umol/L (100)
TIBC = 45-70 umol/L(300 ug/dL)
Serum Ferritin =
15-300 ug/L
Male = 30-300 ug/L
Female = 15-200 ug/L
ANSWER
Red cell Count
MEN
5.5
WOMEN
4.8
INFANT(Cord)
5.0
CHILD (3 months)
4.0
13.0-17.0
11.5-16.5
13.5-19.5
9.5-13.5
40%-54%
76-96
37%- 47%
76-96
44%-64%
106
32%-44%
96
(+/- 1.0 x 1012/L)
Hemoglobin
(gm/dL)
Hematocrit (PCV)
MCV (fL)
PCV
0.45 x 1015 (fL)
Mean Cell Volume (MCV) = ------- = ---------------------RCC
5.0 x 1012
Adult = 76 – 96 fL
450
= ---------- = 90 fL
5
Hb
15 gm/dL
150 gm/L
150 x 1012 (pcg)/L
Mean Cell Hemoglobin = ------- = --------------- = ------------- == ----------------------= 30 pcg
(MCH)
RCC
5.0 x 1012/L
5.0 x 1012
5 x 1012 /L
Adult = 27-32 pcg
-----------------------------------------------------------------------------------------------------------------Hb
150 gm/L
Mean Cell Hemoglobin Concentration (MCHC) = -------- = ------------- = 333 g/L = 33 g/dL
Adult and children = 32-36 g/dL
PCV
0.45 L/L
-----------------------------------------------------------------------------------------------------------------Reticulocyte = 0.2 - 2.0%
= 10 000 - 100 000 / uL (Adults and children)
2.0 - 6.0%
Mean = 150 000 / uL (Infant = cord blood)
-----------------------------------------------------------------------------------------------------------------BLOOD VOLUME : Red Cell Volume
MEN ...………….
25 - 35 ml/Kg
WOMEN .………
20 - 30 ml/Kg
Plasma Volume ...........…………………
40 - 50 ml/Kg
Total Blood Volume ------------------------ 60 - 80 ml/Kg
_____________________________________________________________________________
LEUKOCYTE COUNT: 4.0-11.0x109/L
= 4 000 - 11 000/uL
Neutrophils =
2 000 - 7 500/uL
Red cell life span = 120 days
Platelet life span = ~ 10 days
Neutrophil half life = 7 hours
164
Professor Anwar Sheikha, MD, FRCP, FRCPath., FACP
Lymphocyte =
Monocyte =
Eosinophil =
Basophil =
One Thousand Hematology MCQ
1 500 - 4 000/uL
200 - 800/uL
40 - 440/uL
1 - 100/uL
Serum Iron = 13-32 umol/L (100)
TIBC = 45-70 umol/L(300 ug/dL)
Serum Ferritin =
15-300 ug/L
Male = 30-300 ug/L
Female = 15-200 ug/L
Platelet count = 150 000 - 400 000/uL
ESR = < 20 mm/hr
ANSWERS
TO
500 EXAM BANK
The answers to most of the multiple choice questions could be found in the
attached five pages.
The answers to questions that cannot follow the rules of ABCDE is
provided here:
Q33.
Q34.
Q35.
Q36.
Q37.
Q38.
Q39.
Q40.
Q41.
Q42.
BDCAE
BCDAE
EACDB
DCABE
DABCE
CBADE
CADBE
ABCDE
AEBCD
EDCBA
Q51.
Q52.
Q53.
Q54.
Q55.
Q56.
Q57.
Q58.
Q59.
Q60
G
E
A
D
B
C
F
I
J
H
Q61.
Q62.
Q63.
Q64
Q65.
O
K
L
N
M
Q141.
Q142.
Q143.
Q144.
Q145.
Q146.
Q147.
Q148.
Q149 .
Q150.
E
F
G
I
J
H
D
C
B
A
::::::::::::::::::::::::::::::::::::::::::::::::::::::::::::::::::::::::::::::::::::::::::::::::::::::::::::::::::::::::::::::::::::
Q271.
Q271.
Q273.
Q274.
Q275.
Q276.
Q277.
Q278.
Q279.
Q280.
I
F
A
H
E
J
G
D
C
B
Q291.
Q292.
Q293.
Q294.
Q295.
Q296.
Q297.
Q298.
Q299.
Q300.
P
Q
L
M
N
J
K
R
T
S
165
Professor Anwar Sheikha, MD, FRCP, FRCPath., FACP
Q281.
Q282.
Q283.
Q284.
Q285.
Q286.
Q287.
Q288.
Q289.
Q290.
I
H
C
B
D
E
A
F
G
O
Q301.
Q302.
Q303.
Q304.
Q305.
One Thousand Hematology MCQ
Y
X
W
V
U
Match the following diseases with the most appropriate features:
Each feature should be used once only; Put the right letter in the ANSWER BOX.
No
101
102
103
104
105
106
107
108
109
110
111
112
113
114
115
116
117
118
DISEASE
Hodgkin’s Disease
Myelofibrosis
Multiple Myeloma
Polycythemia
Rubra Vera
Nodular sclerosis HD
Burkitt’s Lymphoma
Essential
Thrombocythemia
Amyloidosis
Tropical
splenomegaly syndrome
T.T.P.
Glanzman’s Disease
ITP (Immune Thrombocytopenic Purpura)
Mycosis Fungoides
ANSWER
von Willebrand Disease
Waldenstrom’s macroglobulinemia
Hemophilia A
AML (M3)
Centrocytic lymphoma
A
C
E
L
B
G
J
R
O
P
M
I
K
D
N
H
Q
F
FEATURES
Prolonged APTT and bleeding time
Osteolytic lesions
IgM Gammopathy
Bleeding with low platelets &
marrow megakaryocytosis
Reed Sternberg cells
Follicular lymphoma
High Hb, Hct. & Red cell count.
Splenomegaly
Factor VIII:c Deficiency
Thrombosis, thrombocytopenia,
CNS symptoms, fever & MAHA.
Lacunar cells
Platelet GP IIb & IIIa deficiency
Tear drop red cells &
extramedullary hemopoiesis
Marked Malarial splenomegaly
plus high polyclonal IgM
Cutaneous T Cell Lymphoma
Thrombocytosis with splenic
atrophy
Macroglossia
Associated with D.I.C.
EBV & starry sky histology
A
B
C
D
E
F
G
H
I
J
K
L
M
N
O
P
Q
R
Q337.
DISEASE OR FACTOR
DEFICIENCY
PT
APTT
TT
B.T.
HEMOPHILIA
(N)
N
(P)
N or P
N
(N)
N
(P)
vWD
166
Professor Anwar Sheikha, MD, FRCP, FRCPath., FACP
One Thousand Hematology MCQ
XII & XI
N
(P)
N
N
VII
(P)
(N)
N
X, V & II
(P)
P
(N)
FIBRINOGEN
(P)
P
(N)
N
LIVER FAILURE
(P)
DIC
ITP
351
352
353
354
355
356
357
358
N
P
(P)
P
(P)
(P)
(P)
(P)
(N)
(N)
(N)
(P)
BLOOD DISEASE
UREMIA
HEREDITARY SPHEROCYTOSIS
G6P DEHYDROGENASE DEFICIENCY
HEREDITARY ELLIPTOCYTOSIS
MAHA “Microangiopathic Hemolytic Anemia”
MYELOFIBROSIS
SICKLE CELL ANEMIA
IRON DEFICIENCY ANEMIA
359 LEAD POISONING
360 MEGALOBLASTIC ANEMIA
ALL
Q371
Q372
Q373
Q374
Q375
Pro-B
cALL
Pre-B
B-ALL
T-ALL
TdT
+
+
+
+
CD10
+
+
+/-
Cyt. 
+
+
-
POSSIBLE
RED CELL ABNORMALITY
J. BURR CELLS
I. SPHEROCYTES
H. HEINZ BODIES
G. ELLIPTOCYTES
A. SHISTOCYTES
B. TEAR DROP RED CELLS
C. SICKLE CELLS
F. HYPOCHROMIC
MICROCYTIC RBC
E. BASOPHILIC STIPPLING
D. HOWELL-JOLLY BODIES
SmIg CD19-22
+
-
N
CD 2-8
(B-Lineage)
(T-Lineage)
+
+
+
+
-
+
Answers to the rest of the exercise will be found in the attached papers.
167
Professor Anwar Sheikha, MD, FRCP, FRCPath., FACP
One Thousand Hematology MCQ
CLINICAL HEMATOLOGY EXAMINATION
Fifth Year Medical Students
February 2008
Q1.
R-CHOP “Rituximab plus Cyclophosphamide, Adriamycin, Vincrisitne &
Prednisolone” is a chemotherapeutic protocol effectively used in the
management of:
A.
Multiple Myeloma
B.
Non Hodgkin Lymphoma
C.
Chronic lymphocytic leukemia
D.
Acute myeloid leukemia
Q2.
ABVD is a chemotherapeutic protocol effectively used in the management of
Hodgkin lymphoma:
A. Multiple Myeloma
B. Non Hodgkin Lymphoma
C. Chronic lymphocytic leukemia
D. Acute myeloid leukemia
Q3.
Danorubicin and Ara-C (3+7) is a chemotherapeutic protocol effectively used in
the management of:
A. Multiple Myeloma
B. Non Hodgkin Lymphoma
C. Chronic lymphocytic leukemia
D. Acute myeloid leukemia
E. Hodgkin lymphoma
Q4.
VAD “Vinblastine, Adriamycin and Dexamethasone” is a chemotherapeutic
protocol effectively used in the management of:
A. Multiple Myeloma
B. Non Hodgkin Lymphoma
C. Chronic lymphocytic leukemia
D. Acute myeloid leukemia
E. Hodgkin lymphoma
Q5. FCR “Fludarabine, Cyclophosphamide & Rituximab is a chemotherapeutic protocol
effectively used in the management of:
A. Multiple Myeloma
B. Non Hodgkin Lymphoma
C. Chronic lymphocytic leukemia
168
Professor Anwar Sheikha, MD, FRCP, FRCPath., FACP
One Thousand Hematology MCQ
D. Acute myeloid leukemia
E. Hodgkin lymphoma
Q6-10. Match the following hemoglobin electrophoresis pattern with the corresponding
hemoglobinopathy. Each choice is used once only.
Q06.
Q07.
Q08.
Q09.
Q10.
β- thalassemia major
Sickle cell disease
β - thalassemia minor
Sickle cell trait
α- thalassemia
…….
…….
…….
…….
…….
(A).
(B).
(C).
(D).
(E).
Hemoglobin F of 100%
Hemoglobin S of 100%
Hemoglobin A2 of 8%
Hemoglobin S of 40%
Hemoglobin H is increased.
Q11. A bleeding child can have any, but one, of the following diseases::
A.
Aplastic Anemia
B
Multiple Myeloma
C.
Acute Leukemia
D.
Immune Thrombocytopenic Purpura
E.
Hemophilia
Q12. Which one of the following is regarded as "HIGH GRADE" lymphoma:
A.
B.
C.
D
E.
Small lymphocytic Lymphoma
Lymphocyte Rich Hodgkin Lymphoma
Mantle cell Lymphoma
Lymphoblastic Lymphoma
Lympho-plasmacytic Lymphoma
Q13. All of the following cells are regarded as primitive hematological cells EXCEPT:
A
Megakaryocytes
B.
Myeloblasts
C.
Pronormoblasts
D.
Monoblasts
E.
Lymphoblasts
Q14. In immunophenotyping of hematological malignancies, positive reaction with
CD3 is highly specific for:
A.
B-cell malignancy
B
T-cell malignancy
C.
Myeloid leukemias
D.
Monoblastic leukemia
E.
Neuroblastoma
Q15. In Iron deficiency anemia:
A.
MCV is low
B.
MCH is low
C.
MCHC is low
169
Professor Anwar Sheikha, MD, FRCP, FRCPath., FACP
D
E.
One Thousand Hematology MCQ
All of the above
None of the above
Prescriptions:

Write down a prescription for a ten year old sickle cell patients appearing in your
clinic at a steady state.

Write down an R-CHOP prescription for a 50 year old patient with Diffuse Large
B-Cell Lymphoma (DLBL) as you would want to appear in the hospital treatment
chart. Patient weighs 66 kilograms and is 178 cm tall.

Write down an intravenous anti-D prescription for an Rh-negative patient with
immune thrombocytopenic purpura who did not respond optimally to
splenectomy.
Answer: It cannot be given to splenectomized and Rh-negative patients

170